You are on page 1of 154

INSURANCE G01 CASE DIGESTS At the trial, petitioner presented communications of the insurance adjuster to Asian Surety revealing

undeclared co-insurances with the following: P30,000 with Wellington Insurance; P25,000 with Empire Surety and P250,000 with Asian Surety undertaken by insured Paramount on the same property covered by its policy with Oriental whereas the only co-insurances declared in the subject policy are those of P30,000.00 with Malayan P50,000.00 with South Sea and P25.000.00 with Victory. The defense of fraud, in the form of non-declaration of coinsurances which was not pleaded in the answer, was also not pleaded in the Motion to Dismiss. The trial court denied the respondents motion. Oriental filed another motion to include additional evidence of the co-insurance which could amount to fraud. The trial court still made Oriental liable for P 61,000. The CA reversed the trial court decision. Pacific Banking filed a motion for reconsideration of the said decision of the respondent Court of Appeals, but this was denied for lack of merit. Issues: 1. WON unrevealed co-insurances Violated policy conditions No. 3 2. WON the insured failed to file the required proof of loss prior to court action. Held: Yes. Petition dismissed. Ratio: 1. Policy Condition No. 3 explicitly provides: 3. The Insured shall give notice to the Company of any insurance already effected, or which may subsequently be effected, covering any of the property hereby insured, and unless such notice be given Page 1 of 154

POLICY
67. RCBC V. CA (FULL CASE POSTED IN THE FB GROUP - ADDITIONAL CASE - PLEASE READ ON YOUR OWN- FIRST CASE TO BE DISCUSSED) PACIFIC BANKING V . CA, 168 SCRA 1 (1988)

68.

Facts: An open fire insurance policy, was issued to Paramount Shirt Manufacturing by Oriental Assurance Corporation to indemnify P61,000.00, caused by fire to the factorys stocks, materials and supplies. The insured was a debtor of Pacific Banking in the amount of (P800,000.00) and the goods described in the policy were held in trust by the insured for Pacific Banking under trust receipts. The policy was endorsed to Pacific Banking as mortgagee/ trustor of the properties insured, with the knowledge and consent of private respondent to the effect that "loss if any under this policy is payable to the Pacific Banking Corporation". A fire broke out on the premises destroying the goods contained in the building. The bank sent a letter of demand to Oriental for indemnity. The company wasnt ready to give since it was awaiting the adjusters report. The company then made an excuse that the insured had not filed any claim with it, nor submitted proof of loss which is a clear violation of Policy Condition No.11, as a result, determination of the liability of private respondent could not be made. Pacific Banking filed in the trial court an action for a sum of money for P61,000.00 against Oriental Assurance.

INSURANCE G01 CASE DIGESTS and the particulars of such insurance or insurances be stated in or endorsed on this Policy by or on behalf of the Company before the occurrence of any loss or damage, all benefit under this policy shall be forfeited. The insured failed to reveal before the loss three other insurances. Had the insurer known that there were many co-insurances, it could have hesitated or plainly desisted from entering into such contract. Hence, the insured was guilty of clear fraud. Concrete evidence of fraud or false declaration by the insured was furnished by the petitioner itself when the facts alleged in the policy under clauses "Co-Insurances Declared" and "Other Insurance Clause" are materially different from the actual number of coinsurances taken over the subject property. As the insurance policy against fire expressly required that notice should be given by the insured of other insurance upon the same property, the total absence of such notice nullifies the policy. Petitioner points out that Condition No. 3 in the policy in relation to the "other insurance clause" supposedly to have been violated, cannot certainly defeat the right of the petitioner to recover the insurance as mortgagee/assignee. Hence, they claimed that the purpose for which the endorsement or assignment was made was to protect the mortgagee/assignee against any untoward act or omission of the insured. It would be absurd to hold that petitioner is barred from recovering the insurance on account of the alleged violation committed by the insured. It is obvious that petitioner has missed all together the import of subject mortgage clause which specifically provides: Loss, if any, under this policy, shall be payable to the PACIFIC BANKING CORPORATION Manila mortgagee/trustor as its interest may appear, it being hereby understood and agreed that this insurance as to the interest of the mortgagee/trustor only herein, shall not be invalidated by any act or neglectexcept fraud or misrepresentation, or arsonof the mortgagor or owner/trustee of the property insured; provided, that in case the mortgagor or owner/ trustee neglects or refuses to pay any premium, the mortgagee/ trustor shall, on demand pay the same. The paragraph clearly states the exceptions to the general rule that insurance as to the interest of the mortgagee, cannot be invalidated; namely: fraud, or misrepresentation or arson. Concealment of the aforecited co-insurances can easily be fraud, or in the very least, misrepresentation. Undoubtedly, it is but fair and just that where the insured who is primarily entitled to receive the proceeds of the policy has by its fraud and/or misrepresentation, forfeited said right. Petitioner further stressed that fraud which was not pleaded as a defense in private respondent's answer or motion to dismiss, should be deemed to have been waived. It will be noted that the fact of fraud was tried by express or at least implied consent of the parties. Petitioner did not only object to the introduction of evidence but on the contrary, presented the very evidence that proved its existence. 2. Generally, the cause of action on the policy accrues when the loss occurs, But when the policy provides that no action shall be brought unless the claim is first presented extrajudicially in the manner provided in the policy, the cause of action will accrue from the time the insurer finally rejects the claim for payment In the case at bar, policy condition No. 11 specifically provides that the insured shall on the happening of any loss or damage give notice to the company and shall within fifteen (15) days after such loss or damage deliver to the private respondent (a) a claim in writing giving particular account as to the articles or goods destroyed and the amount of the loss or damage and (b) particulars of all other insurances, if any. Page 2 of 154

INSURANCE G01 CASE DIGESTS Twenty-four days after the fire did petitioner merely wrote letters to private respondent to serve as a notice of loss. It didnt even furnish other documents. Instead, petitioner shifted upon private respondent the burden of fishing out the necessary information to ascertain the particular account of the articles destroyed by fire as well as the amount of loss. Since the required claim by insured, together with the preliminary submittal of relevant documents had not been complied with, it follows that private respondent could not be deemed to have finally rejected petitioner's claim and therefore there was no cause of action. It appearing that insured has violated or failed to perform the conditions under No. 3 and 11 of the contract, and such violation or want of performance has not been waived by the insurer, the insured cannot recover, much less the herein petitioner. 69. ORIENTAL ASSURANCE V . COURT OF APPEALS, 200 SCRA 459 (1991)

ORIENTAL ASSURANCE v. CA (PANAMA SAW MILL) 200 SCRA 459 MELENCIO-HERRERA; August 9, 1991 NATURE Petition for review on certiorari FACTS - Sometime in January 1986, private respondent Panama Sawmill Co., Inc. (Panama) bought, in Palawan, 1,208 pieces of apitong logs, with a total volume of 2,000 cubic meters. It hired Transpacific Towage, Inc., to transport the logs by sea to Manila and insured it against loss for P1-M with petitioner Oriental Assurance Corporation (Oriental Assurance). - While the logs were being transported, rough seas and strong winds caused damage to one of the two barges resulting in the loss of 497 pieces of logs out of the 598 pieces loaded thereon. - Panama demanded payment for the loss but Oriental Assurance refuse on the ground that its contracted liability was for "TOTAL LOSS ONLY." - Unable to convince Oriental Assurance to pay its claim, Panama filed a Complaint for Damages against Oriental Assurance before the Regional Trial Court. Page 3 of 154

INSURANCE G01 CASE DIGESTS - RTC ordered Oriental Assurance to pay Panama with the view that the insurance contract should be liberally construed in order to avoid a denial of substantial justice; and that the logs loaded in the two barges should be treated separately such that the loss sustained by the shipment in one of them may be considered as "constructive total loss" and correspondingly compensable. CA affirmed in toto. ISSUE WON Oriental Assurance can be held liable under its marine insurance policy based on the theory of a divisible contract of insurance and, consequently, a constructive total loss HELD NO - The terms of the contract constitute the measure of the insurer liability and compliance therewith is a condition precedent to the insured's right to recovery from the insurer. Whether a contract is entire or severable is a question of intention to be determined by the language employed by the parties. The policy in question shows that the subject matter insured was the entire shipment of 2,000 cubic meters of apitong logs. The fact that the logs were loaded on two different barges did not make the contract several and divisible as to the items insured. The logs on the two barges were not separately valued or separately insured. Only one premium was paid for the entire shipment, making for only one cause or consideration. The insurance contract must, therefore, be considered indivisible. - More importantly, the insurer's liability was for "total loss only." A total loss may be either actual or constructive (Sec. 129, Insurance Code). An actual total loss is caused by: (a) A total destruction of the thing insured; (b) The irretrievable loss of the thing by sinking, or by being broken up; (c) Any damage to the thing which renders it valueless to the owner for the purpose for which he held it; or (d) Any other event which effectively deprives the owner of the possession, at the port of destination, of the thing insured. (Section 130, Insurance Code). - A constructive total loss is one which gives to a person insured a right to abandon, under Section 139 of the Insurance Code. This provision reads: SECTION 139. A person insured by a contract of marine insurance may abandon the thing insured, or any particular portion thereof separately valued by the policy, or otherwise separately insured, and recover for a total loss thereof, when the cause of the loss is a peril injured against, (a) If more than three-fourths thereof in value is actually lost, or would have to be expended to recover it from the peril; (b) If it is injured to such an extent as to reduce its value more than three-fourths; xxx xxx xxx Page 4 of 154

INSURANCE G01 CASE DIGESTS - The requirements for the application of Section 139 of the Insurance Code, quoted above, have not been met. The logs involved, although placed in two barges, were not separately valued by the policy, nor separately insured. Resultantly, the logs lost in the damaged barge in relation to the total number of logs loaded on the same barge cannot be made the basis for determining constructive total loss. The logs having been insured as one inseparable unit, the correct basis for determining the existence of constructive total loss is the totality of the shipment of logs. Of the entirety of 1,208, pieces of logs, only 497 pieces thereof were lost or 41.45% of the entire shipment. Since the cost of those 497 pieces does not exceed 75% of the value of all 1,208 pieces of logs, the shipment cannot be said to have sustained a constructive total loss under Section 139(a) of the Insurance Code. Disposition judgment under review is SET ASIDE 70. FORTUNE ASSURANCE V . COURT OF APPEALS, 244 SCRA 308 (1995)

G.R. No. 115278 May 23, 1995 Petitioner: FORTUNE INSURANCE AND SURETY CO., INC. (Fortune) Respondent: PRODUCERS BANK OF THE PHILIPPINES (PBP) FACTS: > PBP filed against Fortune a complaint for recovery of the sum of P725,000.00 under the policy issued by Fortune. The money was allegedly lost during a robbery of Producer's armored vehicle while it was in transit to transfer the money from its Pasay City Branch to its head office in Makati along Taft Avenue. >The armored car was driven by Benjamin Magalong escorted by Security Guard Saturnino Atig. >Driver Magalong was assigned by PRC Management Systems with the PBP by virtue of an Agreement and Atiga was assigned by Unicorn Security Services, Inc. by virtue of a contract of Security Service. >After an investigation conducted by the Pasay police authorities, the driver Magalong and guard Atiga were charged, together with Edelmer Bantigue, Reynaldo Aquino and John Doe, with violation of P.D. 532 (Anti-Highway Robbery Law) before the Fiscal of Pasay City. >Demands were made by PBP but Fortune refused to pay as the loss is excluded from the coverage of the insurance policy which is stipulated under "General Exceptions" Section (b) which reads as follows: Page 5 of 154

INSURANCE G01 CASE DIGESTS GENERAL EXCEPTIONS The company shall not be liable under this policy in report of xxx xxx xxx (b) any loss caused by any dishonest, fraudulent or criminal act of the insured or any officer, employee, partner, director, trustee or authorized representative of the Insured whether acting alone or in conjunction with others. . . . 8. The plaintiff opposes the contention of the defendant and contends that Atiga and Magalong are not its "officer, employee, . . . trustee or authorized representative . . . at the time of the robbery. >RTC & CA: held that there should be recovery. The trial court ruled that Magalong and Atiga were not employees or representatives of Producers. The wages and salaries of both Magalong and Atiga are presumably paid by their respective firms, which alone wields the power to dismiss them. Neither is the Court prepared to accept the proposition that driver Magalong and guard Atiga were the "authorized representatives" of plaintiff. ISSUE: W/N the recovery in the policy is precluded under the general exceptions clause? HELD: YES. Page 6 of 154 Fortune is exempt from liability under the general exceptions clause of the insurance policy. >It should be noted that the insurance policy entered into by the parties is a theft or robbery insurance policy which is a form of casualty insurance (Section 174 of the Insurance Code). Other than what is mentioned in the provision, the rights and obligations of the parties must be determined by the terms of their contract, taking into consideration its purpose and always in accordance with the general principles of insurance law. >The purpose of the exception is to guard against liability should the theft be committed by one having unrestricted access to the property. In such cases, the terms specifying the excluded classes are to be given their meaning as understood in common speech. The terms "service" and "employment" are generally associated with the idea of selection, control, and compensation. >A contract of insurance is a contract of adhesion, thus any ambiguity therein should be resolved against the insurer, or it should be construed liberally in favor of the insured and strictly against the insurer. Limitations of liability should be regarded with extreme jealousy and must be construed in such a way, as to preclude the insurer from non-compliance with its obligation. >If the terms of the contract are clear and unambiguous, there is no room for construction and such terms cannot be enlarged or diminished by judicial construction. >An insurance contract is a contract of indemnity. It is settled that the terms of the policy constitute the measure of the insurer's liability. In the absence of statutory prohibition to the contrary, insurance companies have the same rights as individuals to limit

INSURANCE G01 CASE DIGESTS their liability and to impose whatever conditions they deem best upon their obligations not inconsistent with public policy. >It was clear that Fortunes intention is to exclude and exempt from protection and coverage losses arising from dishonest, fraudulent, or criminal acts of persons granted or having unrestricted access to Producers' money or payroll. When it used then the term "employee," it must have had in mind any person who qualifies as such as generally and universally understood, or jurisprudentially established in the light of the four standards in the determination of the employer-employee relationship, or as statutorily declared even in a limited sense as in the case of Article 106 of the Labor Code which considers the employees under a "labor-only" contract as employees of the party employing them and not of the party who supplied them to the employer. >Fortune claims that Producers' contracts with PRC Management Systems and Unicorn Security Services are "labor-only" contracts. But even granting for the sake of argument that these contracts were not "labor-only" contracts, and PRC Management Systems and Unicorn Security Services were truly independent contractors, we are satisfied that Magalong and Atiga were, in respect of the transfer of Producer's money from its Pasay City branch to its head office in Makati, its "authorized representatives" who served as such with its teller Maribeth Alampay. >Producers entrusted the three with the specific duty to safely transfer the money to its head office, with Alampay to be responsible for its custody in transit; Magalong to drive the armored vehicle which would carry the money; and Atiga to provide the needed security for the money, the vehicle, and his two other companions. In short, for these particular tasks, the three acted as agents of Producers. A "representative" is defined as one who represents or stands in the place of another; one who represents others or another in a special capacity, as an agent, and is interchangeable with "agent."

71.

GREAT PACIFIC LIFE V . COURT OF APPEALS, 89 SCRA 543 (1979)

GREAT PACIFIC LIFE v. CA (NGO HING) 89 SCRA 543 DE CASTRO, J; April 30, 1979 ! NATURE Petition for certiorari ! FACTS - On March 14, 1957, private respondent Ngo Hing filed an application with the Great Pacific Life Assurance Co. (Pacific Life) for a 20 year endowment policy of P50k on the life of his 1 year old daughter, Helen. Ngo Hing supplied the essetntial data which petitioner Mondragon, branch manager of the Pacific Life in Cebu, wrote on the corresponding form in his own handwriting, later typing the data on an application form signed by Ngo Hing. The latter paid the P1077.75 annual premium but retained P1,317 as commission as he was also a duly authorized agent of Pacific Life. The binding deposit receipt was then issued to Ngo Hing; Mondragon handwrote his strong recommendation for the approval of the application on the back of the form. - On April 30, Mondragon received a letter from Pacific Life which stated that the 20 year

Page 7 of 154

INSURANCE G01 CASE DIGESTS endowment plan was not available for minors below 7, but that Pacific Life could consider the same under the Juvenile Triple Action Plan, advising that if the offer was acceptable, the Juvenile Non-Medical Declaration be sent to the company. -Mondragon allegedly failed to inform Ngo Hing of the nonacceptance of the insurance plan, instead writing Pacific Life again, recommending the approval of the endowment plan to children since customers had been asking for such coverage since 1954. -On May 28, 1957, Helen died of influenza. Ngo Hing sought the payment of the proceeds of the insurance, but having failed to do so, filed an action for recovery with the CFI of Cebu. The Court ordered Pacific Life to pay P50k with 6% interest, hence this petition. ! ISSUE WON the binding deposit receipt constituted a temporary contract of the life insurance in question ! HELD NO - The binding deposit receipt is merely a provisional contract and only upon compliance with the ff conditions: (1) that the company be satisfied that the applicant was insurable on standard rates (2) that if the company does not accept the application and offers a different policy, the insurance contract shall not be binding until the applicant accepts the new policy (3) that if the applicant is not found to be insurable on standard rates and the application is disapproved, the insurance shall not be in force at any time and the premium be returned to the applicant. -This implies the receipt is merely an acknowledgement, on behalf of the company, that the Cebu branch of Pacific Life had received the premium and had accepted the application subject to processing by the insurance company, which will approve or reject it depending on whether the applicant is insurable on standard rates. As such, the receipt was never in force-it does not insure outright. No liability attaches until the principal approves the risk and a receipt is given by the agent; because private respondent failed to accept Pacific Life's offer for the Juvenile Triple Action plan, there was no meeting of the minds and thus no contract. Also, being an authorized agent of Pacific Life, Ngo Hing must have known the company did not offer the insurance applied for and merely took a chance on Mondragon's recommendation. Disposition the decision appealed from is set aside, absolving Pacific Life from their civil liabilities

Page 8 of 154

INSURANCE G01 CASE DIGESTS 72. FACTS: Enrique Mora, owner of Oldsmobile sedan model 1956 mortgaged the same to the H.S. Reyes, Inc., with the condition that the former would insure the automobile with the latter as beneficiary. The automobile was thereafter insured with the State Bonding & Insurance Co., Inc., and motor car insurance policy was issued to Enrique Mora. During the effectivity of the insurance contract, the car met with an accident. Enrique Mora, without the knowledge and consent of the H.S. Reyes, Inc., authorized the Bonifacio Bros. Inc. to furnish the labor and materials, some of which were supplied by the Ayala Auto Parts Co. For the cost of labor and materials, Enrique Mora was billed at P2,102.73 through the H.H. Bayne Adjustment Co. The insurance company after claiming a franchise in the amount of P100, drew a check in the amount of P2,002.73, as proceeds of the insurance policy, payable to the order of Enrique Mora or H.S. Reyes,. Inc., and entrusted the check to the H.H. Bayne Adjustment Co. for disposition and delivery to the proper party. In the meantime, the car was delivered to Enrique Mora without the consent of the H.S. Reyes, Inc., and without payment to the Bonifacio Bros. Inc. and the Ayala Auto Parts Co. of the cost of repairs and materials. Upon the theory that the insurance proceeds should be paid directly to them, the Bonifacio Bros. Inc. and the Ayala Auto Parts Co. filed a complaint with the MTC of Manila against Enrique Mora and the State Bonding & Insurance Co., Inc. for the collection of the sum of P2,002.73. BONIFACIO BROTHERS V . MORA, 20 SCRA 261 (1967) The insurance company filed its answer with a counterclaim for interpleader, requiring the Bonifacio Bros. Inc. and the H.S. Reyes, Inc. to interplead in order to determine who has better right to the insurance proceeds in question. Municipal Court rendered a decision declaring the H.S. Reyes, Inc. as having a better right to the disputed amount and ordering State Bonding & Insurance Co. Inc. to pay to the H. S. Reyes, Inc. the said sum of P2,002.73. From this decision, the appellants elevated the case to the CFI of Manila which rendered a decision, affirming the decision of the Municipal Court. The Bonifacio Bros. Inc. and the Ayala Auto Parts Co. moved for reconsideration of the decision, but the trial court denied the motion. Hence, this appeal.

ISSUE: WON there is privity of contract between the Bonifacio Bros. Inc. and the Ayala Auto Parts Co. on the one hand and the insurance company on the other. HELD: NONE Appellant = Bonifacio Bro., Inc. From the undisputed facts and from the pleadings it will be seen that the appellants' alleged cause of action rests exclusively upon the terms of the insurance contract. The appellants seek to recover the insurance proceeds, and for this purpose, they rely upon paragraph 4 of the insurance contract document executed by and between the State Bonding & Insurance Company, Inc. and Enrique Mora. The appellants are not mentioned in the contract as parties thereto nor is there any clause or provision thereof from which we can infer that there is an obligation on the part of the insurance company to pay the cost of repairs directly to them. Page 9 of 154

INSURANCE G01 CASE DIGESTS It is fundamental that contracts take effect only between the parties thereto, except in some specific instances provided by law where the contract contains some stipulation in favor of a third person.1Such stipulation is known as stipulation pour autrui or a provision in favor of a third person not a pay to the contract. Under this doctrine, a third person is allowed to avail himself of a benefit granted to him by the terms of the contract, provided that the contracting parties have clearly and deliberately conferred a favor upon such person. In this connection, this Court has laid down the rule that the fairest test to determine whether the interest of a third person in a contract is a stipulation pour autrui or merely an incidental interest, is to rely upon the intention of the parties as disclosed by their contract.4 In the instant case the insurance contract does not contain any words or clauses to disclose an intent to give any benefit to any repairmen or materialmen in case of repair of the car in question. The parties to the insurance contract omitted such stipulation, which is a circumstance that supports the said conclusion. On the other hand, the "loss payable" clause of the insurance policy stipulates that "Loss, if any, is payable to H.S. Reyes, Inc." indicating that it was only the H.S. Reyes, Inc. which they intended to benefit. If it were the intention of the insurance company to make itself liable to the repair shop or materialmen, it could have easily inserted in the contract a stipulation to that effect. Another cogent reason for not recognizing a right of action by the appellants against the insurance company is that "a policy of insurance is a distinct and independent contract between the insured and insurer, and third persons have no right either in a court of equity, or in a court of law, to the proceeds of it, unless there be some contract of trust, expressed or implied between the insured and third person."5 In this case, no contract of trust, Page 10 of 154 expressed or implied exists. We, therefore, agree with the trial court that no cause of action exists in favor of the appellants in so far as the proceeds of insurance are concerned. The appellants' claim, if at all, is merely equitable in nature and must be made effective through Enrique Mora who entered into a contract with the Bonifacio Bros. Inc. This conclusion is deducible not only from the principle governing the operation and effect of insurance contracts in general, but is clearly covered by the express provisions of section 50 of the Insurance Act which read: The insurance shall be applied exclusively to the proper interests of the person in whose name it is made unless otherwise specified in the policy. The policy in question has been so framed that "Loss, if any, is payable to H.S. Reyes, Inc.," which unmistakably shows the intention of the parties. The final contention of the appellants is that the right of the H.S. Reyes, Inc. to the insurance proceeds arises only if there was loss and not where there is mere damage as in the instant case. Suffice it to say that any attempt to draw a distinction between "loss" and "damage" is uncalled for, because the word "loss" in insurance law embraces injury or damage. Indeed, according to sec. 120 of the Insurance Act, a loss may be either total or partial.

INSURANCE G01 CASE DIGESTS 73. HEIRS OF L.G. MARAMAG V . MARAMAG, 588 SCRA 774 (2009) THE COMPLAINT OR PETITION FAILED TO STATE A CAUSE OF ACTION AS TO DECLARE AS VOID THE DESIGNATION OF EVA AS BENEFICIARY for Loreto revoked her designation and already disqualified her. 6. GREPALIFR CONTENTION: Eva was not designated as an insurance policy beneficiary, that r claims of the illegitimate children were denied because Loreto was ineligible for the insurance due to the misrepresentation in his application form that he was not more than 65 years old. 7.Both Insular and Grepalife countered that the insurance proceeds belong exclusively to the designated beneficiaries in the policies, not to the estate or to the heirs of the insured. Grepalife also reiterated that it had disqualified Eva as a beneficiary when it ascertained that Loreto was legally married to Vicenta Pangilinan Maramag. 8.RTC: In favor of the respondents. Neither could the plaintiffs invoked (sic) the law on donations or the rules on testamentary succession in order to defeat the right of herein defendants to collect the insurance indemnity. The beneficiary in a contract of insuranceis not the donee spoken in the law of donation. The rules on testamentary succession cannot apply here, for the insurance indemnity does not partake of a donation. THE PROCEEDS BELONG EXCLUSIVELY THE BENEFICIARY AND NOT TO THE ESTATE OF THE PERSON. NO SUFFICIENT CAUSE OF ACTION AGAINST THE ILLEGITIMATE FLR THE REDUCTION AND/OR DECLARATION OF INOFFICIOUS OF DONATION AS PRIMARY BENEFICIARY. EVA AS THE CONCUBINE CANNOT BE A BENEFICIARY Page 11 of 154

1. PETITIONERS were legitimate wife and children of Loreto Maramag. 2. PETITIONERS' CONTENTION: a. RESPONDENTS were Loreto''s illegitimate family. b.Eva Maramag was concubine of Loreto and a suspect in the killing of Loreto. Thus, she is disqualified to receive any proceeds from his insurance policies from Insular Life Assurance (INSULAR) and Great Pacific Life Assurance (GREPALIFE) c. The illegitimate children were entitled only to 1/2 of the legitime of the legitimate children, thus the proceeds released to the illegitimate children were inofficious and should be reduced. d. x x x 3. TRO and writ of preliminary injunction were filed by the petitioner because, some part of the insurance proceeds had already been released in favor of one of the illegitimate and the rest are to be releases in favor of the other illegitimate. 4.Insular admitted that Loreto misrepresented Eva as his legitimate wife and Odessa, Karl Brian, and Trisha Angelie as his legitimate children, and that they filed their claims for the insurance proceeds of the insurance policies. 5. INSULAR ALLEGATION:

INSURANCE G01 CASE DIGESTS stipulations or indemnity. In such a case, third parties may directly sue and claim from the insurer. Petitioners are third parties to the insurance contracts with Insular and Grepalife and, thus, are not entitled to the proceeds thereof. Accordingly, respondents Insular and Grepalife have no legal obligation to turn over the insurance proceeds to petitioners. The revocation of Eva as a beneficiary in one policy and her disqualification as such in another are of no moment considering that the designation of the illegitimate children as beneficiaries in Loreto's insurance policies remains valid. Because no legal proscription exists in naming as beneficiaries the children of illicit relationships by the insured,22 the shares of Eva in the insurance proceeds, whether forfeited by the court in view of the prohibition on donations under Article 739 of the Civil Code or by the insurers themselves for reasons based on the insurance contracts, must be awarded to the said illegitimate children, the designated beneficiaries.

9. CA: Affirmed the decision of the RTC. The distribution of the insurance proceeds is governed primarily by the Insurance Code, and the provisions of the Civil Code are irrelevant and inapplicable. ISSUE: Whether or not the legitimate children of the insured decease shall receive the proceeds of the insurance that originally designated to Eva? HELD: No. Even assuming Insular disqualified Eva as a beneficiary, her share should not have been distributed to the legitimate heirs of the insured deceased. RATIONALE: It is evident from the face of the complaint that petitioners are not entitled to a favorable judgment in light of Article 2011 of the Civil Code which expressly provides that insurance contracts shall be governed by special laws, i.e., the Insurance Code. Section 53 of the Insurance Code states SECTION 53. The insurance proceeds shall be applied exclusively to the proper interest of the person in whose name or for whose benefit it is made unless otherwise specified in the policy. Pursuant thereto, it is obvious that the only persons entitled to claim the insurance proceeds are either the insured, if still alive; or the beneficiary, if the insured is already deceased, upon the maturation of the policy. The exception to this rule is a situation where the insurance contract was intended to benefit third persons who are not parties to the same in the form of favorable

Page 12 of 154

INSURANCE G01 CASE DIGESTS 74. COQUIA V. FIELDMEN S INSURANCE , 26 SCRA 178 (1968) Held: 1. YES 2. While the general rule is that only parties to a contract may bring an action based thereon, one exception is found under Article 1311 of the Civil Code. 3. It provides that If a contract should contain some stipulation in favor of a third person, he may demand its fulfillment provided he communicated his acceptance to the obligor before its revocation. 4. These are contracts pour autrui wherein enforcement of a contract may be demanded by a third party for whose benefit it was made, although not a party to the contract. 5. In this case, the policy contained a stipulation which states the following: Section I Liability to Passengers. 1. The Company will, subject to the Limits of Liability and under the Terms of this Policy, indemnify the Insured in the event of accident caused by or arising out of the use of Motor Vehicle against all sums which the Insured will become legally liable to pay in respect of: Death or bodily injury to any fare-paying passenger including the Driver ... who is riding in the Motor Vehicle insured at the time of accident or injury. 6. Another stipulation provides that In the event of death of any person entitled to indemnity under this Policy, the Company will, in respect of the liability incurred by such person, indemnify his personal representatives in terms of and subject to the limitations of this Policy, provided, that such representatives shall, as though they were the Insured, observe, fulfill and be subject to the Terms of this Policy insofar as they can apply. 7. Pursuant to these stipulations, the Company "will indemnify any authorized Driver who is driving the Motor Vehicle" of the Insured and, in the event of death of said Page 13 of 154

Coquia vs Fieldmens Insurance Facts: 1. Fieldmen's Insurance Company, Inc. issued, in favor of the Manila Yellow Taxicab Co., Inc. a common carrier accident insurance policy. 2. It was stipulated in said policy that the company will indemnify the Insured in the event of accident caused by or arising out of the use of Motor Vehicle against all sums which the Insured will become legally liable to pay. 3. While the policy was in force, or on February 10, 1962, a taxicab of the Insured, driven by Carlito Coquia, met a vehicular accident at Mangaldan, Pangasinan, in consequence of which Carlito died. 4. The Insured filed therefor a claim for P5,000.00 to which the Company replied with an offer to pay P2,000.00, by way of compromise. 5. The Insured rejected the same and made a counter-offer for P4,000.00, but the Company did not accept it. 6. The insured and Carlitos parents or the Coquias filed a complaint against the company to collect the proceeds of the insurance policy. 7. As a defence, the company argued lack of cause of action on the part of the Coquias. 8. RTC ruled for the plaintiffs sentencing the company to pay 4k. Issue: 1. Whether or not the Coquias can claim under the policy even if they are alleged to not have a cause of action against the company as they are not parties to the insurance policy?

INSURANCE G01 CASE DIGESTS driver, the Company shall, likewise, "indemnify his personal representatives." 8. Thus, the policy under consideration is typical of contracts pour autrui, this character being made more manifest by the fact that the deceased driver paid fifty percent (50%) of the corresponding premiums, which were deducted from his weekly commissions. 9. Under these conditions, it is clear that the Coquias who, admittedly, are the sole heirs of the deceased have a direct cause of action against the Company,3 and, since they could have maintained this action by themselves, without the assistance of the Insured, it goes without saying that they could and did properly join the latter in filing the complaint herein. 75. LOPEZ V . DEL ROSARIO , 44 PHIL 98 (1922)

Lopez vs. Del Rosario (1922) Facts: 1. Mrs. del Rosario owned a bonded warehouse in Manila. Engaged in the business of a warehouse keeper, she stored copra and other merchandise in building. 2. Among the persons who had copra deposited in the Del Rosario warehouse was Froilan Lopez, the holder of fourteen warehouse receipts in his own name, and the name of Elias T. Zamora. 3. Lopez named a declared value of P107,990.40. The warehouse receipts provided: (1) For insurance at the rate of 1 per cent per month on the declared value; (2) the company reserves to itself the right to raise and/or lower the rates of storage and/or of insurance on giving one calendar month's notice in writing; (3) this warrant carries no insurance unless so noted on the face hereof, cost of which is in addition to storage; (4) the time for which storage and/or insurance is charged is thirty (30) days; (5) payment for storage and/or insurance, etc., shall be made in advance, and/or within five (5) days after presentation of bill. 4. It is admitted that insurance was paid by Lopez to May 18, 1920, but not thereafter. 5. Mrs. Del Rosario secured insurance on the warehouse and its contents with the National Insurance Co., Inc., the Commercial Union Insurance Company, the Alliance Insurance Company, the South British Insurance Co., Ltd., and the British Traders Insurance Co., Ltd. 6. The warehouse caught fire. Everything was destroyed. Only copra worth P49,985 was salvaged. 7. Mrs. del Rosario was able to settle everything except the account of Lopez. Page 14 of 154

INSURANCE G01 CASE DIGESTS the Del Rosario the sum of P81,093.65, with interest at 6 per cent per annum from May 13, 1921, until paid.

Issue: Whether or not Mrs. Del Rosario acted as an agent of Lopez in taking out an insurance on the contents of the warehouse Held: YES. The agency can be deduced from the warehouse receipts, the insurance policies, and the circumstances surrounding the transaction. The law is that a policy effected by bailee and covering by its terms his own property and property held in trust; inures, in the event of a loss, equally and proportionately to the benefit of all the owners of the property insured. Even if one secured insurance covering his own goods and goods stored with him, and even if the owner of the stored goods did not request or know of the insurance, and did not ratify it before the payment of the loss, yet it has been held by a reputable court that the warehouseman is liable to the owner of such stored goods for his share.

76.

DEVELOPMENT BANK V, INTERMEDIATE APPELLATE COURT, 143 SCRA 62 (1986)

Development Insurance vs IAC GR No. 71360 July 16, 1986 Facts: A fire occurred in the building of Philippine Union Realty Development Corporation (PURDC) and it sued for damages from Devt Insurance based on an insurance contract. Devt Insurance failed to answer on time and was declared in default by the trial court. A judgment of default was subsequently rendered on the strength of the evidence submitted ex parte by PURDC, which was allowed full recovery of its claim. Devt Insuracne moved to lift the order of default, invoking excusable neglect, and to vacate the judgment by default; which was denied by the court. The IAC affirmed the decision of the trial court. The face value of the policy is P2,500,000. Devt Insurance is claiming that since at the time of the fire, the building insured was worth P5.8M, they can only be liable to the extent of the proportion between the difference between that amount and the face value, as against the total loss sustained, which is P508,867; making them only liable for only P67,629.31.

Moreover, the Court found in two documents of Mrs. Del Rosario against the insurance companies (agreement for arbitration and the statement of claim) she acknowledged her responsibility to the owners of the stored merchandise, against risk of loss by fire. The award of the arbitrators covered not alone Mrs. Del Rosario's warehouse but the products stored in the warehouse by Lopez and others.

Lopez' rights to the insurance money have not been forfeited by failure to pay the insurance provided for in the warehouse receipts. A preponderance of the proof does not demonstrate that he ever ordered the cancellation of his insurance with the Del Rosario. Nor is it shown that the Lopez ever refused to pay the insurance when the bills were presented to him, and that notice of an intention to cancel the insurance was ever given to him. Lopez can recover from

Issue: Page 15 of 154

INSURANCE G01 CASE DIGESTS Whether the insurer should be liable for the whole amount of the loss Ruling: The insurer is liable for the whole amount. The policy issued to PURDC is an open policy and is subject to the express condition that in the event of loss, whether total or partial, it is understood that the amount of the loss shall be subject to appraisal and the liability of the company, if established, shall be limited to the actual loss, subject to the applicable terms, conditions, warranties and clauses of the policy, and in no case shall exceed the amount of the policy. o An open policy is one in which the value of the thing insured is not agreed upon but is left to be ascertained in case of loss There is no evidence on record that the building was worth P5.8M at the time of the loss; only Devt Insurance says so, and it does not back up its self-serving estimate with any independent corroboration. Since the building was insured at P2.5M, this must be considered the value of the building on the day the fire occurred. The actual loss has been ascertained in this case and the Court will respect such factual determination in the absence of proof that it was arrived at arbitrarily. There is no such showing. Applying the open policy clause as expressly agreed upon by the parties in their contract, the Court holds that PURDC is entitled to the payment of indemnity in the total amount of P508,867. 77. TEAL MOTOR V . ORIENT INSURANCE , 59 PHIL. 809 Facts: -These seven cases related to insurance policies covering the goods, wares, and merchandise contained in the building in the Port Area in the City of Manila which was damaged by a fire of unknown origin the afternoon of Sunday, January 6, 1929. -At the request of the insured, the companies gave additional time for the filing of the claims of loss. These claims were definitely rejected in writing by the insurance companies through their agents on April 15, 1929. - Among the special defenses of the insurance companies is one based upon a clause in the policies which, with the exception of those of the Atlas Assurance Company, Ltd., among other things provides: if the claim be made and rejected, and action or suit be not commenced within three months after such rejection, ... under this Policy shall be forfeited.

all

- While those cases were under advisement here, the Supreme Court noticed that the provision relating to the Atlas policy reads: if the claim be made and rejected and arbitration proceedings be not commenced in pursuance of the 18th Condition of this Policy within three months after such rejection; all benefit under this Policy shall be forfeited. - No such arbitration proceedings were instituted within the three months' period. Page 16 of 154

INSURANCE G01 CASE DIGESTS - The seven suits were filed between the 3rd and the 15th day of August, 1929, or more than three months after the rejection by the defendant companies of plaintiff's claim. Issue: Whether or NOT the claims were filed on time? Ruling: The Supreme Court held that the case was filed out of time. Plaintiff was given such time as it deemed necessary to formulate and present its claim of loss. That claim was investigated by the adjusters for several months, and under the contract of insurance, the insured had three months after rejection in which to bring suit. The issues were virtually joined on the presentation of the claims and their rejection by the companies in writing, and three months thereafter is not an unreasonably short time to draft and file in court an appropriate complaint on a contract of fire insurance. Ratio: A provision requiring presentation of claim within three months after the fire, and the bringing of action within three months after refusal of claim is valid. PAULO ANG and SALLY C. ANG, plaintiffs-appellees, vs. FULTON FIRE INSURANCE CO., ET AL., defendants. FULTON FIRE INSURANCE CO., defendant-appellant. LABRADOR, J.: FACTS September 9, 1953 - Fulton Fire Insurance Company issued a fire insurance policy in favor of P. & S Department Store (owned by the Spouses Paulo Ang and Sally C. Ang) over stocks of general merchandise, consisting principally of dry goods, contained in a building occupied by the Angs at Laoag, Ilocos Norte. The premium is P500.00 annually. September 31, 1954 policy was renewed for another year. December 17, 1954 - the store was destroyed by fire. December 30, 1954 - the Angs executed the first claim form together with all the necessary papers (books of accounts of the insured for the year 1953-1954 and a clearance from the Philippine Constabulary and the police), and they were all forwarded to the Manila Adjustment Company, Fulton's adjusters. January 13, 1955 - Paulo Ang and 10 others were charged for arson in a Criminal Case but Paulo Ang was eventually acquitted. April 6, 1956 - Fulton denied the claim. Page 17 of 154 78. ANG V. FULTON FIRE INSURANCE , 2 SCRA 945 (1961)

G.R. No. L-15862

July 31, 1961

INSURANCE G01 CASE DIGESTS April 19, 1956 - denial received by Angs. May 11, 1956 Angs filed 1 case to assert the claim but against Paramount Surety and Insurance Company (Fultons agent). September 3, 1957 - 1st case was dismissed without prejudice. May 5, 1958 Angs filed 2nd case against Fulton and Paramount Surety to recover from them the face value of the policy, but Paramount was eventually dropped from the complaint. May 26, 1958 ANSWER OF FULTON: (1) denied that the loss by the fire was accidental, alleging that it was occasioned by the willful act of Paulo Ang himself. (2) claimed that under paragraph 13 of the policy, if the loss or damage is occasioned by the willful act of the insured, or if the claim is made and rejected but no action is commenced within 12 months after such rejection, all benefits under the policy would be forfeited. Since Angs received notice of denial on April 18, 1956, and they filed action only on May 5, 1958, all the benefits under the policy have been forfeited. February 12, 1959 REPLY OF ANGS: The 1st case was filed May 11, 1956 but was dismissed without prejudice on September 3, 1957. That period between May 11, 1956 to September 3, 1957 must be deducted from the prescriptive period of 12 months. CFI 2nd case in favor of Angs (ordering Fulton to pay the Angs the sum of P10,000.00, with interest, and an
st

additional sum of P2,000.00 as attorney's fees, and costs). The CFI held that the bringing of the action May 11, 1956, tolled the running of the 12 month period.Fulton appealed directly to the Supreme Court. ISSUE Whether the Angs may validly claim on the policy even with the prohibition on Paragraph 13 of the policy. SC HELD NO. The basic error committed by the trial court is its view that the filing of the action against the agent of the defendant company was "merely a procedural mistake of no significance or consequence, which may be overlooked." The condition contained in the insurance policy that claims must be presented within one year after rejection is not merely a procedural requirement. The condition is an important matter, essential to a prompt settlement of claims against insurance companies, as it demands that insurance suits be brought by the insured while the evidence as to the origin and cause of destruction have not yet disappeared. It is in the nature of a condition precedent to the liability of the insurer, or in other terms, a resolutory cause, the purpose of which is to terminate all liabilities in case the action is not filed by the insured within the period stipulated. The bringing of the action against the Paramount Surety & Insurance Company, the agent of the defendant Company cannot have any legal effect except that of notifying the agent of the claim. Beyond such notification, the filing of the action can serve no other purpose. There is no law giving any effect to such action upon the principal. Besides, there is no condition in the policy that the action Page 18 of 154

INSURANCE G01 CASE DIGESTS must be filed against the agent, and this Court can not by interpretation, extend the clear scope of the agreement beyond what is agreed upon by the parties. The case of E. Macias & Co. vs. China Fire Insurance Co. has settled the issue presented by the appellees in the case at bar definitely against their claim. In that case, We declared that the contractual station in an insurance policy prevails over the statutory limitation, as well as over the exceptions to the statutory limitations that the contract necessarily supersedes the statute (of limitations) and the limitation is in all phases governed by the former. (E. Macias & Co. vs. China Fire Insurance & Co., 46 Phil. pp. 345-353). As stated in said case and in accordance with the decision of the Supreme Court of the United States in Riddlesbarger vs. Hartford Fire Insurance Co. (7 Wall., 386), the rights of the parties flow from the contract of insurance, hence they are not bound by the statute of limitations nor by exemptions thereto. In the words of our own law, their contract is the law between the parties, and their agreement that an action on a claim denied by the insurer must be brought within one year from the denial, governs, not the rules on the prescription of actions. The judgment appealed from is hereby set aside and the case dismissed, with costs against the plaintiffs-appellees. APPEAL GRANTED. 79. FACTS: Emilio Tan took from Sun Insurance Office a P300,000.00 property insurance policy to cover his interest in the electrical supply store of his brother. Four days after the issuance of the policy, the building was burned including the insured store. On August 20, 1983, Tan filed his claim for fire loss with Sun Insurance Office, but on February 29, 1984, Sun Insurance Office wrote Tan denying the latters claim. On April 3, 1984, Tan wrote Sun Insurance Office, seeking reconsideration of the denial of his claim. Sun Insurance Office answered the letter, advising Tans counsel that the Insurers denial of Tans claim remained unchanged. ISSUES: (1)WON the filing of a motion for reconsideration interrupts the 12 months prescriptive period to contest the denial of the insurance claim; and (2)WON the rejection of the claim shall be deemed final only of it contains words to the effect that the denial is final; HELD: (1) No. In this case, Condition 27 of the Insurance Policy of the parties reads: 27. Action or suit clause - If a claim be made and rejected and an action or suit be not commenced either in the Insurance Commission or in any court of competent jurisdiction within twelve (12) months from receipt of notice of such rejection, or in case of arbitration taking place as provided herein, within twelve (12) Page 19 of 154 SUN I NSURANCE OFFICE V . CA, 195 SCRA 193 (1991)

INSURANCE G01 CASE DIGESTS months after due notice of the award made by the arbitrator or arbitrators or umpire, then the claim shall for all purposes be deemed to have been abandoned and shall not thereafter be recoverable hereunder. As the terms are very clear and free from any doubt or ambiguity whatsoever, it must be taken and understood in its plain, ordinary and popular sense. Tan, in his letter addressed to Sun Insurance Office dated April 3, 1984, admitted that he received a copy of the letter of rejection on April 2, 1984. Thus, the 12-month prescriptive period started to run from the said date of April 2, 1984, for such is the plain meaning and intention of Section 27 of the insurance policy. The condition contained in an insurance policy that claims must be presented within one year after rejection is not merely a procedural requirement but an important matter essential to a prompt settlement of claims against insurance companies as it demands that insurance suits be brought by the insured while the evidence as to the origin and cause of destruction have not yet disappeared. It is apparent that Section 27 of the insurance policy was stipulated pursuant to Section 63 of the Insurance Code, which states that: Sec. 63. A condition, stipulation or agreement in any policy of insurance, limiting the time for commencing an action thereunder to a period of less than one year from the time when the cause of action accrues, is void. It also begs to ask, when does the cause of action accrue? The insureds cause of action or his right to file a claim either in the Insurance Commission or in a court of competent jurisdiction commences from the time of the denial of his claim by the Insurer, either expressly or impliedly. But the rejection referred to should be construed as the rejection in the first instance (i.e. at the first occasion or for the first time), not rejection conveyed in a resolution of a petition for reconsideration. Thus, to allow the filing of a motion for reconsideration to suspend the running of the prescriptive period of twelve months, a whole new body of rules on the matter should be promulgated so as to avoid any conflict that may be brought by it, such as: a.whether the mere filing of a plea for reconsideration of a denial is sufficient or must it be supported by arguments/affidavits/material evidence; b.how many petitions for reconsideration should be permitted? (2) No. The Eagle Star case cited by Tan to defend his theory that the rejection of the claim shall be deemed final only of it contains words to the effect that the denial is final is inapplicable in the instant case. Final rejection or denial cannot be taken to mean the rejection of a petition for reconsideration. The Insurance policy in the Eagle Star case provides that the insured should file his claim, first, with the carrier and then with the insurer. The final rejection being referred to in said case is the rejection by the insurance company.

Page 20 of 154

INSURANCE G01 CASE DIGESTS 80. PACIFIC BANKING CORP. V CA, 168 SCRA 1 (1988) then made an excuse that the insured had not filed any claim with it, nor submitted proof of loss which is a clear violation of Policy Condition No.11, as a result, determination of the liability of private respondent could not be made. Pacific Banking filed in the trial court an action for a sum of money for P61,000.00 against Oriental Assurance. At the trial, Pacific Bank presented communications of the insurance adjuster to Asian Surety revealing undeclared co-insurances with the following: P30,000 with Wellington Insurance; P25,000 with Empire Surety and P250,000 with Asian Surety undertaken by insured Paramount on the same property covered by its policy with Oriental whereas the only co-insurances declared in the subject policy are those of P30,000.00 with Malayan P50,000.00 with South Sea and P25.000.00 with Victory. The defense of fraud, in the form of non-declaration of coinsurances which was not pleaded in the answer, was also not pleaded in the Motion to Dismiss. The trial court denied the respondents motion. Oriental filed another motion to include additional evidence of the co-insurance which could amount to fraud. The trial court rendered judgment making Oriental Assurance liable for P61,000.00, but the Court of Appeals reversed the RTC decision.

PACIFIC BANKING CORPORATION vs. CA No. L-41014; November 29, 1988

Facts: On October 21, 1963, an Open Fire Policy was issued to the Paramount Shirt Manufacturing Co. (insured) by which private respondent Oriental Assurance bound itself to indemnify the insured for any los or damage, not exceeding P61,000.00, caused by fire to its property consisting of stocks, materials and supplies usual to a shirt factory while contained in the first to third floors of the building where they are located for a period of one year starting October 21, 1964. At the time the policy was issued, Paramount Shirt was a debtor of Pacific Bank amounting to P800,000.00. Goods in the said policy were held in trust by Paramount for Pacific Bank under trust receipts. Said policy was duly endorsed to the petitioner bank as mortgagee/trustor of the properties insured, with the knowledge and consent of private respondent to the effect that loss if any under this policy is payable to the (Pacific Bank)". While the aforesaid policy was in full fore and effect, a fire broke out on the subject premises destroying the goods contained in its ground and second floors. Pacific Bank sent a letter of demand to Oriental Assurance for indemnity, but the latter wasnt ready to give since it was awaiting the adjusters report. It

Issues: 1. Whether or not the unrevealed co-insurances violated policy conditions no. 3? 2. Whether or not the insured failed to file the required proof of loss prior to court action?

Page 21 of 154

INSURANCE G01 CASE DIGESTS Held and Ratio: 1. YES. Policy Condition 3 provides that the insured must give notice of any insurance already in effect or subsequently be in effect covering same property being insured. Failure to do so, the policy shall be forfeited. Failure to reveal before the loss of the 3 other insurances is a clear misrepresentation or a false declaration. The material fact was asked for but was not revealed. Representations of facts are the foundations of the contract. Pacific itself provided for the evidences in trial court that proved existence of misrepresentation. The evidence adduced shows that twenty-four (24) days after the fire, petitioner merely wrote letters to private respondent to serve as a notice of loss, thereafter, the former did not furnish the latter whatever pertinent documents were necessary to prove and estimate its loss. Instead, petitioner shifted upon private respondent the burden of fishing out the necessary information to ascertain the particular account of the articles destroyed by fire as well as the amount of loss. It is noteworthy that private respondent and its adjuster notified petitioner that insured had not yet filed a written claim nor submitted the supporting documents in compliance with the requirements set forth in the policy. Despite the notice, the latter remained unheedful.

2. YES. Generally, the cause of action on the policy accrues when the loss occurs, But when the policy provides that no action shall be brought unless the claim is first presented extrajudicially in the manner provided in the policy, the cause of action will accrue from the time the insurer finally rejects the claim for payment.

In the case at bar, policy condition No. 11 specifically provides that the insured shall on the happening of any loss or damage give notice to the company and shall within fifteen (15) days after such loss or damage deliver to the private respondent (a) a claim in writing giving particular account as to the articles or goods destroyed and the amount of the loss or damage and (b) particulars of all other insurances, if any. Likewise, insured was required "at his own expense to produce, procure and give to the company all such further particulars, plans, specifications, books, vouchers, invoices, duplicates or copies thereof, documents, proofs and information with respect to the claim".

Since the required claim by insured, together with the preliminary submittal of relevant documents had not been complied with, it follows that private respondent could not be deemed to have finally rejected petitioner's claim and therefore the latter's cause of action had not yet arisen. Compliance with condition No. 11 is a requirement sine qua non to the right to maintain an action as prior thereto no violation of petitioner's right can be attributable to private respondent. This is so, as before such final rejection, there was no real necessity for bringing suit. Petitioner should have endeavored to file the formal claim and procure all the documents, papers, inventory needed by private respondent or its adjuster to ascertain the amount of loss and after compliance await the final rejection of its claim. Indeed, the law does not encourage unnecessary litigation. Page 22 of 154

INSURANCE G01 CASE DIGESTS 81. TRAVELLERS INSURANCE V . CA, 272 SCRA 536 (1997) TRAVELLERS INSURANCE & SURETY CORP. v. CA (MENDOZA) 272 SCRA 536 HERMOSISIMA, JR; May 22, 1997 NATURE The petition herein seeks the review and reversal of the decision of respondent Court of Appeals affirming in toto the judgment of the Regional Trial Court in an action for damages filed by private respondent Vicente Mendoza, Jr. as heir of his mother who was killed in a vehicular accident. FACTS An old lady was hit by a taxicab. The taxicab was later identified and a case was filed against the driver and owner. Later, an amendment was filed to include the insurance company. RTC and CA ordered that the owner, driver as well as the insurance company be held solidarily liable. Even assuming arguendo that there was such a contract, private respondent's cause of action can not prevail because he failed to file the written claim mandated by the Insurance Code (before it was amended-action must be brought within six months from date of the accident (this is whats applicable here) ; after amendment- "action or suit for recovery of damage due to loss or injury must be brought in proper cases, with the Commissioner or the Courts within one year from denial of the claim, otherwise the claimant's right of action shall prescribe" ). He is deemed, under this legal provision, to have waived his rights as against petitionerinsurer. can sue the insurer. Where the contract is for indemnity against actual loss or payment, then third persons cannot proceed against the insurer, the contract being solely to reimburse the insured for liability actually discharged by him thru payment to third persons, said third persons' recourse being thus limited to the insured alone. But in the case at bar, there was no contract shown. What then was the basis of the RTC and the CA to say that the insurance contract was a third-party liability insurance policy? Consequently, the trial court was confused as it did not distinguish between the private respondent's cause of action against the owner and the driver of the Lady Love taxicab and his cause of action against petitioner. The former is based on torts and quasi-delicts while the latter is based on contract.

ISSUE WON RTC and CA erred HELD YES Where the contract provides for indemnity against liability to third persons, then third persons to whom the insured is liable

Page 23 of 154

INSURANCE G01 CASE DIGESTS 82. LOPEZ V . COMPANIA DE SEGUROS, 16 SCRA 855 (1966) Lopez vs. Filipinas Compaia de Seguros G.R. No. L-19613 April 30, 1966 FACTS: Plaintiff applied with the defendant company for the insurance of his properties: Biederman truck tractor and a Winter Weils trailer from loss or damage in the amount of P20,000.00 and P10,000.00, respectively. During the application, the defendant company inquired of the plaintiff the ff: Has any company in respect of the insurance of any car or vehicle (A) declined, cancelled or refused to renew your insurance? (B) increased your premium renewal? Plaintiff answered in negative but the truth was that the American International Underwriters of the Philippines (AIU) had already declined similar application for insurance by the plaintiff with respect of the above-mentioned vehicles. The defendant issued to the plaintiff two Commercial Vehicle Comprehensive Policies covering the said properties. The vehicles mentioned figured in an accident resulting in the total loss of the tractor and partial damage to the trailer. Plaintiff demand upon the defendant for the payment to him the total amt. of damages resulting from the accident. On April 28, 1960, defendant rejected the claim on the ground of concealment of a material fact: that the insured property previously been declined insurance by another company. May 27, 1960, the plaintiff filed with the Office of the Insurance Commissioner a complaint against the said company. As suggested, the plaintiff was willing to submit his claim to arbitration but was contested by the defendant since "the claim of the plaintiff cannot be resolved by arbitration, as recourse to arbitration referred to in the policy contract, envisioned only differences or disputes, 'with respect to the amount of the company's liability,' and not to cases where the company does not admit its liability to the insured. With this rejection, the plaintiff filed his complaint with the CFI of Manila on September 19,1961. Against the above complaint, the defendant-appellee filed on September 29, 1961 a motion to dismiss on the ground of prescription. The latter argued that the plaintiff's claim had already prescribed since it was not filed within twelve months from its rejection by the insurance company as stipulated under paragraph 9 of the General Conditions of Commercial Vehicle Comprehensive Policy Nos. 5598 and 5599, to wit:

If a claim be made and rejected and an action or suit be not commenced within twelve months after such rejection or (in case of an arbitration taking place as provided herein) within twelve months after the arbitrator, arbitrators, or umpire shall have made their award then the claim shall for all purposes be deemed to have been abandoned and shall not thereafter be recovered hereunder.

Page 24 of 154

INSURANCE G01 CASE DIGESTS ISSUE: Whether the complaint filed by the plaintiff-appellant with the Office of the Insurance Comm. on May 27,1960 a commencement of an "action or suit" within the meaning and intent of general condition? No. justice, the claim may not properly be categorized under either term.

RATIO: "Action" and "suit":

Rule 2, Section 1 of the Rules of Court Section 1. Action defined.Action means an ordinary suit in a Court of Justice by which one party prosecutes another for the enforcement or protection of a right, or the prevention or redress of a wrong. (Emphasis supplied.)

An "action or suit" is essentially "for the enforcement or protection of a right, or the prevention or redress of a wrong." (Rule 2, Sec. 1, Rules of Court). There is nothing in the Insurance Law, which empowers the Insurance Commissioner to adjudicate on disputes relating to an insurance company's liability to an insured under a policy issued by the former to the latter. The validity of an insured's claim under a specific policy, its amount, and all such other matters as might involve the interpretation and construction of the insurance policy, are issues which only a regular court of justice may resolve and settle. Consequently, the complaint filed by the appellant herein with the Office of the Insurance Commission could not have been an "action or suit."

Jurisprudence Suit is the prosecution or pursuit of some claim or demand in a court of justice or any proceeding in a court of justice in which a plaintiff pursues his remedy to recover a right or claim. (Emphasis supplied.)

- Upon the authorities, therefore, it is settled that the terms "action" and "suit" are synonymous. Moreover, it is clear that the determinative or operative fact which converts a claim into an "action or suit" is the filing of the same with a "court or justice." Filed elsewhere, as with some other body or office not a court of Page 25 of 154

INSURANCE G01 CASE DIGESTS 83. ACCFA V. ALPHA INSURANCE , 24 SCRA 151 (1968) FACTS from the making of a claim for the loss upon which the action is based, is valid or void? VOID HELD Consequently, the condition of the bond in question, limiting the period for bringing action thereon, is subject to the provisions of Section 61-A of the Insurance Act (No. 2427), as amended by Act 4101 of the pre-Commonwealth Philippine Legislature, prescribing that SEC. 61-A A condition, stipulation or agreement in any policy of insurance, limiting the time for commencing an action thereunder to a period of less than one year from the time when the cause of action accrues is void. The cause of action does not accrue until the party obligated refuses, expressly or impliedly, to comply with its duty (in this case, to pay the amount of the bond). The year for instituting action in court must be reckoned, therefore, from the time of appellee's refusal to comply with its bond; it can not be counted from the creditor's filing of the claim of loss, for that does not import that the surety company will refuse to pay. In so far, therefore, as condition eight of the bond requires action to be filed within one year from the filing of the claim for loss, such stipulation contradicts the public policy expressed in Section 61-A of the Philippine Insurance Act. Condition eight of the bond, therefore, is null and void, and the appellant is not bound to comply with its provisions. Page 26 of 154

1. Alpha insurance issued 5,000 pesos bond to guarantee FACOMA against loss on account of personal dishonesty, amounting to larceny or estafa of its Secretary Treasurer. 2. FACOMA assigned its right to ACCFA, with approval of principal and surety 3. During effectivity of bond, Secretary Treasurer converted and misappropriated to his personal benefit 11,000 of FACOMA funds, which 6,307.33 belonged to ACCFA. 4. Upon discovery of the loss, ACCFA immediately notified the surety company, but despite repeated demands, surety refused to pay 5. Alpha insurance moved to dismiss the complaint for failure to state a cause of action, giving as reason that the same was filed more than one year after plaintiff made claim for loss, contrary to the eighth condition of the bond 6. Granted and motion to dismiss upon reconsideration kaya nagappeal si ACCFA.

ISSUE Whether or not the provision of a fidelity bond that no action shall be had or maintained thereon unless commenced within one year

INSURANCE G01 CASE DIGESTS 84. SAURA I MPORT & EXPORT V . PHIL. INTL SURETY 8 SCRA 143 (1963) 7. The building and its contents, worth P40,685.69 were burned. Saura filed a claim with the Insurer and mortgagee Bank. 8. Upon the presentation of notice of loss with the PNB, Saura learned for the first time that the policy had previously been cancelled by the insurer, when Saura's folder in the Bank's filed was opened and the notice of cancellation (original and duplicate) sent by the Insurer to the Bank, was found. 9. Upon refusal of the Insurer Philippine International Surety to pay the amount of the insurance this present case filed with the Manila CFI against the Insurer, and the PNB was later included as party defendant, after it had refused to prosecute the case jointly with Saura Import & Export Co., Inc. ISSUE: Whether or not there is a valid cancellation of the fire insurance policy. HELD: NO RATIO DECIDENDI: The policy in question does not provide for the notice, its form or period. The Insurance Law, Act No. 2427, does not likewise provide for such notice. This being the case, it devolves upon the Court to apply the generally accepted principles of insurance, regarding cancellation of the insurance policy by the insurer. From what has been stated, actual notice of cancellation in a clear and unequivocal manner, preferably in writing, in view of the importance of an insurance contract, should be given by the insurer to the insured, so that the latter might be given an Page 27 of 154

G.R. No. L-15184, May 31, 1963 PAREDES, J. Topic: Cancellation of non-life policy Sec. 62 FACTS: 1. Saura Import & Export Co Inc., mortgaged to PNB, a parcel of land, to secure the payment of promissory note of P27, 000.00. 2. The mortgage was amended to guarantee an increased amount, bringing the total mortgaged debt to P37, 000.00. 3. Erected on the land mortgaged, was a building of strong materials owned by the mortgagor Saura Import & Export Co., Inc., which had always been covered by insurance, many years prior to the mortgage contract. 4. Pursuant to the requirement, Saura insured the building and its contents with the Philippine International Surety for P29,000.00 against fire for the period of one year from October 2, 1954. 5. The insurance policy was endorsed to the mortgagee PNB, in a Memo which states Loss if any, payable to the Philippine National Bank as their interest may appear, subject to the terms, conditions and warranties of this policy. 6. Barely thirteen (13) days after the issuance of the policy, the insurer cancelled the same. Notice of the cancellation was given to appellee bank in writing. .

INSURANCE G01 CASE DIGESTS opportunity to obtain other insurance for his own protection. The notice should be personal to the insured and not to and/or through any unauthorized person by the policy. In the case at bar, the defendant insurance company, must have realized the paramount importance of sending a notice of cancellation, when it sent the notice of cancellation of the policy to the defendant bank (as mortgagee), but not to the insured with which it (insurance company) had direct dealing. It was the primary duty of the defendant-appellee insurance company to notify the insured, but it did not. It should be stated that the house and its contents were burned on April 6, 1955, at the time when the policy was enforced (October 2, 1954 to October 2, 1955); and that under the facts, as found by the trial court, to which We are bound, it is evident that both the insurance company and the appellee bank failed, wittingly or unwittingly, to notify the insured appellant Saura of the cancellation made. The defendant insurance company contends that it gave notice to the defendant-appellee bank as mortgagee of the property, and that was already a substantial compliance with its duty to notify the insured of the cancellation of the policy. But notice to the bank, as far appellant herein is concerned, is not effective notice. 85. MALAYAN INSURANCE V . CRUZ-ARNALDO , 154 SCRA 672 (1987)

Topic: Cancellation of non-life policy Facts: In 1981, petitioner Malayan issued to the private respondent Pinca, a Fire Insurance Policy on her property for P14,000 effective July 22, 1981 untul July 22, 1982 Malayan allegedly cancelled the policy for non-payment of premiums and sent notice to Pinca (October 15, 1981) A couple of months after, payment of the premium for Pinca was received by Adora, an agent of Malayan (December 24, 1981) Adora remitted the payment to Malayan Three days after, Pincas property was completely burned (Jan 18, 1982) A couple of weeks after, Malayan returned Pincas payment as the policy was previously cancelled Pinca made demands for the proceeds of the policy, but Malayan refused. The Insurance Commission decided in favor of Pinca. Malayan: There was no payment of premium and that the policy had been canceled before the occurrence of the loss

Issue: Whether or not Malayan should be liable for the proceeds of the policy

Held: YES. Malayans argument was not acceptable. Malayan relies on Sec 77 of the Insurance Code: Page 28 of 154

INSURANCE G01 CASE DIGESTS SEC. 77. An insurer is entitled to payment of the premium as soon as the thing is exposed to the peril insured against. Notwithstanding any agreement to the contrary, no policy or contract of insurance issued by an insurance company is valid and binding unless and until the premium thereof has been paid, except in the case of a life or an industrial life policy whenever the grace period provision applies. The above provision is not applicable because payment of the premium was in fact eventually made. The premium invoice issued to Pinca at the time of delivery of the policy on June 7, 1981 was stamped, Payment Received of the amount of P930.60 on 12-2481 by Adora. It suggests an understanding between Malayan and the insured that payment could be made later. Adora at her own risk as she was bound to first check his authority to receive it.

MALAYAN is taking an inconsistent stand. While contending that acceptance of the premium payment was prohibited by the policy, it at the same time insists that the policy never came into force because the premium had not been paid. One surely, cannot have his cake and eat it too. We do not share MALAYAN's view that there was no existing insurance at the time of the loss sustained by Pinca because her policy never became effective for non-payment of premium. Payment was in fact made, rendering the policy operative as of June 22, 1981, and removing it from the provisions of Article 77, Thereafter, the policy could be cancelled on any of the supervening grounds enumerated in Article 64 (except "nonpayment of premium") provided the cancellation was made in accordance therewith and with Article 65.

The payment was made on December 24, 1981, and the fire occured on January 18, 1982. One wonders: suppose the payment had been made and accepted in, say, August 1981, would the commencement date of the policy have been changed to the date of the payment, or would the payment have retroacted to July 22, 1981? If MALAYAN accepted the payment in December 1981 and the insured property had not been burned, would that policy not have expired just the same on July 22, 1982, pursuant to its original terms, and not on December 24, 1982? There is the petitioner's argument, however, that Adora was not authorized to accept the premium payment because six months had elapsed since the issuance by the policy itself. It is argued that this prohibition was binding upon Pinca, who made the payment to

Section 64 reads as follows: SEC. 64. No policy of insurance other than life shall be cancelled by the insurer except upon prior notice thereof to the insured, and no notice of cancellation shall be effective unless it is based on the occurrence, after the effective date of the policy, of one or more of the following: (a) non-payment of premium; (b) conviction of a crime arising out of acts increasing the hazard insured against; Page 29 of 154

INSURANCE G01 CASE DIGESTS (c) discovery of fraud or material misrepresentation; (d) discovery of willful, or reckless acts or commissions increasing the hazard insured against; (e) physical changes in the property insured which result in the property becoming uninsurable;or (f) a determination by the Commissioner that the continuation of the policy would violate or would place the insurer in violation of this Code. As for the method of cancellation, Section 65 provides as follows: SEC. 65. All notices of cancellation mentioned in the preceding section shall be in writing, mailed or delivered to the named insured at the address shown in the policy, and shall state (a) which of the grounds set forth in section sixtyfour is relied upon and (b) that, upon written request of the named insured, the insurer will furnish the facts on which the cancellation is based. A valid cancellation must, therefore, require concurrence of the following conditions: (1) There must be prior notice of cancellation to the insured; (2) The notice must be based on the occurrence, after the effective date of the policy, of one or more of the grounds mentioned; (3) The notice must be (a) in writing, (b) mailed, or delivered to the named insured, (c) at the address shown in the policy; (4) It must state (a) which of the grounds mentioned in Section 64 is relied upon and (b) that upon written request of the insured, the insurer will furnish the facts on which the cancellation is based. All MALAYAN's offers to show that the cancellation was communicated to the insured is its employee's testimony that the said cancellation was sent "by mail through our mailing section." without more It stands to reason that if Pinca had really received the said notice, she would not have made payment on the original policy on December 24, 1981. Instead, she would have asked for a new insurance, effective on that date and until one year later, and so taken advantage of the extended period. The Court finds that if she did pay on that date, it was because she honestly believed that the policy issued on June 7, 1981, was still in effect and she was willing to make her payment retroact to July 22, 1981, its stipulated commencement date. Adora, incidentally, had not been informed of the cancellation either and saw no reason not to accept the said payment

Page 30 of 154

INSURANCE G01 CASE DIGESTS

WARRANTIES
86. AMERICAN HOME INSURANCE V. TANTUCO ENTERPRISES, 366 SCRA 740 (2011) G.R. No. 138941October 8, 2001

ISSUE: Whether or not the Court of Appeals erred in its legal interpretation of 'Fire ExtinguishingAppliances Warranty' of the policy.

FACTS: Respondent Tantuco Enterprises, Inc. is engaged in the coconut oil milling and refining industry.It owns two oil mills which were separately covered by fire insurance policies issued by petitionerAmerican Home Assurance Co., Philippine Branch. The first oil mill was insured for P3,000,000.00 underPolicy No. 3067432324-3 for the period March 1, 1991 to 1992. The new oil mill was insured forP6,000,000.00 under Policy No. 306-7432321-9 for the same term. Official receipts indicating paymentfor the full amount of the premium were issued by the petitioner's agent.A fire that broke out in the early morning of September 30,1991 gutted and consumed the new oil mill.Respondent immediately notified the petitioner of the incident but petitioner rejected respondent'sclaim for the insurance proceeds on the ground that no policy was issued by it covering the burned oilmill. It stated that the description of the insured establishment referred to another building thus: "Ourpolicy nos. 306-7432321-9 (Ps 6M) and 3067432324-4 (Ps 3M) extend insurance coverage to your oilmill under Building No. 5, whilst the affected oil mill was under Building No. 14."

HELD: In construing the words used descriptive of a building insured, the greatest liberality is shown bythe courts in giving effect to the insurance. In view of the custom of insurance agents to examinebuildings before writing policies upon them, and since a mistake as to the identity and character of thebuilding is extremely unlikely, the courts are inclined to consider that the policy of insurance covers anybuilding which the parties manifestly intended to insure, however inaccurate the description may be.Notwithstanding, therefore, the misdescription in the policy, it is beyond dispute, to our mind, that whatthe parties manifestly intended to insure was the new oil mill. If the parties really intended to protectthe first oil mill,then there is no need to specify it as new . In determining what the parties intended, the courts will read and construe the policy as a whole and if possible, give effect to all the parts of the contract, keeping in mind always, however, the prime rulethat in the event of doubt, this doubt is to be resolved against the insurer. In determining the intent of the parties to the contract, the courts will consider the purpose and object of the contract.

Page 31 of 154

INSURANCE G01 CASE DIGESTS 87. ANG GIOK V. SPRINGFIELD FIRE & MUTUAL INSURANCE , 56 PHIL. 375 (1931) 56 Phil. 375 (1931) Warranties Facts: Ang Giok Chip doing business under the name and style of Hua Bee Kong Si was formerly the owner of a warehouse situated in Manila. The contents of the warehouse were insured with the three insurance companies for the total sum of P60,000. One insurance policy, in the amount of P10,000, was taken out with the Springfield Fire & Marine Insurance Company the warranty includes a Rider: no hazardous material be stored in the building, that the Insured be permitted to stored a small quantity of the hazardous goods specified below, but not exceeding in all 3 per cent of the total value of the whole of the goods or merchandise contained in said warehouse. The warehouse was destroyed by fire on January 11, 1928, while the policy issued by the latter company was in force. The plaintiff instituted action in the Court of First Instance of Manila against the defendant to recover a proportional part of the loss coming to P8,170.59. Four special defenses were interposed on behalf of the insurance company, one being planted on a violation of warranty F fixing the amount of hazardous goods which might be stored in the insured building. The CFI rendered in favor of the plaintiff. But the SC held that it is unnecessary for us to discuss three of the four special defenses which were made by the insurance company. We think, however, that it would be a reasonable deduction to conclude that more than 3 per cent of the total value of the merchandise contained in the warehouse constituted hazardous goods, and that this per cent reached as high as 39. We place reliance on the consular invoices and on the testimony of the adjuster, Herridge. As such only one issue is to be resolved. Issue: WON a warranty referred to in the policy as forming part of the contract of insurance and in the form of a rider to the insurance policy, is null and void because not complying with the Philippine Insurance Act? Held: No, the Philippine Law was taken verbatim from law of California. Thus, the court should follow in fundamental points, at least, the construction placed by California courts on a California Law. The SC reached the definite conclusion that warranty F, a rider attached to the face of the insurance policy, and referred to in contract of insurance, is valid and sufficient under section 65 of the Insurance Act. Ratio: Section 65 of the Insurance Act and its counterpart, section 265 of the Civil Code of California, will bear analysis as tested by reason and authority. The law says that every express warranty must be "contained in the policy itself." The word "contained," according to the dictionaries, means "included," inclosed," "embraced," "comprehended," etc. When, therefore, the courts speak of a rider attached to the policy, and thus "embodied" therein, or of a warranty "incorporated" in the policy, it is believed that the phrase "contained in the policy itself" must necessarily include such rider and warranty. The SC thinks it wrong to hold that the California law represents a radical departure from the basic principles governing the law of insurance. We turn to two of such well recognized doctrines. In the first place, it is well settled that a rider attached to a policy is a part of the contract, to the same extent and with like effect as it actually embodied therein. (I Couch, Cyclopedia of Insurance Law, sec. 159.) In the second place, it is equally well settled that an express Page 32 of 154

INSURANCE G01 CASE DIGESTS warranty must appear upon the face of the policy, or be clearly incorporated therein and made a part thereof by explicit reference, or by words clearly evidencing such intention. (4 Couch, Cyclopedia of Insurance Law, sec. 862.) Referring to the jurisprudence of California, another rule of insurance adopted in that State is in point. It is admitted that the policy before us was accepted by the plaintiff. The receipt of this policy by the insured without objection binds both the acceptor and the insured to the terms thereof. The insured may not thereafter be heard to say that he did not read the policy or know its terms, since it is his duty to read his policy and it will be assumed that he did so. 88. QUA CHEE GAN V. LAW UNION AND ROCK INSURANCE , 98 PHIL 85 (1955) QUA CHEE GAN vs LAW UNION AND ROCK INSURANCE represented by agent, Warner, Barnes and Co., Ltd. (December 17, 1955) FACTS: Before the last war, Qua Chee Gan owned four warehouses or bodegas in Tabaco, Albay, used for the storage of stocks of copra and of hemp, baled and loose, in which the he dealth extensively. They had been insured with Law Union and Rock Insurance since 1937, and the loss made payable to the Philippine National Bank as mortgage of the hemp and crops, to the extent of its interest. On June, 1940, the insurance stood as follows: Policy No. Property Insured Amount 2637164 Bodega No. 1 (Building) P15,000.00 (Exhibit "LL") 2637165 Bodega No. 2 (Building) 10,000.00 (Exhibit "JJ") Bodega No. 3 (Building) 25,000.00 Bodega No. 4 (Building) 10,000.00 Hemp Press moved by steam 5,000.00 engine 2637345 Merchandise contents (copra and 150,000.00 (Exhibit "X") empty sacks of Bodega No. 1) 2637346 Merchandise contents (hemp) of 150,000.00 (Exhibit "Y") Bodega No. 3 2637067 Merchandise contents (loose 5,000.00 (Exhibit "GG") hemp) of Bodega No. 4 Total P370,000.00

Page 33 of 154

INSURANCE G01 CASE DIGESTS On July 21, 1940, fire broke out near the Bodegas and last for almost one week which completely destroyed Bodegas 1,2 and 4. Qua Chee Gan claims 398,562.81 (reduced to 370,000, the amount in the insurance). Fire adjusters conducted an investigation. Law Union resisted the payment claiming violation of warranties and conditions, filing of fraudulent claims, and that the fire had been deliberately caused by the insured/other persons in connivance. Qua Chee Gan and Qua Chee Pao (brother)were tried for Arson but the trial court acquitted them. Qua Chee Gan then filed a civil case to claim the proceeds of the fire insurance policies. CFI ruled in favour of him. PNB filed a complaint in intervention but it was dismissed because Qua Chee Gan managed to pay his indebtedness. ISSUE: WON Qua Chee Gan breached provisions of warranty that he cannot be entitled to claim insurance proceeds. HELD: NO 1st cause of action - Hydrants: NO Memo. of Warranty. The undernoted Appliances for the extinction of fire being kept on the premises insured hereby, and it being declared and understood that there is an ample and constant water supply with sufficient pressure available at all seasons for the same, it is hereby warranted that the said appliances shall be maintained in efficient working order during the currency of this policy, by reason whereof a discount of 2 1/2 per cent is allowed on the premium chargeable under this policy. Hydrants in the compound, not less in number than one for each 150 feet of external wall measurement of building, protected, with not less than 100 feet of hose piping and nozzles for every two hydrants kept under cover in convenient places, the hydrants being supplied with water pressure by a pumping engine, or from some other source, capable of discharging at the rate of not less than 200 gallons of water per minute into the upper story of the highest building protected, and a trained brigade of not less than 20 men to work the same. Since the bodegas had an external wall perimeter of 1,640 feet, there should have been 11 fire hydrants and Qua Chee Gan only had 2. Another pair is available also but it belongs to the municipality of Tabaco. Law Union is barred by estoppel because they know that the number of hydrants demanded in the rider never existed from the very beginning but they still issued the policies in question subject to such warranty, and received the corresponding premiums. It would be perilously close to conniving at fraud upon the insured to allow Law Union to claim now as void ab initio the policies that it had issued to the Qua Chee Gan without warning of their fatal defect, of which it was informed, and after it had misled the defendant into believing that the policies were effective. Court finds it unusual also that during the period for insurance, the insurer suddenly realized that the premises are hazardous and offered the rider warranty for a discount on the premium. Rule: Where the insurer, at the time of the issuance of a policy of insurance, has knowledge of existing facts which, if insisted on, would invalidate the contract from its very inception, such knowledge constitutes a waiver of conditions in the contract inconsistent with the facts, and the insurer is estopped thereafter from asserting the breach Page 34 of 154

INSURANCE G01 CASE DIGESTS of such conditions. The law is charitable enough to assume, in the absence of any showing to the contrary, that an insurance company intends to execute a valid contract in return for the premium received; and when the policy contains a condition which renders it voidable at its inception, and this result is known to the insurer, it will be presumed to have intended to waive the conditions and to execute a binding contract, rather than to have deceived the insured into thinking he is insured when in fact he is not, and to have taken his money without consideration. (29 Am. Jur., Insurance, section 807, at pp. 611-612.) Ratio: To allow a company to accept one's money for a policy of insurance which it then knows to be void and of no effect, though it knows as it must, that the assured believes it to be valid and binding, is so contrary to the dictates of honesty and fair dealing, and so closely related to positive fraud, as to the abhorent to fairminded men. It would be to allow the company to treat the policy as valid long enough to get the preium on it, and leave it at liberty to repudiate it the next moment. This cannot be deemed to be the real intention of the parties. To hold that a literal construction of the policy expressed the true intention of the company would be to indict it, for fraudulent purposes and designs which we cannot believe it to be guilty of (Wilson vs. Commercial Union Assurance Co., 96 Atl. 540, 543-544). The alleged violation of the warranty of 100 feet of fire hose for every two hydrants, must be equally rejected, since it is based on the assumption that the insured was bound to maintain no less than eleven hydrants (one per 150 feet of wall), which requirement appellant is estopped from enforcing. As to maintenance of a trained fire brigade of 20 men, the record is preponderant that the same was organized, and drilled, from time to give, although not maintained as a permanently separate unit, which the warranty did not require. Anyway, it would be unreasonable to expect the insured to maintain for his compound alone a fire fighting force that many municipalities in the Islands do not even possess. 2nd cause of action Hemp Warranty: No Prohibition on Oils (animal and/or vegetable and/or mineral and/or their liquid products having a flash point below 300 degrees Fahrenheit Law Union argues that Qua Chee Gan admitted that he had 36 cans of gasoline in Bodega 2. It is well to note that gasoline is not specifically mentioned among the prohibited articles listed in the so-called "hemp warranty." It is ambiguous and uncertain; for in ordinary parlance, "Oils" mean "lubricants" and not gasoline or kerosene. And how many insured, it may well be wondered, are in a position to understand or determine "flash point below 300 Fahrenheit. Here, again, by reason of the exclusive control of the insurance company over the terms and phraseology of the contract, the ambiguity must be held strictly against the insurer and liberraly in favor of the insured, specially to avoid a forfeiture. We see no reason why the prohibition of keeping gasoline in the premises could not be expressed clearly and unmistakably, in the language and terms that the general public can readily understand, without resort to obscure esoteric expression. If the company intended to rely upon a condition of that character, it ought to have been plainly expressed in the policy. Another point that is in favor of the insured is that the gasoline kept in Bodega No. 2 was only incidental to his business, being no more than a customary 2 day's supply for the five or six motor vehicles used for transporting of the stored merchandise. "It is well settled that the keeping of inflammable oils on the premises though prohibited by the policy does not void it if such keeping is incidental to the Page 35 of 154

INSURANCE G01 CASE DIGESTS business." Bachrach vs. British American Ass. Co., 17 Phil. 555, 560); and "according to the weight of authority, even though there are printed prohibitions against keeping certain articles on the insured premises the policy will not be avoided by a violation of these prohibitions, if the prohibited articles are necessary or in customary use in carrying on the trade or business conducted on the premises." (45 C. J. S., p. 311; also 4 Couch on Insurance, section 966b). It should also be noted that the "Hemp Warranty" forbade storage only "in the building to which this insurance applies and/or in any building communicating therewith", and it is undisputed that no gasoline was stored in the burned bodegas, and that "Bodega No. 2" which was not burned and where the gasoline was found, stood isolated from the other insured bodegas. 89. PIONEER INSURANCE V. YAP, 61 SCRA 426 (1974)
GR L-36232; 61 SCRA 426, (1947) Topic: Warranties: Effect Warranties Facts: Olivia Yap owned a two-storey building in Manila where she sold shopping bags and footwear. Her son-in-law Chua Soon Poon was in charge of the store. She took a fire insurance of Php25,000 on the building with the petitioner to cover her stocks and furniture. One of the conditions in the policy stated that the insured should notify the insurance company of any insurance policies already effected or may be subsequently effected covering the property unless such notice be given and the particulars of such insurance or insurances be stated in, or endorsed on this Policy by or on behalf of the Company before the occurrence of any loss or damage, all benefits under this Policy shall be forfeited any false declarations will render this policy null and void. At the time the petitioner issued the policy, Great American Insurance issued another policy on the same properties of Yap. This issuance was noted with the petitioner. Later in the year however, she took out another policy with Federal Insurance covering the same properties again, only this time she did not notify the petitioner. In December 1962, a fire burned down the building. Yap filed an insurance claim against the petitioner but the latter denied. The petitioner claimed there was a breach of contract. Yap filed a complaint for payment of the value of the insurance policy with the petitioner. Petitioner claimed she is not entitled because (1) none of the properties covered by the policy were destroyed by the fire, and (2) even if these properties were burned, she is still not entitled for breach of contract. Petitioner claims the proceeds were forfeited. The RTC decided in favor of Yap. The CA affirmed. Issue: Should the petitioner be liable based on the fire insurance policy? Held: NO

Page 36 of 154

INSURANCE G01 CASE DIGESTS


Rationale: There was a violation by respondent Oliva Yap of the co-insurance clause contained in Policy No. 4219 that resulted in the avoidance of petitioner's liability. The insurance policy for P20,000.00 issued by the Great American Insurance Company covering the same properties of respondent Yap and duly noted on Policy No. 4219 as c-insurance, ceased, by agreement of the parties (Exhibit "1-L"), to be recognized by them as a co-insurance policy. The Court of Appeals says that the Great American Insurance policy was substituted by the Federal Insurance policy for the same amount, and because it was a mere case of substitution, there was no necessity for its endorsement on Policy No. 4219. This finding, as well as reasoning, suffers from several flaws. There is no evidence to establish and prove such a substitution. If anything was substituted for the Great American Insurance policy, it could only be the Northwest Insurance policy for the same amount of P20,000.00. The endorsement (Exhibit "1-K") quoted above shows the clear intention of the parties to recognize on the date the endorsement was made (August 29, 1962), the existence of only one coinsurance, and that is the Northwest Insurance policy, which according to the stipulation of the parties during the hearing, was issued on August 20, 1962 (t.s.n., January 12, 1965, pp. 3-4) and endorsed only on August 20, 1962. The finding of the Court of Appeals that the Great American Insurance policy was substituted by the Federal Insurance policy is unsubstantiated by the evidence of record and indeed contrary to said stipulation and admission of respondent, and is grounded entirely on speculation, surmises or conjectures, hence, not binding on the Supreme Court. By the plain terms of the policy, other insurance without the consent of petitioner would ipso facto avoid the contract. It required no affirmative act of election on the part of the company to make operative the clause avoiding the contract, wherever the specified conditions should occur. Its obligations ceased, unless, being informed of the fact, it consented to the additional insurance. The validity of a clause in a fire insurance policy to the effect that the procurement of additional insurance without the consent of the insurer renders ipso facto the policy void is well-settled: In Milwaukee Mechanids' Lumber Co., vs. Gibson, 199 Ark. 542, 134 S. W. 2d 521, 522, a substantially identical clause was sustained and enforced, the court saying: "The rule in this state and practically all of the states is to the effect that a clause in a policy to the effect that the procurement of additional insurance without the consent of the insurer renders the policy void is a valid provision. The earlier cases of Planters Mutual Insurance Co., vs. Green, 72 Ark. 305, 80 S.W. 92, are to the same effect." And see Vance, Insurance, 2nd Ed., 725. (Reach vs. Arkansas Farmers Mut. Fire Ins. Co., [Ark. Nov. 14, 1949] 224 S. W. 2d 48, 49.) 2. Where a policy contains a clause providing that the policy shall be void if insured has or shall procure any other insurance on the property, the procurement of additional insurance without the consent of the insurer avoids the policy." (Planters' Mut. Ins. Ass'n vs. Green [Supreme Court of Arkansas, March 19, 1904] 80 S.W. 151.) The obvious purpose of the aforesaid requirement in the policy is to prevent over-insurance and thus avert the perpetration of fraud. The public, as well as the insurer, is interested in preventing the situation in which a fire would be profitable to the insured. According to Justice Story: "The insured has no right to complain, for he assents to comply with all the stipulation on his side, in order to entitle himself to the benefit of the contract, which, upon reason or principle, he has no right to ask the court to dispense with the performance of his own part of the agreement, and yet to bind the other party to obligations, which, but for those stipulation would not have been entered into."

Page 37 of 154

INSURANCE G01 CASE DIGESTS 90. PRUDENTIAL GUARANTEE V. TRANS A SIA SHIPPING, 491 SCRA 411 (2006) Chico-Nazario, J. Facts 1. TRANS-ASIA is the owner of the vessel M/V Asia Korea. 2. In consideration of payment of premiums, defendant [PRUDENTIAL] insured M/V Asia Korea for loss/damage of the hull and machinery arising from perils, inter alia, of fire and explosion for the sum of P40 Million, beginning [from] the period [of] July 1, 1993 up to July 1, 1994. 3. This is evidenced by a Marine Policy. 4. On October 25, 1993, while the policy was in force, a fire broke out while [M/V Asia Korea was] undergoing repairs at the port of Cebu. 5. On October 26, 1993 plaintiff [TRANS-ASIA] filed its notice of claim for damage sustained by the vessel. This is evidenced by a letter/formal claim of even date. 6. Plaintiff [TRANS-ASIA] reserved its right to subsequently notify defendant [PRUDENTIAL] as to the full amount of the claim upon final survey and determination by average adjuster Richard Hogg International (Phil.) of the damage sustained by reason of fire. An adjusters report on the fire in question was submitted by Richard Hogg International. 7. Plaintiff [TRANS-ASIA] executed a document denominated "Loan and Trust receipt in which Trans-Asia received from Prudential 3M. 8. Thereafter, Prudential denied the insurance claim of TransAsia because Trans-Asia violated the Warranty which stated, "WARRANTED VESSEL CLASSED AND CLASS MAINTAINED". 9. Prudential requested for the return of the 3M. 10. TRANS-ASIA filed a Complaint5 for Sum of Money against PRUDENTIAL with the RTC of Cebu City alleging that the 3M was the balance of the indemnity due. 11. Prudential answered saying that they wanted the 3M back because Trans-Asia violated the warranty abovementioned. 12. RTC- in favor of Prudential. a. Trans-Asia was required to maintain the vessel and found that Trans-Asia violated it. 13. CA reversed. a. Prudential was not able to prove that Trans-Asia violated the warranty. b. The rider stating the warranty clause was attached without the intervention of Trans-Asia. c. The loan was in effect a subrogation. 3M was partial payment of the indemnity. 14. Hence, this petition.

Issues 1. Whether or not Trans-Asia violated its warranty to maintain the vessel. o NO. 2. Whether or not the loan agreement as evidenced by the trust receipt was a form of subrogation. o Yes. Ruling: First Issue Page 38 of 154

INSURANCE G01 CASE DIGESTS We sustain the findings of the Court of Appeals that PRUDENTIAL was not successful in discharging the burden of evidence that TRANS-ASIA breached the subject policy condition on CLASSED AND CLASS MAINTAINED. Foremost, PRUDENTIAL, through the Senior Manager of its Marine and Aviation Division, Lucio Fernandez, made a categorical admission that at the time of the procurement of the insurance contract in July 1993, TRANS-ASIAs vessel, "M/V Asia Korea" was properly classed by Bureau Veritas. We are not unmindful of the clear language of Sec. 74 of the Insurance Code which provides that, "the violation of a material warranty, or other material provision of a policy on the part of either party thereto, entitles the other to rescind." It is generally accepted that "[a] warranty is a statement or promise set forth in the policy, or by reference incorporated therein, the untruth or non-fulfillment of which in any respect, and without reference to whether the insurer was in fact prejudiced by such untruth or nonfulfillment, renders the policy voidable by the insurer."25 However, it is similarly indubitable that for the breach of a warranty to avoid a policy, the same must be duly shown by the party alleging the same. We cannot sustain an allegation that is unfounded. Consequently, PRUDENTIAL, not having shown that TRANS-ASIA breached the warranty condition, CLASSED AND CLASS MAINTAINED, it remains that TRANSASIA must be allowed to recover its rightful claims on the policy. Assuming arguendo that TRANS-ASIA violated the policy condition on WARRANTED VESSEL CLASSED AND CLASS MAINTAINED, PRUDENTIAL made a valid waiver of the same. The Court of Appeals, in reversing the Judgment of the RTC which held that TRANS-ASIA breached the warranty provision on CLASSED AND CLASS MAINTAINED, underscored that PRUDENTIAL can be deemed to have made a valid waiver of TRANS-ASIAs breach of warranty as alleged, ratiocinating, thus: Third, after the loss, Prudential renewed the insurance policy of Trans-Asia for two (2) consecutive years, from noon of 01 July 1994 to noon of 01 July 1995, and then again until noon of 01 July 1996. This renewal is deemed a waiver of any breach of warrant. Second Issue What is clear from the wordings of the so-called "Loan and Trust Receipt Agreement" is that appellant is obligated to hand over to appellee "whatever recovery (Trans Asia) may make and deliver to (Prudential) all documents necessary to prove its interest in the said property." For all intents and purposes therefore, the money receipted is payment under the policy, with Prudential having the right of subrogation to whatever net recovery Trans-Asia may obtain from third parties resulting from the fire. In the law on insurance, subrogation is an equitable assignment to the insurer of all remedies which the insured may have against third person whose negligence or wrongful act caused the loss covered by the insurance policy, which is created as the legal effect of payment by the insurer as an assignee in equity. The loss in the first instance is that of the insured but after reimbursement or compensation, it becomes the loss of the insurer. It has been referred to as the doctrine of substitution and rests on the principle that substantial justice should be attained regardless of form, that is, its Page 39 of 154

INSURANCE G01 CASE DIGESTS basis is the doing of complete, essential, and perfect justice between all the parties without regard to form.31 We agree. Notwithstanding its designation, the tenor of the "Loan and Trust Receipt" evidences that the real nature of the transaction between the parties was that the amount of P3,000,000.00 was not intended as a loan whereby TRANSASIA is obligated to pay PRUDENTIAL, but rather, the same was a partial payment or an advance on the policy of the claims due to TRANS-ASIA.

SC affirms. Petition denied. Side Issue on attorneys fees. In pursuant to Sec. 244 of the Insurance Code, there was an unreasonable delay on the part of PRUDENTIAL to pay TRANS-ASIA, as in fact, it refuted the latters right to the insurance claims, from the time proof of loss was shown and the ascertainment of the loss was made by the insurance adjuster. Evidently, PRUDENTIALs unreasonable delay in satisfying TRANS-ASIAs unpaid claims compelled the latter to file a suit for collection.

Page 40 of 154

INSURANCE G01 CASE DIGESTS

PREMIUM
o 91. ARCE V. CAPITAL INSURANCE , 117 SCRA 63 (1982) COMPANY/INSURER: Capital Insurance INSURED: Pedro Arce

COMPANY to have his signature on the check Identified preparatory to encashment. At that time the COMPANY reiterated that the check was given "not as an obligation, but as a concession" because the renewal premium had not been paid, The INSURED cashed the check but then sued the COMPANY on the policy. Issue

Facts Arces house was insured under a fire insurance policy On November 27, 1965, the insurer sent to the insured a Renewal Certificate to cover the period December 5, 1965 to December 5, 1966. The COMPANY also requested payment of the corresponding premium in the amount of P 38.10. Anticipating that the premium could not be paid on time, the INSURED, thru his wife, promised to pay it on January 4, 1966. The COMPANY accepted the promise but the premium was not paid on January 4, 1966. On January 8, 1966, the house of the INSURED was totally destroyed by fire. On January 10, 1966, INSURED's wife presented a claim for indemnity to the COMPANY. o She was told that no indemnity was due because the premium on the policy was not paid. o Nonetheless the COMPANY tendered a check for P300.00 as financial aid which was received by the INSURED's daughter, Evelina R. Arce as full settlement o The INSURED and his wife went to the office of the

Whether or not the insurer can be held liable despite the nonpayment of the premiums Held and Ratio No Relevant Provisions Insurance Code SEC. 72. An insurer is entitled to payment of premium as soon as the thing insured is exposed to the perils insured against, unless there is clear agreement to grant credit extension for the premium due. No policy issued by an insurance company is valid and binding unless and until the premium thereof has been paid Contract Stipulation IT IS HEREBY DECLARED AND AGREED that not. withstanding anything to the contrary contained in the within policy, this insurance will be deemed valid and binding upon the Company only when the premium and documentary stamps therefor have actually been paid in full and duly acknowledged in an official receipt signed by an authorized official/representative of the Company, Page 41 of 154

INSURANCE G01 CASE DIGESTS Ruling It is obvious from both the Insurance Act, as amended, and the stipulation of the parties that time is of the essence in respect of the payment of the insurance premium so that if it is not paid the contract does not take effect unless there is still another stipulation to the contrary. In the instant case, the INSURED was given a grace period to pay the premium but the period having expired with no payment made, he cannot insist that the COMPANY is nonetheless obligated to him. WON the non payment of premium due resulted in the cancellation of the contract of insurance Held: No Ratio: We can not agree with appellant's theory that non-payment by it of the premium due, produced the cancellation of the contract of insurance. Such theory would place exclusively in the hands of one of the contracting parties the right to decide whether the contract should stand or not. Rather the correct view would seem to be this: as the contract had become perfected, the parties could demand from each other the performance of whatever obligations they had assumed. In the case of the insurer, it is obvious that it had the right to demand from the insured the completion of the payment of the premium due or sue for the rescission of the contract. As it chose to demand specific performance of the insured's obligation to pay the balance of the premium, the latter's duty to pay is indeed indubitable. PHIL PHOENIX V WOODWORKS 92 SCRA 419 (1979) Facts: July 21, 1960: Woodworks, Inc. was issued a fire policy for its building machinery and equipment by Philippine Phoenix Surety & Insurance Co. for P500K covering July 21, 1960 to July 21, 1961. Woodworks did not pay the premium totalling to P10,593.36. April 19, 1961: It was alleged that Woodworks notified Philippine Phoenix the cancellation of the Policy so Philippine Phoenix credited P3,110.25 for the unexpired period of 94 days and demanded in writing the paymentof P7,483.11

92. PHIL. PHOENIX SURETY & ASSURANCE V. WOODWORKS, I NC., 20 SCRA 1270 (1967) Facts: That on April 1, 1960, plaintiff issued to defendant Fire Policy No. 9652 for the amount of P300,000.00, under the terms and conditions therein set forth in said policy a copy of which is hereto attached and made a part hereof as Annex "A"; That the premiums of said policy as stated in Annex "A" amounted to P6,051.95; the margin fee pursuant to the adopted plan as an implementation of Republic Act 2609 amounted to P363.72, copy of said adopted plan is hereto attached as Annex "B" and made a part hereof, the documentary stamps attached to the policy was P96.42; That the defendant paid P3,000.00 on September 22, 1960 under official receipt No. 30245 of plaintiff; That plaintiff made several demands on defendant to pay the amount of P3,522.09. Issue:

Page 42 of 154

INSURANCE G01 CASE DIGESTS

Woodworks refused stating that it need not pay premium "because the Insurer did not stand liable for any indemnity during the period the premiums were not paid." Philippine Phoenix filed with the CFI to recover its earned premium of P7,483.11 Woodworks: to pay the premium after the issuance of the policy put an end to the insurance contract and rendered the policy unenforceable CFI: favored Philippine Phoenix

The burden is on an insured to keep a policy in force by the payment of premiums, rather than on the insurer to exert every effort to prevent the insured from allowing a policy to elapse through a failure to make premium payments.

93. MAKATI TUSCANY CONDOMINIUM V. COURT OF APPEALS, 215 SCRA 463 (1992) 215 SCRA 462 BELLOSILLO; November 6, 1992 NATURE Appeal from decision of the CA FACTS - American Home Assurance Co. (AHAC), represented by American International Underwriters (Phils.), Inc., issued in favor of petitioner Makati Tuscany Condominium Corporation an insurance policy on the latter's building and premises, for the period 1 March 1982 to1 March 1983. The premium was paid on installments all of which were accepted by AHAC.

ISSUE: W/N there was a valid insurance contract despite no premium payment was paid

HELD: NO. Reversed

Policy provides for pre-payment of premium. To constitute an extension of credit there must be a clear and express agreement therefor and there nust be acceptance of the extension - none here Since the premium had not been paid, the policy must be deemed to have lapsed. failure to make a payment of a premium or assessment at the time provided for, the policy shall become void or forfeited, or the obligation of the insurer shall cease, or words to like effect, because the contract so prescribes and because such a stipulation is a material and essential part of the contract. This is true, for instance, in the case of life, health and accident, fire and hail insurance policies Explicit in the Policy itself is plaintiff's agreement to indemnify defendant for loss by fire only "after payment of premium" Compliance by the insured with the terms of the contract is a condition precedent to the right of recovery.

- A second policy was issued to renew the first one, this time covering the period 1 March 1983 to 1 March 1984. This was also pain in installment basis.

- A third policy was again issued for the period 1 March 1984 to 1 March 1985. For this, petitioner made two installment payments, both accepted by AHAC. Thereafter, Page 43 of 154

INSURANCE G01 CASE DIGESTS petitioner refused to pay the balance of the premium. AHAC filed an action to recover the unpaid balance of P314,103.05. the receipts ordinarily issued by AHAC on premium payments the only plausible conclusion is that AHAC has no right to demand their payment after the lapse of the term of said policy on March 1, 1985. Therefore, Tuscany was justified in refusing to pay the same.

- Petitioner explained that it discontinued the payment of premiums because the policy did not contain a credit clause in its favor and the receipts for the installment payments covering the policy for 1984-85, as well as the two (2) previous policies, stated the following reservations: 2. Acceptance of this payment shall not waive any of the company rights to deny liability on any claim under the policy arising before such payments or after the expiration of the credit clause of the policy; and 3. Subject to no loss prior to premium payment. If there be any loss such is not covered.

- CA modified the decision by ordering Tuscany to pay the balance of the premiums due on the third policy plus legal interest until fully paid, and affirming the denial of the counterclaim.

- Petitioner further claimed that the policy was never binding and valid, and no risk attached to the policy. It then pleaded a counterclaim for P152k for the premiums already paid for 1984-85, and in its answer with amended counterclaim, sought the refund of P924,206.10 representing the premium payments for 1982-85.

- Trial court dismissed the complaint and the counterclaim upon the following findings: (1) payment of the premiums of the three policies were made during the term of said policies, hence, it could not be said, inspite of the reservations, that no risk attached under the policies; (2) as regards the unpaid premiums, in view of the reservation in

Petitioners Claims Petitioner argues that where the premiums is not actually paid in full, the policy would only be effective if there is an acknowledgment in the policy of the receipt of premium pursuant to Sec. 78 of the Insurance Code. The absence of an express acknowledgment in the policies of such receipt of the corresponding premium payments, and petitioner's failure to pay said premiums on or before the effective dates of said policies rendered them invalid. Petitioner thus concludes that there cannot be a perfected contract of insurance upon mere partial payment of the premiums because under Sec. 77 of the Insurance Code, no contract of insurance is valid and binding unless the premium thereof has been paid, notwithstanding any agreement to the contrary. ISSUE WON payment by installment of the premiums due on an insurance policy invalidates the contract of insurance Page 44 of 154

INSURANCE G01 CASE DIGESTS

HELD Ratio Where the risk is entire and the contract is indivisible, the insured is not entitled to a refund of the premiums paid if the insurer was exposed to the risk insured for any period, however brief or momentary.

Reasoning - The obligation to pay premiums when due is ordinarily as indivisible obligation to pay the entire premium. Here, the parties herein agreed to make the premiums payable in installments, and there is no pretense that the parties never envisioned to make the insurance contract binding between them. And the insured never informed the insurer that it was terminating the policy because the terms were unacceptable.

- Section 78 of the Insurance Code in effect allows waiver by the insurer of the condition of prepayment by making an acknowledgment in the insurance policy of receipt of premium as conclusive evidence of payment so far as to make the policy binding despite the fact that premium is actually unpaid. Section 77 merely precludes the parties from stipulating that the policy is valid even if premiums are not paid, but does not expressly prohibit an agreement granting credit extension, and such an agreement is not contrary to morals, good customs, public order or public policy.

- At the very least, both parties should be deemed in estoppel to question the arrangement they have voluntarily accepted. Disposition Judgment affirmed. Costs against petitioner.

- There is nothing in Section 77 which suggests that the parties may not agree to allow payment of the premiums in installment, or to consider the contract as valid and binding upon payment of the first premium.

- The records clearly show that petitioner and private respondent intended subject insurance policies to be binding and effective notwithstanding the staggered payment of the premiums. Acceptance of payments speaks loudly of the insurer's intention to honor the policies it issued to petitioner. Page 45 of 154

INSURANCE G01 CASE DIGESTS 94. TIBAY V. COURT OF APPEALS, 257 SCRA 126 (1996) FACTS: Insurer: Fortune Insured: Violeta Tibay and/or Nicolas Ronaldo Property Insured: 2 storey residential building located in Makati Insurance Contract: Fire Insurance Policy for 600,000php Total Amount of Premium: 2,983.50php but Tibay only paid 600php thus leaving a balance. Building completely destroyed by fire and two days after, Tibay paid the unpaid premium and filed with Fortune a claim on fire insurance policy. No policy of insurance can ever pretend to be efficacious or effective until premium has been fully paid. Premium is the elixir vitae of insurance business, and all actuarial calculations and various tabulations of probabilities of losses under the risks insured against are based on the sound hypothesis of prompt payment of premium.

95. MALAYAN INSURANCE V. COURT OF APPEALS, 154 SCRA 672 (1987) MALAYAN INSURANCE CO., INC. v. ARNALDO and PINCA 154 SCRA 672 CRUZ; October 12, 1987 ! FACTS ! - On June 7, 1981, Malayan Insurance Co. (MICO), issued fire insurance for the amount of P14,000 on the property of private respondent, Pinca, effective July 1981-1982. MICO later allegedly cancelled the policy for non-payment of the premium and sent a notice to Pinca. On Dec. 24 Adora, an agent of MICO, received Pinca's payment, which was remitted to MICO. On Jan. 18, 1982, Pinca's property was completely burned. On Feb. 5, MICO returned Pinca's payment to Adora on the ground that her policy had been cancelled; the latter refused to accept it. Her demand for payment having been rejected by MICO, Pinca went to the Insurance Commission. Public respondent Arnaldo, the Insurance Commissioner, sustained Pinca, hence this petition from MICO. Records show MICO received Arnaldo's decision on Page 46 of 154

ISSUE: W/N fire insurance policy is valid, binding, and enforceable upon mere partial payment of premium? HELD: NO The consideration is the premium, which must be paid at the time and in the way and manner specified in the policy, if not paid, the policy will lapse and be forfeited by its own terms. Clearly the policy provides for payment in FULL The premium has only been partially paid and balance paid only after the peril insured against occurred thus contract did not take effect and insured cannot collect at all on the policy. But the controversy lies in the phrase unless and until the premium thereof has been paid.

INSURANCE G01 CASE DIGESTS April 10; MICO filed a MFR on April 25 which was denied on June 4; MICO received notice of this denial on June 14; instant petition was filed on July 2. of any court in all cases at 15 days from the notice of the decision appealed from. Since the MFR was filed only 15 days after receiving notice of the decision, it was already 18 days late by July 2. So whichever is applied, the petition is still late.

ISSUES Procedural 1. WON the petition should be dismissed for late filing Substantive 2. WON there was a valid insurance contract at the time of the loss 3. WON Adora was authorized to receive such payment 4. WON an adjuster is indispensable in the valuation of the loss Substantive 2. YES - A valid cancellation requires the following conditions based on Sections 64-65 of the Code: prior notice which must be based on the occurrence of one or more of the grounds mentioned in Sec 64 (in this case, non-payment of premium), after the effective date of the policy; the notice must be written and mailed to the address on the policy; it must state the ground(s) for cancellation and the insurer must furnish details upon the request of the insured. - It is undisputed that payment of premium was made. Petitioner relies heavily on Sec 77 of the Insurance Code to contest this, the said provision requiring payment of premium as soon as the thing is exposed to the peril insured against and that the policy is invalid without it. However, this is not applicable in the instant case as payment was eventually made. It is to be noted that the premium invoice was stamped "Payment Received#, indicating an understanding between the parties that payment could be made later. This is furthered by the fact that Adora had earlier told her to call him anytime she was ready with her payment. The Court also finds it strange that MICO only sought to return Pinca's Jan. 15 payment only on Feb. 5, long after her house had burned downthis makes petitioner's motives highly suspect. MICO claims to have sent a notice to Pinca, who flatly denied receiving one. Pinca did not have to prove this since the strict Page 47 of 154

HELD Procedural 1. YES - Petitioner invokes Sec 416 of the Insurance Code which grants it 30 days from notice of the Insurance Commission within which to appeal by certiorari with the Court. MICO filed its MFR on April 25, 15 days after the notice; the reglementary period began to run again after June 13. Since the petition was filed only on July 2, it was tardy by 4 days. Alternatively it invokes Rule 45 of the Rules of Court for certiorari but the petition still exceeds the 15 day limit from the June 13 notice. -Respondents, on the other hand, invoke Sec. 39 of B.P. 129 which pegs the period for appeal from decisions

INSURANCE G01 CASE DIGESTS language of Sec 64 requires that MICO ensure the cancellation was actually sent to and received by the insured. - MICO also suggests that Pinca knew the policy had been cancelled and was paying the premium in order to renew the policy. A close study of the transcripts show, however, that Pinca only meant to renew the policy had it been cancelled but not if it was still in effectit was conditional. Payment was thus legally made on the original transaction and validly received by Adora, who was not informed of the alleged cancellation and thus saw no reason to reject the payment. 3. YES - Sec. 306 of the Insurance Code provides that any insurance company that delivers a policy to its agent is deemed to have authorized such agent to receive payment of premium on its behalf. It is a well-known principle under the law of agency that payment to an authorized agent is equivalent to payment to the principal himself. MICO's acknowledgement of Adora as its agent thus defeats its contention that he was not authorized to receive payments on its behalf. 4. NO - In absence of fraud, the amount of the loss may be determined on the basis of such proof offered by the insured. Here, the certification of the Integrated National Police as the extent of the loss should suffice. Disposition petition is DENIED. 96. UCPB GENERAL INSURANCE V. MASAGANA TELEMART, 356 SCRA 307 (2001) Facts: On April 15, 1991, petitioner issued five (5) insurance policies covering respondent's various property described therein against fire, for the period from May 22, 1991 to May 22, 1992. In March 1992, petitioner evaluated the policies and decided not to renew them upon expiration of their terms on May 22, 1992. Petitioner advised respondent's broker, Zuellig Insurance Brokers, Inc. of its intention not to renew the policies. On April 6, 1992, petitioner gave written notice to respondent of the non-renewal of the policies at the address stated in the policies. On June 13, 1992, fire razed respondent's property covered by three of the insurance policies petitioner issued. On July 13, 1992, respondent presented to petitioner's cashier at its head office five (5) manager's checks in the total amount of P225,753.95, representing premium for the renewal of the policies from May 22, 1992 to May 22, 1993. No notice of loss was filed by respondent under the policies prior to July 14, 1992. On July 14, 1992, respondent filed with petitioner its formal claim for indemnification of the insured property razed by fire. On the same day, July 14, 1992, petitioner returned to respondent the five (5) manager's checks that it tendered, and at the same time rejected respondent's claim for the reasons: o that the policies had expired and were not renewed, and Page 48 of 154

INSURANCE G01 CASE DIGESTS that the fire occurred on June 13, 1992, before respondent's tender of premium payment. On July 21, 1992, respondent filed with the Regional Trial Court, Branch 58, Makati City, a civil complaint against petitioner for recovery of P18,645,000.00, representing the face value of the policies covering respondent's insured property razed by fire, and for attorney's fees. After due trial, on March 10, 1993, the Regional Trial Court, Branch 58, Makati, rendered decision in favor of the plaintiff and against the defendant. In due time, petitioner appealed to the Court of Appeals. The CA promulgated its decision affirming that of the RTC. Hence, this appeal. o premium for renewal of the policies was tendered on July 13, 1992, a month after the fire occurred on June 13, 1992. The assured did not even give the insurer a notice of loss within a reasonable time after occurrence of the fire.

97. AMERICAN HOME INSURANCE V. TANTUCO ENTERPRISES, 366 SCRA 740 (2001) PREMIUMS AMERICAN HOME ASSURANCE COMPANY V TANTUCO ENTERPRISES FACTS: 1. Respondent Tantuco Enterprises, Inc. is engaged in the coconut oil mining and refining industry. a. It owns 2 mills. b. Both located at its factory compound in Lucena City. 2. Respondent Tantuco commenced its business in ONLY ONE OIL MILL. In 1998, it started operating its second mill and it was referred to as the new oil mill. 3. The two mills were separately covered by fire insurance policies issued by Petitioner American Home Assurance Co., Philippine Branch. a. FIRST OIL MILL: insured for P.3,000,000.00 b. SECOND OIL MILL: insured for P.6,000,000.00

Issue: Whether the fire insurance policies issued by petitioner to the respondent covering the period May 22, 1991 to May 22, 1992, had expired on the latter date or had been extended or renewed by an implied credit arrangement though actual payment of premium was tendered on a later date after the occurrence of the risk (fire) insured against. HELD: NO An insurance policy, other than life, issued originally or on renewal, is not valid and binding until actual payment of the premium. Any agreement to the contrary is void. 11The parties may not agree expressly or impliedly on the extension of creditor time to pay the premium and consider the policy binding before actual payment. The case of Malayan Insurance Co., Inc. vs. Cruz-Arnaldo, 12 cited by the Court of Appeals, is not applicable. In that case, payment of the premium was in fact actually made on December 24, 1981, and the fire occurred on January 18, 1982. Here, the payment of the

Page 49 of 154

INSURANCE G01 CASE DIGESTS c. Official receIpts indicating PAYMENT FOR THE FULL AMOUNT OF THE PREMIUM were issued by the petitioner's agent. 4. Fire broke out and consumed the new oil mill. Respondent Tantuco immediately informed the petitioner of the incident. 5. PETITIONER REJECTED THE RESPONDEN'D CLAIM FOR THE INSURANCE PROCEEDS BASIS: a. that no policy was issued by it covering the burned oil mill. It stated that the description of the insured establishment referred to another building (the 2 policies extended the coverage to oil mill under Building No. 5 but the affected oil mill was under Building no. 14). b. The respondent is barred by estoppel from claiming that the description of the insured oil mill in the policy was wrong, because it retained the policy without having the same corrected before the fire. c. The petitioner included the Important Notice in the policy that states Please read and examine the policy and if incorrect, return it immediately for alteration. Whether or not Petitioner American Home Assurance is liable on the insurance policy? HELD: Yes. The Petitioner American Home Assurance is liable on the insurance policy. RATIONALE: The petition is lack of merit. In construing the words used descriptive of a building insured, the greatest liberality is shown by the courts in giving effect to the insurance. The courts are inclined to consider that the policy of insurance covers any building which the parties manifestly intended to insure, however inaccurate the description may be. Also, in determining what the parties intended, the courts will read and construe the policy as a whole and if possible, give effect to all the parts of the contract, that in case there is doubt, this doubt is to be resolved against the insurer. Object and purpose of the contract must be considered. PREMIUM ISSUE: PETITIONER'S CONTENTIONS: 6. RTC: Petitioner Insurance is liable on the insurance policy. 7. CA: Affirmed the decision of the RTC in toto. a. To further attempt to avoid the liability, petitioner claims that respondent forfeited the renewal policy for it failed to pay the full amount of the premium and breach of the Fire Extinguishing Appliances Warranty.

ISSUE:

Page 50 of 154

INSURANCE G01 CASE DIGESTS b. Respondent only paid for P75,147.00 instead of P89,770.20 which leaves the difference of 14,623.20. The petitioner argues that the deficiency suffices to invalidate the policy under Section 77 of the Insurance Code. c. Also, that even if Mr. Borja, who acknowledged the that the amount was lacking by reason of rebate is illegal. SC ruled that it fails to impress. It is true that there is a policy's condition for payment of the renewal premium on time and the respondent's non-compliance with it. Yet, it did not contain any specific and definite allegation that the respondent did not pay the premium, or that it did not pay the full amount, or that it sis not pay the amount on time. The issue of inadequacy of payment was never raised in the pre-trial proceedings and there is the petitioner failed to present any witness to testify that respondent indeed failed to pay the full amount of the premium. NO BREACH FOR NOT PROVIDING FIRE EXTINGUISHING APPLIANCES. 98. GREAT PACIFIC LIFE INSURANCE V. COURT OF APPEALS, 184 SCRA 501 (1990)

Facts: 1. This case involves an insured's claim for refund of the first premium on the endowment policy on his life, upon being notified by the insurer that the policy never took effect despite the premium payment. 2. Teodoro Cortez applied for a 20-year endowment policy for 30,000. 3. His application was accepted, approved and a policy was issued. 4. The policy was delivered to him by the underwriter Margarita Seiga. 5. The effective date indicated on the face of the policy in question was December 25, 1972. The annual premium was P1,416.60. Mrs. Siega assured him that the first premium may be paid within the grace period of thirty (30) days from date of delivery of the policy 6. The first premium was then paid in 3 installments. 7. In a letter dated June 1, 1973 (Exh. E), defendant advised plaintiff that Policy No. 221944 (Exh. A) was not in force. To make it enforceable and operative, plaintiff was asked to remit the balance of P1,015.60 to complete his initial annual premium due December 15, 1972, and to see Dr. Felipe V. Remollo for another full medical examination at his own expense. 8. Cortez' reaction to the company's act was to immediately inform it that he was cancelling the policy and he demanded the return of his premium plus damages. 9. RTC Ruled in favour of Cortez 10. CA Affirmed Issue: Page 51 of 154

INSURANCE G01 CASE DIGESTS 1. Whether or not Cortez is entitled to a refund of his premium. Held: 1. YES 2. When the petitioner advised private respondent on June 1, 1973, four months after he had paid the first premium, that his policy had never been in force, and that he must pay another premium and undergo another medical examination to make the policy effective, the petitioner committed a serious breach of the contract of insurance. 3. Petitioner should have informed Cortez of the deadline for paying the first premium before or at least upon delivery of the policy to him, so he could have complied with what was needful and would not have been misled into believing that his life and his family were protected by the policy, when actually they were not. 4. And, if the premium paid by Cortez was unacceptable for being late, it was the company's duty to return it. 5. By accepting his premiums without giving him the corresponding protection, the company acted in bad faith. 6. Sections 79, 81 and 82 of P.D. 612 of the Insurance Code of 1978 provide when the insured is entitled to the return of premium paid. 7. Since his policy was in fact inoperative or ineffectual from the beginning, the company was never at risk, hence, it is not entitled to keep the premium. 99. STOKES V. MALAYAN, 127 SCRA 766 (1984) FACTS: 1. Daniel Adolfson had a subsisting Malayan car insurance policy with coverage against own damage as well as 3rd party liability when his car figured in a vehicular accident with another car, resulting to damage to both vehicles. 2. At the time of the accident, Adolfsons car was being driven by James Stokes, who was authorized to do so by Adolfson. Stokes, an Irish tourist who had been in the Philippines for only 90 days, had a valid and subsisting Irish drivers license but without a Philippine drivers license. 3. Adolfson filed a claim with Malayan but the latter refused to pay contending that Stokes was not an authorized driver under the Authorized Driver clause of the insurance policy in relation to Section 21 of the Land Transportation Office. ISSUE: Whether or not Malayan is liable to pay the insurance claim of Adolfson HELD: NO. A contract of insurance is a contract of indemnity upon the terms and conditions specified therein. When the insurer is called upon to pay in case of loss or damage, he has the right to insist upon compliance with the terms of the contract. If the insured cannot bring himself within the terms and conditions of the contract, he is not entitled as a rule to recover for the loss or damage suffered. For the terms of the contract constitute the measure of the insurers liability, and compliance therewith is a condition precedent to the right of recovery. At the time of the accident, Stokes had been in the Philippines for more than 90 days. Hence, under the law, he could not drive a motor vehicle without a Philippine drivers license. He was therefore not an authorized driver under the terms of the insurance policy in question, and Malayan was right in denying the claim of the insured. Page 52 of 154

INSURANCE G01 CASE DIGESTS 100. CAPITAL INSURANCE V. PLASTIC ERA, 65 SCRA 134 (1975) Acceptance of premium within the stipulated period for payment thereof, including the agreed period of grace, merely assures continued effectivity of the insurance policy in accordance with its terms. Such acceptance does not estop the insurer from interposing any valid defense under the terms of the insurance policy. The principle of estoppel is an equitable principle rooted upon natural justice which prevents a person from going back on his own acts and representations to the prejudice of another whom he has led to rely upon them. The principle does not apply to the instant case. In accepting the premium payment of the insured, Malayan was not guilty of any inequitable act or representation. There is nothing inconsistent between acceptance of premium due under an insurance policy and the enforcement of its terms. GR No. L-22375 July 18, 1975 Facts: Capital Insurance delivered to Plastic Era its open fire policy. Capital Insurance undertook to insure Plastic Era's building, equipments, raw materials, products and accessories located in Mandaluyong, Rizal. The policy expressly provided that if the property insured would be destroyed or damaged by fire after the payment of the premiums, at any time between Dec. 15, 1960 and 1PM of Dec. 15, 1961, the insurance company shall make good all such loss or damage in an amount not exceeding 100K. Plastic Era failed to pay the corresponding insurance premium. However, its duly authorized representative executed an acknowledgment receipt with a corresponding 'promise to pay.' Plastic Era delivered to Capital Insurance a check for the amount of 1K, as partial payment of the premium, postdated Jan 16, 1961 and drawn against the Bank of America. When Capital Insurance tried to deposit the check a month after the date, it was dishonored for lack of funds. January 18, 1961, 2 days after the premium was due, the property insured by Plastic Era was destroyed by fire. They notified Capital of the loss and filed its claim for indemnity through the Manila Adjustment Company, who estimated the loss at P283,875. The same property was insured with Philamgen for 200K. Plastic Era demanded from Capital the payment of the sum of 100K as indemnity, but was denied because they had failed to pay the premium. Issue: Page 53 of 154

INSURANCE G01 CASE DIGESTS Whether there was a perfected contract of insurance between Capital and Plastic Era Whether there was payment of premium (either through the acknowledgment receipt with a promise to pay, or through the postdated check) Ruling: Capital Insurance was held to be liable to Plastic Era It is clear from the terms of the policy that it is only upon payment of the premiums that Capital agrees to insure the properties of Plastic Era against loss or damage in an amount not exceeding P100K. The only question/issue is whether there was payment. The issuance of the postdated check did not produce the effect of payment. o The mere delivery of a bill of exchange in payment of a debt does not immediately effect payment. It simply suspends the action arising from the original obligation in satisfaction of which it was delivered, until payment is accomplished either actually or presumptively. It shall only produce the effect of payment when they have been encashed, or when through the fault of the creditor they have been impaired. However, in Capital's acceptance of the promissory note, it implicitly agreed to modify the tenor of the insurance policy and in effect, waived the provision that it would only pay for the loss or damage in case the same occurs after the payment of the premium. The insurance policy is silent as to the mode of payment. Therefore, Capital Insurance is deemed to have accepted the promissory note in payment of the premium. This acceptance of the note rendered the policy immediately operative on the date it was delivered. When the damage or loss of the insured property occurred, the insurance policy was in full force and effect. The fact that the check issued by Plastic Era in partial payment of the promissory note was later on dishonored did not in any way operate as a forfeiture of its rights under the policy, there being no express stipulation therein to that effect. Moreover, by accepting its promise to pay the insurance premium within 30 days from the effectivity date of the policy, Capital Insurance had in effect extended credit to Plastic Era. The payment of the premium on the insurance policy therefore became an independent obligation the non-fulfillment of which would entitle Capital Insurance to recover. It did not have the right to cancel the policy for nonpayment of the premium except by putting Plastic Era in default and giving it personal notice to that effect. Capital is likewise estopped from claiming a forfeiture of its policy for non-payment even if the check was dishonored. Although the check was due for payment on Jan 16, and Plastic Era had sufficient funds during that date, Capital decided to hold the check of 35 days before presenting it for payment.

Page 54 of 154

INSURANCE G01 CASE DIGESTS

LOSS
101. CCC INSURANCE V. COURT OF APPEALS, 31 SCRA 264

AUTHORIZED DRIVER: Any of the following: (a) The insured; (b) Any person driving on the Insured's order or with his permission, provided that the person driving is permitted in accordance with licensing laws or regulations to drive the motor vehiclecovered by this Policy, or has been so permitted and is not disqualified by order of a court of law or by reason of any enactment or regulation from driving such Motor Vehicle. (Emphasis ours) It has been found as a fact by the Court of Appeals that Domingo Reyes, the, driver who was at the wheel of the insured car at the time of the accident, does not know how to read and write; that he was able to secure a driver's license, without passing any examination therefor, by paying P25.00 to a certain woman 4. The appellant insurer insists that, under the established facts of this case, Reyes, being admittedly one who cannot read and write, who has never passed any examination for drivers, and has not applied for a license from the duly constituted government agency entrusted with the duty of licensing drivers, cannot be considered an authorized driver. 5. Under Section 24 of the Revised Motor Vehicles Law, Act 3992 of the Philippine Legislature, as amended by Republic Acts Nos. 587, 1204 and 2863, the issuance of a driving license without previous examination does not necessarily imply that the license issued is invalid.
3.

Facts: 1. Carlos F. Robes took an insurance, with the CCC Insurance Corporation, on his Dodge Kingsway car against loss or damage through accident for an amount not exceeding P8,000.00 2. The insured vehicle, while being driven by the owner's driver, became involved in a vehicular collision along Rizal Avenue Extension, Potrero, Malabon, Rizal. The car was damaged, and the repair was estimated to cost P5,300.00. 3. The insurance company refused to pay for the repair or restore the car. 4. The insurance company disclaimed liability for payment, alleging that there had been violation of the insurance contract because the one driving the car at the time of the incident was not an "authorized driver." 5. RTC Ruled for plaintiff and ordered defendant insurer to pay. 6. The CA affirmed the RTC Issue:
1.

Whether or not the damage to the insured car was not covered by the insurance policy because at the time of the accident it was being driven by one who was not an authorized driver.

Held: The damage is under the insurance policy. The driver falls under the definition of an authorized driver 2. Under the law
1.

Page 55 of 154

INSURANCE G01 CASE DIGESTS This insurance does not cover any loss or damage occasioned by or through or in consequence, directly or indirectly, of any of the following occurrences, namely: xxx xxx (d) Mutiny, riot, military or popular uprising, insurrection, rebellion, revolution, military or usurped power.

102. COUNTRY BANKERS INSURANCE V. LLANGA BAY AND COMMUNITY MULTI-PURPOSE COOPERATIVE , 374 SCRA 653 (2002)
COUNTRY BANKERS INSURANCE CORPORATION, petitioner, vs. LIANGA BAY AND COMMUNITY MULTI-PURPOSE COOPERATIVE, INC., respondent. DECISION

DE LEON, JR., J.: FACTS: ~Petitioner Country Bankers is a domestic corporation principally engaged in the insurance business. ~Respondent Lianga Coop is a duly registered cooperative judicially declared insolvent. ~Petitioner and the respondent entered into a 1-year fire insurance contract over respondents stocks-in-trade (inside their building at Lianga Bay, Surigao del Sur) for the sum of Two Hundred Thousand Pesos (P200,000.00). ~Respondents building was gutted by fire and reduced to ashes. ~Respondent filed an insurance claim submitting: (a) the Spot Report of Pfc. Arturo V. Juarbal, INP Investigator; (b) the Sworn Statement of Jose Lomocso; and (c) the Sworn Statement of Ernesto Urbiztondo. ~The petitioner denied the insurance claim on the ground that, based on the submitted documents, the building was set on fire by two (2) NPA rebels who wanted to obtain canned goods, rice and medicines as provisions for their comrades in the forest, and that such loss was an excepted risk under paragraph No. 6 of the policy, which states:

~Respondent filed in the trial court the complaint for recovery of loss, damage or liability against petitioner. RTC --- Trial court decided in favor of the respondent ordering petitioner to pay respondent P200,000.00, with interest at twelve percent (12%) per annum from the date of filing of the complaint until fully paid, as well as Fifty Thousand Pesos (P50,000.00) as actual damages, Fifty Thousand Pesos (P50,000.00) as exemplary damages, Five Thousand Pesos (P5,000.00) as litigation expenses, Ten Thousand Pesos (P10,000.00) as attorneys fees, and the costs of suit. CA --- affirmed

ISSUE: Whether petitioner Country Bankers is liable to pay the claim plus interest, damages, attorneys fees and costs of suit.

HELD: NO, Country Bankers is liable only for the insurance claim but not for 12% interest, damages and fees. Since petitioners defense is non-coverage by reason of the exemption or exception clause in the fire insurance policy, it has the Page 56 of 154

INSURANCE G01 CASE DIGESTS burden of proving the facts upon which such excepted risk is based, by a preponderance of evidence.[7] But petitioner failed to do so. The petitioner relied on the Sworn Statements of Lomocso and Urbiztondo and the Spot Report of Pfc. Arturo V. Juarbal. But the Sworn Statements were found to be inadmissible in evidence, for being hearsay, inasmuch as they did not take the witness stand and could not therefore be cross-examined. As to Investigator Juarbals Spot Report, it was based on the personal knowledge of the caretaker Lomocso who witnessed every single incident surrounding the facts and circumstances of the case. And so said Spot Report was also inadmissible for being hearsay as it was not based on the investigators personal knowledge although he took the witness stand. The petitioners evidence to prove its defense is sadly wanting and thus, it is liable to the respondent under the Fire Insurance Policy. But we do not sustain the trial courts imposition of 12% interest on the insurance claim as well as the monetary award for actual and exemplary damages, litigation expenses and attorneys fees for lack of legal and valid basis. Concerning the application of the proper interest rates, the following guidelines were set in Eastern Shipping Lines, Inc. v. Court of Appeals and Mercantile Insurance Co., Inc.:[15] I. When an obligation, regardless of its source, i.e., law, contracts, quasi-contracts, delicts or quasidelicts, is breached, the contravenor can be held liable for damages. The provisions under Title XVIII on Damages of the Civil Code govern in determining the measure of recoverable damages. II. With regard particularly to an award of interest in the concept of actual and compensatory damages, the rate of interest, as well as the accrual thereof, is imposed, as follows: 1. When the obligation is breached, and it consists in the payment of a sum of money, i.e., a loan or forbearance of money, the interest due should be that which may have been stipulated in writing. Furthermore, the interest due shall itself earn legal interest from the time it is judicially demanded. In the absence of stipulation, the rate of interest shall be 12% per annum to be computed from default, i.e., from judicial or extrajudicial demand under and subject to the provisions of Article 1169 of the Civil Code. 2. When an obligation, not constituting a loan or forbearance of money, is breached, an interest on the amount of damages awarded may be imposed at the discretion of the court at the rate of 6% per annum. No interest, however, shall be adjudged on unliquidated claims or damages except when or until the demand can be established with reasonable certainty. Accordingly, where the demand is established with reasonable certainty, the interest shall begin to run from the time the claim is made judicially or extrajudicially (Art. 1169, Civil Code) but when such certainty cannot be so reasonably established at the time the demand is made, the interest shall begin to run only from the date the judgment of the court is made (at which time the quantification of damages may be deemed to have been reasonably ascertained). The actual base for the computation of legal interest shall, in any case, be on the amount finally adjudged. 3. When the judgment of the court awarding a sum of money becomes final and executory, the rate of legal interest, whether the case falls under paragraph 1 or paragraph 2, above, shall be 12% per annum from such finality until its satisfaction, this Page 57 of 154

INSURANCE G01 CASE DIGESTS interim period being deemed to be by then an equivalent to a forbearance of credit. damages, Five Thousand Pesos (P5,000.00) as litigation expenses, and Ten Thousand Pesos (P10,000.00) as attorneys fees are hereby DELETED. Costs against the petitioner. SO ORDERED.

The insurance claim in this case is evidently not a forbearance of money, goods or credit, and thus the interest rate should be 6% from the date of filing of the complaint. We find no justification for the award of actual damages. Well-entrenched is the doctrine that actual, compensatory and consequential damages must be proved, and cannot be presumed. The justification, if any, for such an award of actual damages does not appear in the body of the decision of the trial court. We likewise find no legal and valid basis for granting exemplary damages. Article 2229 of the New Civil Code provides that exemplary damages may be imposed by way of example or correction for the public good. However, it cannot be recovered as a matter of right. It is based entirely on the discretion of the court. And we find no justification for the award of litigation expenses and attorneys fees. Article 2208 of the New Civil Code enumerates the instances where such may be awarded and, in all cases, it must be reasonable, just and equitable if the same were to be granted. The award of attorneys fees is the exception rather than the general rule. As such, it is necessary for the court to make findings of facts and law that would bring the case within the exception and justify the grant of such award. We find none in this case. WHEREFORE, the appealed Decision is MODIFIED. The rate of interest on the adjudged principal amount of Two Hundred Thousand Pesos (P200,000.00) shall be six percent (6%) per annum computed from the date of filing of the Complaint in the trial court. The awards in the amounts of Fifty Thousand Pesos (P50,000.00) as actual damages, Fifty Thousand Pesos (P50,000.00) as exemplary

103. PARIS-MANILA PERFUMERY V. PHOENIX ASSURANCE , 49 PHIL. 753 (1926)

Paris-Manila Perfume Co. v. Phoenix Assurance Co. G.R. No. L-25845 December 17, 1926 Lessons Applicable: Loss, the immediate cause of which was the peril insured against, if the proximate cause thereof was NOT excepted in the contract (Insurance) Facts:

May 22, 1924: A fire insurance policy was issued by Phoenix Assurance Company, Limited to Messrs. Paris-Manila Perfumery Co. (Peter Johnson, Prop.) for P13,000 o also insured with other insurance companies for P1,200 and P5,000 respectively July 4, 1924: The Perfumery was burned unknown of the cause totalling a loss of P38.025.56 Phoenix refused to pay nor to appoint an arbitrator stating that the policy did not cover any loss or damage occasioned by explosion and stating that the claim was fraudulent RTC: ordered Phoenix to pay P13,000 Phoenix appealed o The insurance policy contains: Page 58 of 154

INSURANCE G01 CASE DIGESTS Unless otherwise expressly stated in the policy the insurance does not cover (h) Loss or damage occasioned by the explosion; but loss or damage by explosion of gas for illuminating or domestic purposes in a building in which gas is not generated and which does not form a part of any gas works, will be deemed to be loss by fire within the meaning of this policy. ISSUE: W/N Phoenix should be liable for the loss because there was no explosion which is an exemption from the policy HELD: YES.
On Sept. 23, 1979, San Miguel Corporation shipped from

104.

FGU INSURANCE V. COURT OF APPEALS, 454 SCRA 337 (2005)

FGU Insurance vs. Court of Appeals G.R. No. 137775; March 31, 2005

Facts:
Anco Enterprises, a partnership between Ang Gui and Co To, was

engaged in the shipping business. It owned the M/T ANCO tugboat and the D/B Lucio barge. Since the D/B Lucio had no engine of its own, it could not maneuver by itself and had to be towed by a tugboat for it to move from one place to another.

If it be a fact that the fire resulted from an explosion that fact, if proven, would be a complete defense, the burden of the proof of that fact is upon the defendant, and upon that point, there is a failure of proof lower court found as a fact that there was no fraud in the insurance, and that the value of the property destroyed by the fire was more than the amount of the insurance.

Mandaue City, Cebu on board the D/B Lucio, for towage by M/T ANCO, the following cargoes:

25,000 cases Pale Pilsen and 350 cases Cerveza Negra - consignee SMCs Beer Marketing Division (BMD)Estancia Beer Sales Office, Estancia, Iloilo 15,000 cases Pale Pilsen and 200 cases Cerveza Negra - consignee SMCs BMD-San Jose Beer Sales Office, San Jose, Antique
The D/B Lucio was towed by the M/T ANCO all the way from

Mandaue City, Cebu to San Jose, Antique. It arrived at San Jose at around 1:00 PM of September 30, 1979. Page 59 of 154

INSURANCE G01 CASE DIGESTS ANCO, as a partnership, was dissolved hence, on January 26, 1993, SMC filed a second amended complaint which was admitted by the Court impleading the surviving partner, Co To and the Estate of Ang Gui represented by Lucio, Julian and Jaime, all surnamed Ang.
ANCO admitted that the cases of beer Pale Pilsen and Cerveza

In the afternoon of the same day, the clouds over the area were

dark and the waves were already big. The arrastre workers unloading the cargoes of SMC on board the D/B Lucio began to complain about their difficulty in unloading the cargoes. SMCs District Sales Supervisor, Fernando Macabuag, requested ANCOs representative to transfer the barge to a safer place because the vessel might not be able to withstand the big waves.
ANCOs representative did not heed the request because he was

confident that the barge could withstand the waves. At that time, only the M/T ANCO was left at the wharf of San Jose, Antique, as all other vessels already left the wharf to seek shelter. With the waves growing bigger and bigger, only 10,790 cases of beer were discharged into the custody of the arrastre operator.
At about 10:00 to 11:00 pm of October 1, 1979, the crew of D/B

Lucio abandoned the vessel because the barges rope attached to the wharf was cut off by the big waves. At around midnight, the barge run aground and was broken and the cargoes of beer in the barge were swept away.
As a result, ANCO failed to deliver to SMCs consignee 29,210

Negra mentioned in the complaint were indeed loaded on the vessel belonging to ANCO. It claimed however that it had an agreement with SMC that ANCO would not be liable for any losses or damages resulting to the cargoes by reason of fortuitous event. Since the cases of beer Pale Pilsen and Cerveza Negra were lost by reason of a storm, a fortuitous event which battered and sunk the vessel in which they were loaded, they should not be held liable. ANCO further asserted that there was an agreement between them and SMC to insure the cargoes in order to recover indemnity in case of loss. Pursuant to that agreement, the cargoes to the extent of 20,000 cases was insured with FGU Insurance Corporation (FGU) for the total amount of P858,500.00 per Marine Insurance Policy No. 29591.
In its answer to the Third-Party complaint, FGU admitted the

cases of Pale Pilsen and 550 cases of Cerveza Negra. The value per case of Pale Pilsen was P42.50. The value of a case of Cerveza Negra was P47.10, hence, SMCs claim against ANCO amounted P1,346,197.00.
SMC filed a complaint for breach of contract of carriage against

existence of the Insurance Policy but maintained that the alleged loss of the cargoes covered by the said insurance policy cannot be attributed directly or indirectly to any of the risks insured against in the said insurance policy. According to FGU, it is only liable under the policy to Third-party Plaintiff ANCO and/or Plaintiff SMC in case of any of the following:

ANCO for the amount of P1,346,197. Upon Ang Guis death,

a) Total loss of the entire shipment; b) Loss of any case as a result of the sinking of the vessel; or c) Loss as a result of the vessel being on fire. Page 60 of 154

INSURANCE G01 CASE DIGESTS thus, making said blatant negligence the proximate cause of the loss of the cargoes. One of the purposes for taking out insurance is to protect the insured against the consequences of his own negligence and that of his agents. Thus, it is a basic rule in insurance that the carelessness and negligence of the insured or his agents constitute no defense on the part of the insurer. This rule however presupposes that the loss has occurred due to causes which could not have been prevented by the insured, despite the exercise of due diligence. The question now is whether there is a certain degree of negligence on the part of the insured or his agents that will deprive him the right to recover under the insurance contract. We say there is. However, to what extent such negligence must go in order to exonerate the insurer from liability must be evaluated in light of the circumstances surrounding each case. When evidence show that the insureds negligence or recklessness is so gross as to be sufficient to constitute a willful act, the insurer must be exonerated. The United States Supreme Court has made a distinction between ordinary negligence and gross negligence or negligence amounting to misconduct and its effect on the insureds right to recover under the insurance contract. According to the Court, while mistake and negligence of the master or crew are incident to navigation and constitute a part of the perils that the insurer is obliged to incur, such negligence or recklessness must not be of such gross character as to amount to misconduct or wrongful acts; otherwise, such negligence shall release the insurer from liability under the insurance contract.

Furthermore, FGU alleged that the Third-Party Plaintiff ANCO

and Plaintiff SMC failed to exercise ordinary diligence or the diligence of a good father of the family in the care and supervision of the cargoes insured to prevent its loss and/or destruction.
The RTC held that while the cargoes were indeed lost due to

fortuitous event, there was failure on ANCOs part, through their representatives, to observe the degree of diligence required that would exonerate them from liability. The trial court thus held the Estate of Ang Gui and Co To liable to SMC for the amount of the lost shipment.
With respect to the Third-Party complaint, the RTC found FGU

liable to 53% of the amount of the lost cargoes. It was upheld by the Court of Appeals. Issue: Whether or not FGU can be held liable under the insurance policy to reimburse ANCO for the loss of the cargoes despite the findings of the Court of Appeals that such loss was occasioned by the blatant negligence of the latters employees. Held: NO. The third-party complaint against FGU insurance is dismissed but ANCO is still liable to SMC. Ratio: To be exempt from responsibility, the natural disaster should have been the proximate and only cause of the loss. But, according to the Court of Appeals, there was blatant negligence on the part of M/T ANCOs crewmembers, first in leaving the engine-less barge D/B Lucio at the mercy of the storm without the assistance of the tugboat, and again in failing to heed the request of SMCs representatives to have the barge transferred to a safer place, as was done by the other vessels in the port;

Page 61 of 154

INSURANCE G01 CASE DIGESTS In the case at bar, both the trial court and the appellate court had concluded from the evidence that the crewmembers of both the D/B Lucio and the M/T ANCO were blatantly negligent. To wit: There was blatant negligence on the part of the employees of defendants-appellants when the patron (operator) of the tug boat immediately left the barge at the San Jose, Antique wharf despite the looming bad weather. Negligence was likewise exhibited by the defendants-appellants representative who did not heed Macabuags request that the barge be moved to a more secure place. The prudent thing to do, as was done by the other sea vessels at San Jose, Antique during the time in question, was to transfer the vessel to a safer wharf. The negligence of the defendants-appellants is proved by the fact that on 01 October 1979, the only simple vessel left at the wharf in San Jose was the D/B Lucio.

Page 62 of 154

INSURANCE G01 CASE DIGESTS

NOTICE AND PROOF OF LOSS


105. MALAYAN INSURANCE VS. CRUZ-ARNALDO, 154 SCRA 672 (1987) MALAYAN INSURANCE CO., INC. (MICO), petitioner, vs. GREGORIA CRUZ ARNALDO, in her capacity as the INSURANCE COMMISSIONER, and CORONACION PINCA, respondents. G.R. No. L-67835 October 12, 1987 Facts: 1. Jun 7 1981: MICO issued to Coronacion Pinca a Fire Insurance Policy for her property effective July 22, 1981 to July 22, 1982 2. Oct 151981: MICO allegedly cancelled the policy for nonpayment, of the premium and sent the corresponding notice to Pinca 3. Dec 24 1981: payment of the premium for Pinca was received by Domingo Adora, as agent of MICO 4. Jan 15 1982: Adora remitted this payment to MICO, together with other payments 5. Jan 18 1982: Pinca's property was completely burned 6. Feb 5 1982: Pinca's payment was returned by MICO to Adora on the ground that her policy had been cancelled earlier but Adora refused to accept it and instead demanded for payment 7. MICO filed its MR on April 25, 1981 (or 15 days after such notice) a. The reglementary period began to run again after Jun 13, 1981 (date of its receipt of notice of the denial of the said MR) b. Under Section 416 of the Insurance Code, the period for appeal is thirty days from notice of the decision of the Insurance Commission.

c. As the herein petition was filed on July 2, 1981, or 19 days later, there is no question that it is tardy by four days. 8. Insurance Commission ruling: in favor of Pinca 9. Hence this appeal a. MICO contends that it should not be held liable because: i. Adora was not authorized to receive premium payment because 6 months had elapsed since the issuance by the policy itself... this prohibition was binding upon Pinca, who made the payment to Adora at her own risk as she was bound to first check his authority to receive it ii. No payment of premium and that the policy had been cancelled before the occurrence of the loss iii. The claim was allowed without sufficient proof of loss

ISSUE: (We will now focus on the final issue raised by MICO, the proper valuation of the loss incurred.) Whether the valuation fixed based on the certification issued by the Integrated National Police is sufficient HELD: Yes, petition DENIED. RATIO:
1.

The last point raised by the petitioner should not pose much difficulty. The valuation fixed in fire insurance policy is conclusive in case of total loss in the absence of fraud, which is not shown here. Page 63 of 154

INSURANCE G01 CASE DIGESTS a. Loss and its amount may be determined on the basis of such proof as may be offered by the insured, which need not be of such persuasiveness as is required in judicial proceedings. b. If, as in this case, the insured files notice and preliminary proof of loss and the insurer fails to specify to the former all the defects thereof and without unnecessary delay, all objections to notice and proof of loss are deemed waived under Section 90 of the Insurance Code. The certification 26 issued by the Integrated National Police, Lao-ang, Samar, as to the extent of Pinca's loss should be considered sufficient. Notably,MICO submitted no evidence to the contrary nor did it even question the extent of the loss in its answer before the Insurance Commission. It is also worth observing that Pinca's property was not the only building bumed in the fire that razed the commercial district of Lao-ang, Samar, on January 18, 1982. There is nothing in the Insurance Code that makes the participation of an adjuster in the assessment of the loss imperative or indespensable, as MICO suggests. Section 325, which it cites, simply speaks of the licensing and duties of adjusters. We see in this cases an obvious design to evade or at least delay the discharge of a just obligation through efforts bordering on bad faith if not plain duplicity, We note that the motion for reconsideration was filed on the fifteenth day from notice of the decision of the Insurance Commission and that there was a feeble attempt to show that the notice of denial of the said motion was not received on June 13, 1982, to further hinder the proceedings and justify the filing of the petition with this Court fourteen days after June 18, 1982. We also look askance at the alleged cancellation, of which the insured and MICO's agent himself had no knowledge, and the curious fact that although Pinca's payment was remitted to MICO's by its agent on January 15, 1982, MICO sought to return it to Adora only on February 5, 1982, after it presumably had learned of the occurrence of the loss insured against on January 18, 1982. These circumstances make the motives of the petitioner highly suspect, to say the least, and cast serious doubts upon its candor and bona fides.

2.

3.

4.

Page 64 of 154

INSURANCE G01 CASE DIGESTS 106. YU BAN CHUAN V. FIELDMENS INSURANCE , 14 SCRA 491 (1965) Yu Ban Chuan vs. Fieldmen's Insurance Co. Inc. G.R. No. L-19851. June 29, 1965 Facts: On March, 1959, plaintiff Yu Ban Chuan began his business enterprise under the name of "CMC Trading," which was engaged in the wholesale dealing of general merchandise and school supplies. The plaintiff insured against fire the stock merchandise with defendant Fieldmen's Insurance Co. Inc. on December 14, 1959, with an "open" policy limiting the insurer's liability to the amount of P200,000 for a period of one year. The plaintiff again insured against fire the same stock of merchandise with another company which was Paramount Surety & Insurance Co., on January 7, 1960, with another "open" policy limiting liability thereunder to P140,000 for also a period of one year. Plaintiff transferred his business establishment. Both Fieldmen's and Paramount acknowledged the existence of its co-insurance and both agreed to transfer the coverage of its respective insurance policies to the plaintiff's new store. On January 31, 1960, while both insurance policies were in full force and effect, plaintiff's business establishment was totally destroyed by fire. The next day after the occurrence of the fire, plaintiff verbally notified the respective agents of the defendants- insurers of such incident; and on the same day, plaintiff and H. H. Bayne Adjustment Co. and Manila Adjustment Co., adjusters of defendants Fieldmen's and Paramount, respectively, executed "non-waiver" agreements for the purpose of determining the circumstances of the fire and the value or amount of loss and damage to the merchandise insured under said policies. Pursuant to such agreements, H. H. Bayne Adjustment Co. and Manila Adjustment Co. sent letters requiring the plaintiff to submit certain papers and documents. On February 8, 1960, plaintiff gave a written notice of the occurrence of the fire to the defendants, and, in answer to the letters of the adjusters, plaintiff submitted his separate formal fire claims, together with some of the supporting papers that where required. Because of plaintiff's non-compliance or failure to submit the required documents the defendants rejected plaintiff's claims, and denied liability under their respective policies, evidently upon their respective adjusters' recommendations. The plaintiff commenced suit in the Court of First Instance of Manila. The defendants answered the complaint with identical special defenses; to wit: 1) insured's failure to prove the loss claimed; 2) false and fraudulent claim; and 3) arson or causes not independent of the will of the insured; After trial, the CFI upheld the claim of the plaintiff, but refused to award damages or interest at more than the legal rate. Both parties appealed. In proving the value of his loss, the plaintiff relied upon a merchandise inventory as of December 31, 1959, which he had Page 65 of 154

INSURANCE G01 CASE DIGESTS allegedly submitted on January 15, 1960 to the Bureau of Internal Revenue. The inventory reflected the total value of stocks at P328,202.67. The plaintiff also claimed purchases for the month of January 1960 in the amount of P34,505.08 and sales in the amount of P12,000, thus the resulting balance of the stocks allegedly burned was estimated by the plaintiff to be P350,707.75. Issue: Whether or not plaintiff's appeal for the award of damages at more than the legal rate must be allowed? No. Ratio: The fact of the filing of the inventory as of January 15, 1960 should be considered as true, since there was no evidence to the contrary that would necessarily show the falsity of the inventory as of the date stamped therein as certified by the chief of the administrative division. The court a quo, however, committed error in accepting as true the actual existence at the burned premises of the stocks mentioned in the inventory. Six of the many copies of the invoices submitted by the plaintiff to the adjusters uncover a clear case of fraud and misrepresentation as to the liability of Fieldmen's and Paramount's respective policies. Five of the invoices alone inflate the supposed stocks by P248,370.00. On sight, the exhibit excites incredulity. The plaintiff, Yu Ban Chuan, adopted a uniform, too uniform, in fact, to be believed, explanation for all the invoices, that he did not buy the merchandise at the companies' addresses but bought from the agents who brought the goods to him and that the originals of the invoices were burned and that he requested for true copies from the agents whom he met casually in the streets after the fire and these agents delivered the exhibits to him but he did not remember or know the names of these agents, nor did he know their whereabouts. In other words, he wants the court to believe also that these agents performed a vanishing act after each one of them had turned in the copy of each invoice to the plaintiff. The plaintiff adhered to the inventory as the immaculate basis for the actual worth of stocks that were burned, on the ground that it was made from actual count, and in compliance with law. But this inventory is not binding on the defendants, since it was prepared without their intervention. It is well to note that plaintiff had every reason to show that the value of his stock of goods exceeded the amount of insurance that he carried. The inventory having been made prior to the fire, was no proof of the existence of these goods at the store when the fire occurred. True, there were merchandise that were actually destroyed by fire, but when fraud is conceived, what is true is subtly hidden by the schemer beneath proper and legal appearances, including the preparation of the inventory. Shielding himself under Section 82 of the Insurance Act, the plaintiff asserts that in submitting his proof of loss he was "not bound to give such proof as would be necessary in a court of justice". The assertion is correct, but does not give him any justification for Page 66 of 154

INSURANCE G01 CASE DIGESTS submitting false proofs. Their falsity is the best evidence of the fraudulent character and the unmeritoriousness of plaintiff's claim. For the foregoing reasons, the appealed judgment was hereby reversed, and the plaintiff's appeal against the non-award of damages to him must be dismissed.

HELD (16 cases only and not 66cases) The fire was seen by a large number of people, and if it be a fact that the plaintiff had 66 cases of piece goods in the building at the time, it was his duty to have offered the evidence of some disinterested eyewitness as to the identity of the pieces or particles remaining of the 50 cases, and of the physical facts, for the purpose of showing that the 50 cases were in the bodega at the time of the fire. Although the original entries in plaintiff's books would be evidence which should have some weight as to the amount of stock which he had in March, and which he purchased during the months of April, May, and June, and what he sold during that time, such entries are of but little, if any, value as to the amount of goods which he had in the bodega at the time of the fire. In any event, they are not sufficient to overcome the absence of any evidence of the physical facts existing after the fire, and the rule of reason that the 50 cases of goods would not be consumed and completely wiped out of existence, without leaving some evidence of their destruction, which could be found among the remains and debris in the building after the fire. Here, the facts existing at and after the fire are conclusive evidence that there were only 16 cases of goods in thebodega at the time of the fire, and the majority of this court are of the opinion that plaintiff's claim is not only fraudulent, but that he knew it was fraudulent at the time it was made, and that, for such reason, he is not entitled to recover anything. Page 67 of 154

107.

GO LU V. YORKSHIRE INSURANCE , 43 PHIL. 633 (1922) GO LU V. YORKSHIRE (Proof of Loss) FACTS

1. Go Lu was a merchant buying and selling Bolt goods which he stored in a bodega in Manila. 2. Yorkshire issued insurance for the goods stored in the bodega 3. Unfortunately, the bodega was razed by a fire 4. At the time of the fire, Go Lu claims that there were 66 cases of bolt goods stored in the bodega 5. Yorkshire admit the issuance of the policies but contends that not more than 16 cases were destroyed. It alleges that Go Lu submitted fraudulent proof of the amount of loss. ISSUE What is the amount and value of the amount of goods which Go Lu had in the bodega at the time of the fire?

INSURANCE G01 CASE DIGESTS

108. FINMAN S GENERAL ASSURANCE V. COURT OF APPEALS, 361 SCRA 214 (2001) FINMAN'S GENERAL ASSURANCE vs. COURT OF APPEALS and USUPHIL Topic: Notice and Proof of Loss FACTS: 1. Usuphil obtained a fire insurance policy from Finman's Assurance covering certain properties, e.g., office, furniture, fixtures, shop machinery and other trade equipment, for damages and loss arising from fire. 2. Usuphil filed a claim for the loss of the insured properties due to fire. Finman's appointed Adjuster H.H. Bayne to undertake the valuation and adjustment of the loss. H.H. Bayne then required Usuphil to file a formal claim and submit proof of loss. 3.Usuphil submitted its Sworn Statement of Loss and Formal Claim signed by its Manager. It likewise submitted Proof of Loss signed by its Accounting Manager and countersigned by H.H. Baynes Adjuster. 4. Despite repeated demands by private respondent, Finman's refused to pay the insurance claim. Hence, Usuphil filed a complaint against Finman's for the unpaid insurance claim. 5. In its Answer, Finman's maintained that the claim could not be allowed because it failed to comply with Policy Condition No. 13 regarding the submission of certain documents to prove the loss. Policy Condition No. 13 reads:

13. The insured shall give immediate written notice to the Company of any loss, protect the property from further damage, forthwith separate the damaged and undamaged personal property, put it in the best possible order, furnish a complete inventory of the destroyed, damaged, and undamaged property, showing in detail quantities, costs, actual cash value and the amount of loss claimed; AND WITHIN SIXTY DAYS AFTER THE LOSS, UNLESS SUCH TIME IS EXTENDED IN WRITING BY THE COMPANY, THE INSURED SHALL RENDER TO THE COMPANY A PROOF OF LOSS, signed and sworn to by the insured, stating the knowledge and belief of the insured as to the following: the time and origin of the loss, the interest of the insured and of all others in the property, the actual cash value of each item thereof and the amount of loss thereto, all encumbrances thereon, all other contracts of insurance, whether valid or not, covering any of said property, any changes in the title, use, occupation, location, possession or exposures of said property since the issuing of this policy by whom and for what purpose any buildings herein described and the several parts thereof were occupied at the time of loss and whether or not it then stood on leased ground, and shall furnish a copy of all the descriptions and schedules in all policies, and if required verified plans and specifications of any building, fixtures, or machinery destroyed or damaged. The insured, as often as may be reasonably required, shall exhibit to any person designated by the company all that remains of any property herein described, and submit to examination under oath by any person named by the Company, and subscribe the same; and, as often as may be reasonably required, shall produce for examination all books of account, bills, invoices, and other vouchers or certified copies thereof if originals be lost, at such reasonable time and place as may be designated by the Company or its representative and shall permit extracts and copies thereof to be made. Page 68 of 154

INSURANCE G01 CASE DIGESTS that at a meeting between Usuphil and Finman's, the latter summoned the said manager to reconcile the claims. One who clothes another with apparent authority as his agent and holds him to the public as such, cannot later be allowed to deny the authority of such person to act as his agent when such third person entered into the contract in good faith and in an honest belief that he is such agent.

No claim under this policy shall be payable unless the terms of this condition have been complied with. 6. RTC ruled in favor of Usuphil. CA affirmed. Hence this present petition. Finman's alleged that complate diregard of the required documents communicated by Bayne, through a letter justified their failure to grant Usuphil's claim. ISSUE: WON disallowance of Usuphils claim is justified by its failure to submit the required documents in accordance with Policy Condition No. 13 HELD: NO RATIO DECIDENDI: According to the facts of this case, after the occurrence of the fire, Usuphil immediately notified Finman's thereof. Thereafter, it submitted the following documents: (1) Sworn Statement of Loss and Formal Claim and; (2) Proof of Loss. The submission of these documents constitutes substantial compliance with the above provision. Indeed, as regards the submission of documents to prove loss, substantial, not strict as urged by petitioner, compliance with the requirements will always be deemed sufficient. In any case, Finman itself acknowledged its liability when through its Finance Manager, it signed the document indicating that the amount due private Usuphil is P842,683.40. The contention that such manager was without authority to bind the corporation is without merit. The evidence indicate

109.

PACIFIC TIMBER V. COURT OF APPEALS, 112 SCRA 199 (1982)

Topic: Notice and Proof of Loss Pacific Timber Export Corp vs Court of Appeals Facts: March 19, 1963 Pacific Timber Export Corp secured temporary insurance from Workmens Insurance Company for its exportation of 1,250,000 board feet of Philippine lauan and Apitong logs to be shipped from Diapitan Bay, Quezon Province to Okinawa and Tokyo, Japan o Cover Note No. 1010 was issued insuring the said cargo subject to the terms and conditions of the 2 marine cargo policies issued on April 2, 1963 March 29, 1963 after the issuance of the cover note but before the issuance of the two marine policies, some of the logs intended to be exported were lost during loading operations in the Diapitan Bay. In a letter dated April 4, 1963, Pacific Timber informed the insurer about the loss of approximately 32 pieces of logs On July 17, 1963, Insurer requested their adjuster to inspect the loss and assess the damage. The latter reported that the loss of 30 pieces of logs is not covered by the Policies. Page 69 of 154

INSURANCE G01 CASE DIGESTS The insurer denied the claim on the ground that the investigation revealed that the entire shipment of logs covered by 2 policies were received in good order at their point of destination. Loss cannot be considered covered under the Cover Note as it became null and void by virtue of the issuance of the marine policies. therefore no cause for petitioner to lose what is due it as if there had been payment of premium. The defense of DELAY cannot be sustained. The law requires this ground of delay to be promptly and specifically asserted when a claim on the insurance agreement is made. Instead of invoking the ground of delay in objecting to petitioners claim of recovery on the Note, it took steps clearly indicative that his particular ground for objection to the claim was never in its mind. The nature of the round places the insurer on duty to inquire when the loss took place, so that it could determine whether delay would be a valid ground. From April 1963 to July 1963, enough time was available for Insurer to determine if petitioner was guilty of delay in communicating the loss to Insurer Sec 84 of the Insurance Act provides: Sec. 84 Delay in the presentation to an insurer of notice or proof of loss is waived if caused by any act of his or if he omits to take objection promptly and specifically upon that ground. Waiver can successfully be raised against private respondent

Issue: Whether or not the Insurance Company was correct in denying the claim based on the Cover Note issued without consideration? Held: NO. The Cover Note was not without consideration. The fact that no separate premium was paid on the Cover Note before the loss insured against occurred, does not militate against the validity of petitioners contention for no such premium could have been paid since by the nature of the Cover Note,, it did not contain, as all Cover Notes do not contain particulars of the shipment that would serve as basis for the computation of premiums. If the Note is to be treated as a separate policy instead of integrating it to the regular policies subsequently issued, the purpose and function of the Cove Note would be set at naught or rendered meaningless. It was not necessary to ask petitioner to pay premium on the Note, for the loss insured against having already occurred, the more practical procedure is simply to deduct the premium form the amount due the petitioner on the Note. The non payment is

Page 70 of 154

INSURANCE G01 CASE DIGESTS

DOUBLE INSURANCE
110. STA. ANA V. COMMERCIAL UNION ASSURANCE , 55 PHIL. 329 (1930) Facts On the 1st of October, 1925, the plaintiff Ulpiano Santa Ana took out a three-thousand-peso fire insurance policy on the house in the Phoenix Assurance Company (Exhibit C), and six-thousandpeso policy in the Guardian Assurance Company, Limited (Exhibit D), for a period of one year from that date until 4 o'clock in the afternoon of October 1, 1926, paying the respective premiums of P97.50 and P196 to said companies through their duly authorized Philippine agent, Kerr & Company. (Exhibit C and D) On November 19, 1925, the plaintiff Ulpiano Santa Ana mortgaged said house to the plaintiff Rafael Garcia for P5,000, for a period of two years, the contract being drawn up as a retro sale (Exhibit A) for the sum of P5,000, and the policies issued by the Phoenix Assurance Company and the Guardian Assurance Company, Limited, were endorsed to the mortgagee, Rafael Garcia (Exhibits C, D, and E). On December 16, 1925, the plaintiff Urpiano Santa Ana reinsured said house with the defendant companies, the Globe and Rutgers Fire Insurance Company of New York, and the Commercial Union Assurance Company, Limited of London, through their common agent duly authorized to represent them in the Philippine Islands, the Pacific Commercial Company, for the amount of P3,000 each, paying the 90-peso premium due upon each policy, which was to be effective for one year from the aforementioned date until 4 o'clock in the afternoon of December 16, 1926 (Exhibits B and B-1).

On September 20, 1926, Ulpiano Santa Ana took out another insurance policy on the house in question for P6,000 in the "Filipinas, Compania de Seguros, which issued the one-year policy Exhibit E, upon receiving from said plaintiff the amount of P195 as premium thereon. Issue: W/N the notice to the other insurance companies of the fact that ulpiano has procured other insurances are enough to not annul the insurance in her favour. Held: no. It should be noted that clause three of the "Filipinas" policy drawn up in Spanish, and the english policies issued by the four other companies, provided that any outstanding insurance upon the whole or a portion of the objects thereby assured must be declared by the insured in writing and he must cause the company to add or insert it in the policy, without which such policy shall be null and avoid, and the insured will not be entitled to indemnity in case of loss.lawphil.net Ulpiano Santa Ana maintains that he gave the required notice to all the insurance companies; To Kerr & Company through their sub-agent, Mariano Morelos; to the Pacific Commercial Company through their employee, Guillermo de Leon; and to the "Filipinas, Compania de Seguros" through their agent, Juan Grey; telling them he had paid for other insurance on the same property. But he has been contradicted in this by all the persons mentioned, and this deprives his allegations of probative force, especially considering that such advises or notices, so basic and essential to the existence and validity of the policies, must be given in writing as required in the noted attached to the four policies above mentioned, and must be given in writing as required in the note attached to the four policies above mentioned, and must be endorsed upon each of them, so that in case of necessity, as in the instant one, when a loss Page 71 of 154

INSURANCE G01 CASE DIGESTS occurs, the insured may clearly show that he has fulfilled this indispensable requisite, since all companies, to which people apply for insurance upon property already assured, have an interest in knowing what other policies issued by other companies the insured already holds, for the purpose of knowing just what interest the applicant has in the preservation of the property, and the care and precaution to be taken for the prevention of loss. Without deciding whether notice of other insurance upon the same property must be given in writing, or whether a verbal notice is sufficient to render an insurance valid which requires such notice, whether oral or written , we hold that in the absolute absence of such notice when it is one of the conditions specified in the fire insurance policy, the policy is null and void. 111. UNION MANUFACTURING V. PHI GUARANTY, 47 SCRA 271 (1972)

UNION MANUFACTURING CO., INC. VS. PHILIPPINE GUARANTY CO., INC. 47 SCRA 271 (G.R. NO. L-27932) OCTOBER 30, 1972 DOUBLE INSURANCE Petitioner: Republic Bank Respondent: Philippine Guaranty Co.. Inc. J. Fernando: FACTS: On January 12, 1962, the Union Manufacturing Co., Inc. obtained certain loans from the Republic Bank in the total sum of 415,000.00. To secure the payment thereof, UMC executed real and chattel mortgage on certain properties. The Republic Bank procured from the defendant Philippine Guaranty Co., Inc. an insurance coverage on loss against fire for 500,000.00 over the properties of the UMC, as described in defendants cover note dated September 25, 1962, with the annotation that loss or damage, if any, under said cover note is payable to Republic Bank as its interest may appear, subject however to the printed conditions of said defendants Fire Insurance Policy Form. On September 6, 1964, a fire occurred in the premises of UMC and on October 6, 1964, UMC filed its fire claim with the PGC Inc., thru its adjuster, H.H. Bayne Adjustment Co., which was denied by said defendant in its letter dated November 26, 1964 on the following ground: Policy Condition No. 3 and/or the Other Insurance Clause Page 72 of 154

INSURANCE G01 CASE DIGESTS of the policy was violated because you did not give notice to us of the other insurance which you had taken from New India for 80,000.00. Sincere Insurance for 25,000.00 and Manila Insurance for 200,000.00 with the result that these insurances of which we became aware of only after the fire, were not endorsed on our policy. ISSUE: Whether Republic Bank can recover. HELD: Without deciding- whether notice of other insurance upon the same property must be given in writing, or whether a verbal notice is sufficient to render an insurance valid which requires such notice, whether oral or written, we hold that in the absolute absence of such notice when it is one of the conditions specified in the fire insurance policy, the policy is null and void. (Santa Ana vs. Commercial Union Ass. Co., 55 Phil. 128). If the insured has violated or failed to perform the conditions of the contract, and such a violation or want of performance has not been waived by the insurer, then the insured cannot recover. Courts are not permitted to make contracts for the parties. The functions and duty of the courts consist simply in enforcing and carrying out the contracts actually made. While it is true, as a general rule, that contracts of insurance are construed most favorably to the insured, yet contracts of insurance, like other contracts, are to be construed according to the sense and meaning of the terms which the parties themselves have used. If such terms are clear and unambiguous they must be taken and understood in their plain, ordinary and popular sense. The annotation then, must be deemed to be a warranty that the property was not insured by any other policy. Violation thereof entitles the insurer to rescind. xxx The materiality of non disclosure of other insurance policies is not open to doubt.

The insurance contract may be rather onerous, but that in itself does not justify the abrogation of its express terms, terms which the insured accepted or adhered to and which is the law between the contracting parties. **lifted digest

112. PIONEER INSURANCE V. YAP, 61 SCRA 426 (1974) INSURANCE: Double Insurance PIONEER INSURANCE and SURETY CORPORATION vs OLIVA YAP (December 19, 1974) FACTS: Oliva Yap was the owner of a store in a 2-storey building in Manila where she sells bags and footwear. Chua Soon Poon, the son-in-law was in charge of the store. Yap procured a fire insurance policy from Pioneer Insurance and Surety Corp. (25,000 face value) on her stocks and office furniture. One of the conditions in the policy is that she shall notify the insurer of any other insurance already effected or subsequently procured, and state or endorse it in the policy. Otherwise the Policy shall be forfeited. There was another insurance from Great American Insurance Company which was noted on Pioneers policy and the parties endorsed Great Americans policy. Yap took out another fire insurance policy from Federal Insurance Company but without notice to and consent of Pioneer.

Page 73 of 154

INSURANCE G01 CASE DIGESTS A fire broke out in the building and the store was burned. Yap filed an insurance claim but Pioneer refused saying theres a violation of terms and conditions. CFI and CA: In favour of Oliva Yap company to make operative the clause avoiding the contract, wherever the specified conditions should occur. Its obligations ceased, unless, being informed of the fact, it consented to the additional insurance. The obvious purpose of the aforesaid requirement in the policy is to prevent over-insurance and thus avert the perpetration of fraud. The public, as well as the insurer, is interested in preventing the situation in which a fire would be profitable to the insured. According to Justice Story: "The insured has no right to complain, for he assents to comply with all the stipulation on his side, in order to entitle himself to the benefit of the contract, which, upon reason or principle, he has no right to ask the court to dispense with the performance of his own part of the agreement, and yet to bind the other party to obligations, which, but for those stipulation would not have been entered into."

ISSUE: WON Pioneer should be absolved from liability on account of Yaps violation of co-insurance clause. Yap argues that Federal merely substituted Great American. HELD: YES. Yap violated it. The insurance policy issued by Great American Insurance Company duly noted on Pioneers Policy co-insurance, ceased, by agreement of the parties to be recognized by them as a co-insurance policy. The Court of Appeals says that Great American policy was substituted by the Federal Insurance policy for the same amount, and because it was a mere case of substitution, there was no necessity for its endorsement on Pioneer policy. There is no evidence to establish and prove such a substitution. If anything was substituted for Great American policy, it could only be the Northwest Insurance policy for the same amount of P20,000.00. The endorsement shows the clear intention of the parties to recognize on the date the endorsement was made (August 29, 1962), the existence of only one coinsurance, and that is the Northwest Insurance policy, which according to the stipulation of the parties during the hearing, was issued on August 20, 1962 and endorsed only on August 20, 1962. The finding that Great American policy was substituted by the Federal Insurance policy is unsubstantiated. By the plain terms of the policy, other insurance without the consent of Pioneer would ipso facto avoid the contract. It required no affirmative act of election on the part of the

Page 74 of 154

INSURANCE G01 CASE DIGESTS 113. GEAGONIA V. COURT OF APPEALS, 241 SCRA 152 (1995) Geagonia vs CA GR 114427, 241 SCRA 152; February 6, 1995 Topic: Double Insurance: Payment of claim where there is overinsurance by double insurance Facts: Petitioner owned Normans Mart in Agusan del Sur. In December 1989, he filed insurance policy (fire insurance) from respondent for Php100,000, valid for one year, covering Stock-intrade consisting principally of dry goods such as RTW's for men and women wear and other usual to assured's business." Under the policy, he noted Mercantile Insurance as co-insurer for Php50,000. The policy contained the condition (Condition 3) that the insured shall notify the company of any insurance/s already affected or subsequently effected covering the same properties. The stipulation further states that unless notice be given and the particulars of such insurance or insurances be stated therein or endorsed in this policy pursuant to Section 50 of the Insurance Code, all benefits under this policy shall be deemed forfeited, provided however, that this condition shall not apply when the total insurance or insurances in force at the time of the loss or damage is not more than P200,000.00. His goods totaled a valued of Php392,130. The market burned in May 1990 and the petitioners stocks were completely destroyed. He filed to claim the insurance proceeds from the respondent, but was denied because the same goods were also covered by a third insurer (Philippines First Insurance) not disclosed with the respondent. The policies with the third insurer indicate petitioner as insured and show that there was even a mortgage attached over the properties with a loss payable clause to the mortgagee. Petitioner filed with the Insurance Commission for recovery of the Php100,000 and attorneys fees. He also claims that the value of the goods actually valued at Php1,000,000 at time of loss. Petition was denied. Upon reconsideration, he attached a document Annex A admitting that at the time he obtained the policy from the respondent, the policies by Philippine First Insurance were already existing. He claims though that nothing in the respondents policy required him to disclose other subsisting policies and that the respondents agent failed to inform him of this, if there was any. Respondent denied the allegations and claimed he is not entitled to the proceeds for violating the condition. The insurance commission reversed and stated the petitioner had no knowledge that its creditor Cebu Tesing obtained the policy from Philippine First Insurance over the goods, and even benefitted from it upon the fire. Respondents motion for reconsideration was denied, thus it appealed to the CA. The CA reversed the insurance commission and found the petitioner knew of the existence of the third policy because it was in his name and he even paid the premiums. Issue: 1. Did the petitioner actually have knowledge of the third policys existence thus violating condition 3 of the policy with the respondent? 2. If he had, may he still recover the proceeds? Held: 1. YES 2. NO Rationale: We agree with the Court of Appeals that the petitioner knew of the prior policies issued by the PFIC. His letter of 18 January 1991 to the private respondent conclusively proves this knowledge. His testimony to the contrary before the Insurance Commissioner and which the latter relied upon cannot prevail over a written admission madeante litem motam. It was, indeed, incredible that he did not know about the prior policies since these policies were not new or Page 75 of 154

INSURANCE G01 CASE DIGESTS original. Policy No. GA-28144 was a renewal of Policy No. F-24758, while Policy No. GA-28146 had been renewed twice, the previous policy being F-24792. Condition 3 of the private respondent's Policy No. F-14622 is a condition which is not proscribed by law. Its incorporation in the policy is allowed by Section 75 of the Insurance Code which provides that "[a] policy may declare that a violation of specified provisions thereof shall avoid it, otherwise the breach of an immaterial provision does not avoid the policy." Such a condition is a provision which invariably appears in fire insurance policies and is intended to prevent an increase in the moral hazard. It is commonly known as the additional or "other insurance" clause and has been upheld as valid and as a warranty that no other insurance exists. Its violation would thus avoid the policy. However, in order to constitute a violation, the other insurance must be upon same subject matter, the same interest therein, and the same risk. It must be underscored that unlike the "other insurance" clauses involved in General Insurance and Surety Corp. vs. Ng Hua or in Pioneer Insurance & Surety Corp. vs. Yap, Condition 3 in the private respondent's policy No. F-14622 does not absolutely declare void any violation thereof. It expressly provides that the condition "shall not apply when the total insurance or insurances in force at the time of the loss or damage is not more than P200,000.00." It is a cardinal rule on insurance that a policy or insurance contract is to be interpreted liberally in favor of the insured and strictly against the company, the reason being, undoubtedly, to afford the greatest protection which the insured was endeavoring to secure when he applied for insurance. It is also a cardinal principle of law that forfeitures are not favored and that any construction which would result in the forfeiture of the policy benefits for the person claiming, will be avoided, if it is possible to construe the policy in a manner which would permit recovery, as, for example, by finding a waiver for such forfeiture. Stated differently, provisions, conditions or exceptions in policies which tend to work a forfeiture of insurance policies should be construed most strictly against those for whose benefits they are inserted, and most favorably toward those against whom they are intended to operate. The reason for this is that, except for riders which may later be inserted, the insured sees the contract already in its final form and has had no voice in the selection or arrangement of the words employed therein. On the other hand, the language of the contract was carefully chosen and deliberated upon by experts and legal advisers who had acted exclusively in the interest of the insurers and the technical language employed therein is rarely understood by ordinary laymen. With these principles in mind, we are of the opinion that Condition 3 of the subject policy is not totally free from ambiguity and must, perforce, be meticulously analyzed. Such analysis leads us to conclude that (a) the prohibition applies only to double insurance, and (b) the nullity of the policy shall only be to the extent exceeding P200,000.00 of the total policies obtained. The first conclusion is supported by the portion of the condition referring to other insurance "covering any of the property or properties consisting of stocks in trade, goods in process and/or inventories only hereby insured," and the portion regarding the insured's declaration on the subheading CO-INSURANCE that the coinsurer is Mercantile Insurance Co., Inc. in the sum of P50,000.00. A double insurance exists where the same person is insured by several insurers separately in respect of the same subject and interest. As earlier stated, the insurable interests of a mortgagor and a mortgagee on the mortgaged property are distinct and separate. Since the two policies of the PFIC do not cover the same interest as that covered by the policy of the private respondent, no double insurance exists. The non-disclosure then of the former policies was not fatal to the petitioner's right to recover on the private respondent's policy. Furthermore, by stating within Condition 3 itself that such condition shall not apply if the total insurance in force at the time of loss does not exceed P200,000.00, the private respondent was amenable to assume a co-insurer's liability up to a loss not Page 76 of 154

INSURANCE G01 CASE DIGESTS exceeding P200,000.00. What it had in mind was to discourage over-insurance. Indeed, the rationale behind the incorporation of "other insurance" clause in fire policies is to prevent over-insurance and thus avert the perpetration of fraud. When a property owner obtains insurance policies from two or more insurers in a total amount that exceeds the property's value, the insured may have an inducement to destroy the property for the purpose of collecting the insurance. The public as well as the insurer is interested in preventing a situation in which a fire would be profitable to the insured. 4. In Jan. 1964, a fire broke out destroying the subject property. 5. Pacific Banking demanded payment from Oriental. a. Pacific did not present any proof of loss. 6. Oriental did not accede to the demand because it was waiting for the insurance adjuster report. 7. The insurance adjust notified Pacific Banking that Paramount had not filed any claim with Oriental yet, nor submitted proof of loss which is a clear violation of Policy Condition No.11, and for which reason, determination of the liability of Oriental could not be had. a. Pacific Banking answered the insurance adjuster saying that it can get a proof of loss by verifying from the records of the Bureau of Customs the entries of merchandise taken into the customs bonded warehouse razed by fire. 8. But still, Oriental refused to pay. 9. Pacific Banking filed an action for sum of money against Oriental P61,000. 10. Oriental raised the following defenses: a. lack of formal claim by insured over the loss and b. premature filing of the suit as neither plaintiff nor insured had submitted any proof of loss on the basis of which defendant would determine its liability and the amount thereof 11. During the trial, Pacific presented in evidence a document revealing undeclared co-insurances with 6 other insurance companies taken by Paramount covering the same goods destroyed. a. It will be noted that the defense of fraud and/or violation of Condition No. 3 in the Policy, in the Page 77 of 154

114. PACIFIC BANKING CORP. V. COURT OF APPEALS, 168 SCRA 1 (1968) Paras, J. Facts 1. An open fire policy was issued to Paramount Shirt by insurer, Oriental Assurance. a. Oriental bound itself to indemnify the insured for any loss or damage, not exceeding P61,000.00, caused by fire to its property consisting of stocks, materials and supplies usual to a shirt factory from Oct. 21, 1965 to Oct. 21, 1964 2. Paramount Shirt owed money from Pacific Banking in the amount of P800,000. As security, the goods insured was held by Pacific Banking in trust. 3. Said policy was duly endorsed to petitioner as mortgagee/ trustor of the properties insured, with the knowledge and consent of Oriental to the effect that "loss if any under this policy is payable to the Pacific Banking Corporation".

INSURANCE G01 CASE DIGESTS form of non-declaration of co-insurances which was not pleaded in the answer was also not pleaded in the Motion to Dismiss. 12. RTC ruled in favor of Pacific Banking. Oriental should pay. 13. CA reversed the decision. 14. Hence, this petition. deception. Otherwise stated, had the insurer known that there were many co-insurances, it could have hesitated or plainly desisted from entering into such contract. Hence, the insured was guilty of clear fraud (Rollo, p. 25). Petitioner's contention that the allegation of fraud is but a mere inference or suspicion is untenable. In fact, concrete evidence of fraud or false declaration by the insured was furnished by the petitioner itself when the facts alleged in the policy under clauses "Co-Insurances Declared" and "Other Insurance Clause" are materially different from the actual number of co-insurances taken over the subject property. On the question of Pacific Bankings right to the proceeds as the mortgagee, the SC held that: o It is but fair and just that where the insured who is primarily entitled to receive the proceeds of the policy has by its fraud and/or misrepresentation, forfeited said right, with more reason Pacific which is merely claiming as indorsee of said insured, cannot be entitled to such proceeds. o It will be noted that the fact of fraud was tried by express or at least implied consent of the parties. Pacific did not only object to the introduction of evidence but on the contrary, presented the very evidence that proved its existence.

Issues 1. Whether or not the non-declaration of co-insurances violates the policy condition which required the insured to reveal other insurances already effected. a. Yes. 2. Whether or not failure of the insured to file the required proof of loss prior to court action. a. Yes.

Ruling First Issue As the insurance policy against fire expressly required that notice should be given by the insured of other insurance upon the same property, the total absence of such notice nullifies the policy. It is not disputed that the insured failed to reveal before the loss three other insurances. As found by the Court of Appeals, by reason of said unrevealed insurances, the insured had been guilty of a false declaration; a clear misrepresentation and a vital one because where the insured had been asked to reveal but did not, that was

Second Issue Generally, the cause of action on the policy accrues when the loss occurs, But when the policy provides that no action shall be brought unless the claim is first presented Page 78 of 154

INSURANCE G01 CASE DIGESTS extrajudicially in the manner provided in the policy, the cause of action will accrue from the time the insurer finally rejects the claim for payment (Eagle Star Insurance v. Chia Yu, 55 Phil 701 [1955]). In the case at bar, policy condition No. 11 specifically provides that the insured shall on the happening of any loss or damage give notice to the company and shall within fifteen (15) days after such loss or damage deliver to the private respondent (a) a claim in writing giving particular account as to the articles or goods destroyed and the amount of the loss or damage and (b) particulars of all other insurances, if any. Likewise, insured was required "at his own expense to produce, procure and give to the company all such further particulars, plans, specifications, books, vouchers, invoices, duplicates or copies thereof, documents, proofs and information with respect to the claim". (Record on Appeal, pp. 18-20). The evidence adduced shows that twenty-four (24) days after the fire, petitioner merely wrote letters to private respondent to serve as a notice of loss, thereafter, the former did not furnish the latter whatever pertinent documents were necessary to prove and estimate its loss. Instead, petitioner shifted upon private respondent the burden of fishing out the necessary information to ascertain the particular account of the articles destroyed by fire as well as the amount of loss. It is noteworthy that private respondent and its adjuster notified petitioner that insured had not yet filed a written claim nor submitted the supporting documents in compliance with the requirements set forth in the policy. Despite the notice, the latter remained unheedful. Since the required claim by insured, together with the preliminary submittal of relevant documents had not been complied with, it follows that private respondent could not be deemed to have finally rejected petitioner's claim and therefore the latter's cause of action had not yet arisen. SC affirms CA. Petition dismissed.

Page 79 of 154

INSURANCE G01 CASE DIGESTS

REINSURANCE
115. PIONEER INSURANCE V. COURT OF APPEALS, 175 SCRA 668 (1989) PIONEER INSURANCE & SURETY CORP V CA, BORMAHECO, MAGLANA AND LIM G.R. No. 84197 LIM V CA, PIONEER INSURANCE AND SURETY CORP, BORMAHECO, CERVANTES AND MAGLANA FACTS: 2 consolidated petitions 1965 Jacob S. Lim was the owner-operator of Southern Air Lines (SAL) a single proprietorship May 17, 1985 at Tokyo, Japan, Japan Domestic Airlines (JDA) and Lim entered into and executed a sales contract for the sale and purchase of and purchase of two (2) DC-3A Type aircrafts and one (1) set of necessary spare parts for the total agreed price of US $109,000.00 to be paid in installments. One DC-3 Aircraft with Registry No. PIC-718, arrived in Manila on June 7,1965 while the other aircraft, arrived in Manila on July 18,1965. On May 22, 1965, Pioneer Insurance and Surety Corporation (Pioneer, petitioner in G.R. No. 84197) as surety executed and issued its Surety Bond No. 6639 (Exhibit C) in favor of JDA, in behalf of its principal, Lim, for the balance price of the aircrafts and spare parts. It appears that Border Machinery and Heavy Equipment Company, Inc. (Bormaheco), Francisco and Modesto Cervantes (Cervanteses) and Constancio Maglana (respondents in both petitions) contributed some funds used in the purchase of the above aircrafts and spare parts.

The funds were supposed to be their contributions to a new corporation proposed by Lim to expand his airline business. They executed two (2) separate indemnity agreements (Exhibits D-1 and D-2) in favor of Pioneer, one signed by Maglana and the other jointly signed by Lim for SAL, Bormaheco and the Cervanteses. indemnity agreements stipulated that the indemnitors principally agree and bind themselves jointly and severally to indemnify and hold and save harmless Pioneer from and against any/all damages, losses, costs, damages, taxes, penalties, charges and expenses of whatever kind and nature which Pioneer may incur in consequence of having become surety June 10, 1965 - Lim doing business under the name and style of SAL executed in favor of Pioneer as deed of chattel mortgage as security for the latter's suretyship in favor of the former. It was stipulated therein that Lim transfer and convey to the surety the two aircrafts. The deed (Exhibit D) was duly registered with the Office of the Register of Deeds of the City of Manila and with the Civil Aeronautics Administration pursuant to the Chattel Mortgage Law and the Civil Aeronautics Law (Republic Act No. 776), respectively. Lim defaulted on his subsequent installment payments prompting JDA to request payments from the surety. Pioneer paid a total sum of P298,626.12. Pioneer then filed a petition for the extrajudicial foreclosure of the said chattel mortgage before the Sheriff of Davao City. The Cervanteses and Maglana, however, filed a third party claim alleging that they are co-owners of the aircrafts, On July 19, 1966, Pioneer filed an action for judicial foreclosure with an application for a writ of preliminary Page 80 of 154

INSURANCE G01 CASE DIGESTS attachment against Lim and respondents, the Cervanteses, Bormaheco and Maglana. ISSUES/HELD/RATIO: G.R. No. 84197-relevant discussion 1. Has Pioneer a cause of action against defendants with respect to so much of its obligations to JDA as has been paid with reinsurance money? NO 2. If the answer to the preceding question is in the negative, has Pioneer still any claim against defendants, considering the amount it has realized from the sale of the mortgaged properties? NO The total amount paid by Pioneer to JDA is P299,666.29. Since Pioneer has collected P295,000.00 from the reinsurers, the uninsured portion of what it paid to JDA is the difference between the two amounts, or P3,666.28. This is the amount for which Pioneer may sue defendants, assuming that the indemnity agreement is still valid and effective. But since the amount realized from the sale of the mortgaged chattels are P35,000.00 for one of the airplanes and P2,050.00 for a spare engine, or a total of P37,050.00, Pioneer is still overpaid by P33,383.72. Therefore, Pioneer has no more claim against defendants. The payment to the petitioner made by the reinsurers was not disputed in the appellate court. Considering this admitted payment, the only issue that cropped up was the effect of payment made by the reinsurers to the petitioner. Therefore, the petitioner's argument that the respondents had no interest in the reinsurance contract as this is strictly between the petitioner as insured and the reinsuring company pursuant to Section 91 (should be Section 98) of the Insurance Code has no basis. In general a reinsurer, on payment of a loss acquires the same rights by subrogation as are acquired in similar cases where the original insurer pays a loss (Universal Ins. Co. v. Old Time Molasses Co. C.C.A. La., 46 F 2nd 925). The rules of practice in actions on original insurance policies are in general applicable to actions or contracts of reinsurance. (Delaware, Ins. Co. v. Pennsylvania Fire Ins. Co., 55 S.E. 330,126 GA. 380, 7 Ann. Con. 1134). Hence the applicable law is Article 2207 of the new Civil Code, to wit: Art. 2207. If the plaintiffs property has been insured, and he has received indemnity from the insurance company for the injury or loss arising out of the wrong or breach of contract complained of, the insurance company shall be subrogated to the rights of the insured against the wrongdoer or the person who has violated the contract. If the amount paid by the insurance company does not fully cover the injury or loss, the aggrieved party shall be entitled to recover the deficiency from the person causing the loss or injury. Interpreting the aforesaid provision, we ruled in the case of Phil. Air Lines, Inc. v. Heald Lumber Co. (101 Phil. 1031 [1957]) which we subsequently applied in Manila Mahogany Manufacturing Corporation v. Court of Appeals (154 SCRA 650 [1987]): Note that if a property is insured and the owner receives the indemnity from the insurer, it is provided in said article that the insurer is deemed Page 81 of 154

INSURANCE G01 CASE DIGESTS subrogated to the rights of the insured against the wrongdoer and if the amount paid by the insurer does not fully cover the loss, then the aggrieved party is the one entitled to recover the deficiency. Evidently, under this legal provision, the real party in interest with regard to the portion of the indemnity paid is the insurer and not the insured. (Emphasis supplied). It is clear from the records that Pioneer sued in its own name and not as an attorney-in-fact of the reinsurer. Moreover, the indemnity agreement ceased to be valid and effective after the execution of the chattel mortgage. This is judicial admission and aside from the chattel mortgage there is no other security for the claim sought to be enforced by this action, which necessarily means that the indemnity agreement had ceased to have any force and effect at the time this action was instituted. Sec 2, Rule 129, Revised Rules of Court. The indemnity agreement was ipso jure extinguished upon the foreclosure of the chattel mortgage. These defendants, as indemnitors, would be entitled to be subrogated to the right of Pioneer should they make payments to the latter. Articles 2067 and 2080 of the New Civil Code of the Philippines. Other bases/issues: Independently of the preceding proposition Pioneer's election of the remedy of foreclosure precludes any further action to recover any unpaid balance of the price. SAL or Lim, having failed to pay the second to the eight and last installments to JDA and Pioneer as surety having made of the payments to JDA, the alternative remedies open to Pioneer were as provided in Article 1484 of the New Civil Code, known as the Recto Law (=p). Pioneer exercised the remedy of foreclosure of the chattel mortgage both by extrajudicial foreclosure and the instant suit. Also, Pioneer's liability as surety to JDA had already prescribed when Pioneer paid the same. Consequently, Pioneer has no more cause of action to recover from these defendants, as supposed indemnitors, what it has paid to JDA. By virtue of an express stipulation in the surety bond, the failure of JDA to present its claim to Pioneer within ten days from default of Lim or SAL on every installment, released Pioneer from liability from the claim. Therefore, Pioneer is not entitled to exact reimbursement from these defendants thru the indemnity. (Art 1318) Petition in G.R. No. 84197 is not meritorious. G.R. No. 84157 1. What legal rules govern the relationship among co-investors whose agreement was to do business through the corporate vehicle but who failed to incorporate the entity in which they had chosen to invest? 2. How are the losses to be treated in situations where their contributions to the intended 'corporation' were invested not through the corporate form? Principles governing: While it has been held that as between themselves the rights of the stockholders in a defectively incorporated association should be governed by the supposed charter and the laws of the state relating thereto and not by the rules governing partners, it is ordinarily held that persons who attempt, but fail, to form a corporation and who carry on business under the corporate name occupy the position of partners inter. Thus, where persons associate themselves together under articles to purchase property to carry on a business, and their organization Page 82 of 154

INSURANCE G01 CASE DIGESTS is so defective as to come short of creating a corporation within the statute, they become in legal effect partners inter se, and their rights as members of the company to the property acquired by the company will be recognized. Petitioner denied having received any amount from respondents Bormaheco, the Cervanteses and Maglana. The trial court and the appellate court, however, found through Exhibit 58, that the petitioner received the amount of P151,000.00 representing the participation of Bormaheco and Atty. Constancio B. Maglana in the ownership of the subject airplanes and spare parts. The record shows that defendant Maglana gave P75,000.00 to petitioner Jacob Lim thru the Cervanteses. It is therefore clear that the petitioner never had the intention to form a corporation with the respondents despite his representations to them. This gives credence to the cross-claims of the respondents to the effect that they were induced and lured by the petitioner to make contributions to a proposed corporation which was never formed because the petitioner reneged on their agreement. Applying therefore the principles of law earlier cited to the facts of the case, necessarily, no de facto partnership was created among the parties which would entitle the petitioner to a reimbursement of the supposed losses of the proposed corporation. The record shows that the petitioner was acting on his own and not in behalf of his other would-be incorporators in transacting the sale of the airplanes and spare parts. DISPOSITIVE: petitions are DISMISSED. Decision of CA AFFIRMED. 116. GIBSON V. REVILLA, 92 SCRA 219 (1979)

GIBSON v. REVILLA and LEPANTO CONSOLIDATED MINING COMPANY FACTS: Lepanto Consolidated Mining Company filed a complaint against Malayan Insurance Company, Inc. The civil suit thus instituted by Lepanto against Malayan was founded on the fact that Malayan issued a Marine Open Policy covering all shipments of copper, gold, and silver concentrates in bulk from Poro, San Fernando, La Union to Tacoma, Washington or to other places in the United States. Thereafter, Malayan obtained reinsurance abroad through Sedgwick, Collins & Co., Limited, a London insurance brokerage. The Memorandum of Insurance issued by Sedgwick to Malayan listed three groups of underwriters or reinsurers Lloyds 62.808%, Companies (I.L.U.) 34.705%, Other companies 2.487%. At the top of the list of underwriting members of Lloyds is Syndicate No. 448, assuming 2.48% of the risk assumed by the reinsurer, which syndicate number petitioner Ivor Robert Dayton Gibson claims to be himself. Petitioner then filed a motion to intervene as defendant, which motion was denied by the lower court. ISSUE: WHETHER OR NOT THE LOWER COURT COMMITTED, REVERSIBLE ERROR IN REFUSING THE INTERVENTION OF THE PETITIONER IN THE SUIT BETWEEN LEPANTO AND MALAYAN COMPANIES. HELD: No. The respondent Judge committed no error of law in denying petitioners Motion to Intervene and neither has he abused his discretion in his denial of petitioners Motion for Intervention. We agree with the holding of the respondent court that since Page 83 of 154

INSURANCE G01 CASE DIGESTS movant Ivor Robert Dayton Gibson appears to be only one of several re-insurers of the risks and liabilities assumed by Malayan Insurance Company, Inc., it is highly probable that other re-insurers may likewise intervene. If petitioner is allowed to intervene, We hold that there is good and sufficient basis for the Court a quo to declare that the trial between Lepanto and Malayan would be definitely disrupted and would certainly unduly delay the proceedings between the parties especially at the stage where Lepanto had already rested its case and that the issue would also be compounded as more parties and more matters will have to be litigated. In other words, the Courts discretion is justified and reasonable. We also hold that respondent Judge committed no reversible error in further sustaining the fourth ground of Lepantos Opposition to the Motion to Intervene that the rights, if any, of petitioner are not prejudiced by the present suit and will be fully protected in a separate action against him and his coinsurers by Malayan. Petitioners contention that he has to pay once Malayan is finally adjudged to pay Lepanto because of the very nature of a contract of reinsurance and considering that the re-insurer is obliged to pay as may be paid thereon (referring to the original policies), although this is subject to other stipulations and conditions of the reinsurance contract, is without merit. The general rule in the law of reinsurance is that there-insurer is entitled to avail itself of every defense which the re-insured (which is Malayan)might urge in an action by the person originally insured (which is Lepanto). As to the effect of the clause to pay as may be paid thereon contained in petitioners re-insurance contract, Arnould, on the Law of Marine Insurance and Average, 13thEd., Vol. 1, Section 327, p. 315, states the rule, this: It has been decided that this clause does not preclude the reinsurer from insisting upon proper proof that a loss strictly within the terms of the original policy has taken place. This clause does not enable the original underwriter to recover from his reinsurer to an extent beyond the subscription of the latter. Wherefore, in view of the foregoing, the petition is hereby dismissed. No costs.

117. ARTEX DEVELOPMENT V. WELLINGTON INSURANCE , 51 SCRA 352 (1973)

ARTEX DEVELOPMENT CO INC v. WELLINGTON INSURANCE CO INC 51 SCRA 352 TEEHANKEE; June 27, 1973 FACTS - Wellington Insurance Co. Inc. insured for P24,346,509.00 the buildings, stocks and machinery of plaintiff Artex Development Co. Inc. against loss or damage by fire or lighting upon payment of the plaintiff of the corresponding premiums; that said properties were insured for an additional sum of P883,034.00; that defendant insured plaintiff against business interruption (use and occupancy) for P5,200,000.00; Wellington entered into a contract of reinsurance with Alexander and Alexander, Inc. of New York. USA. - The buildings, stocks and machineries of plaintiffs spinning department were burned. - Notice of the loss and damage was given the defendant; that as per report of the adjusters, the total property loss of the plaintiff was the sum of P10,106,554.40 and the total business interruption loss was P3,000,000.00; Page 84 of 154

INSURANCE G01 CASE DIGESTS - That defendant has paid to the plaintiff the sum of P6,481,870.07 of the property loss suffered by plaintiff and P1,864,134.08 on its business interruption loss, leaving a balance of P3,624,683.43 and P1,748,460.00, respectively. - The counsel for Artex filed a Manifestation saying that in view of the Deeds of Discharge and Collateral Agreement, the only remaining liability subject of litigation shall be the proportion of the loss reinsured with or through Alexander and Alexander, Inc. of New York, USA, namely, P397,813.00. - The document recited further that Artex acknowledges receipt of the sum of P3.6M paid by the insurer in full and final settlement of all or any claims of Artex against its insurer. It discharges its insurer from all actions, proceedings, claims, demands, costs and expenses in respect thereof. - With regard the balance unpaid, Wellington contends that Artex should have been directed against the reinsurers to cover the liability and not against Wellington. benefit or favor to the insured, the insured, not being privy to the reinsurance contract, has no cause of action against the reinsurer. It is expressly provided in Section 91 the Insurance Act 1 that "(T)he original insured has no interest in a contract of insurance."

ISSUE WON the insured (Artex) has a cause of action against the reinsurer HELD NO - Unless there is a specific grant in, or assignment of, the reinsurance contract in favor of the insured or a manifest intention of the contracting parties to the reinsurance contract to grant such Page 85 of 154

INSURANCE G01 CASE DIGESTS Respondent ignored the petitioner demand for payment of 150,000.00 for the loss of the shipment plus 100,000.0 0 as unrealized profits. Respondent Pioneer denied the claim of petitioner for the full amount of 100,000.00 on the ground that its liability depended upon the total loss of vessel only. The trial court decided in favor of the plaintiff (petitioner). The appellate court modified the trial courts decision and absolved Pioneer from liability after finding that there was a breach of implied warranty of seaworthiness on the part of the petitioners and that the loss of the insured cargo was caused by the perils of the ship and not by the perils of sea. It ruled that the loss is not covered by the marine insurance policy. ISSUE: Whether or not the implied warranty of seaworthiness in marine insurance attaches to the shipper who is not the shipowner. HELD: Section 113 of the Insurance Code provides: In every marine insurance upon a ship or freight, or freightage, or upon any thing which is the subject of marine insurance, a warranty is implied that the ship is seaworthy. Section 99 of the same Code also provides in part. Marine insurance includes: (1) Insurance against loss of or damage to: (a) Vessels, craft, aircraft, vehicles, goods, freights, cargoes, merchandise, From the above-quoted provisions, there can be no mistaking the fact that the term cargo can be the subject of marine insurance and that once it is so made, the implied warranty of seaworthiness immediately attaches to whoever is insuring the cargo whether he be the shipowner or not.

MARINE INSURANCE
118. ROQUE V. INTERMEDIATE APPELLATE COURT, 139 SCRA 596 (1985) ROQUE V. INTERMEDIATE APPELLATE COURT, 139 SCRA 596 (1985) Petitioner: Isabela Roque, doing busines under the name and style of Isabela Roque Timber Enterprises and Ong Chiong Respondent: Intermediate Appelate Court and Pioneer Insurance And Surety Corporation

FACTS: Manila Bay Lighterage Corporation (Manila Bay) a common carrier, entered into a contract with petitioners whereby the former would load and cary on board its barge Marble 10 about 422.18 cubic meters of logs from Malampaya Sound, Palawan to North Harbor Manila. The petitioners insured the logs against loss for 100,000.00 with respondent Pioneer Insurance and Surety Corporation (Pioneer). The petitioner loaded on the barge, 811 pieces of logs at Malampaya Sound, Palawan for carriage and delivery to North Harbor, Port of Manila, but the shipment never reached its destination because Marble 10 sank with the 811 pieces of logs somewhere off Cabuli Point in Palawan on its way to Manila. As alleged by the petitioners in their complaint and as found by both the trial and appellate courts, the barge where the logs were loaded was not seaworthy such that it developed a leak. The appellate court further found that one of the hatches was left open causing water to enter the barge and because the barge was not provided with the necessary cover or tarpauline, the ordinary splash of seawaves brought more water inside the barge.

Page 86 of 154

INSURANCE G01 CASE DIGESTS Moreover, the fact that the unseaworthiness of the ship was unknown to the insured is immaterial in ordinary marine insurance and may not be used by him as a defense in order to recover on the marine insurance policy. Since the law provides for an implied warranty of seaworthiness in every contract of ordinary marine insurance, it becomes the obligation of a cargo owner to look for a reliable common carrier which keeps its vessels in seaworthy condition. The shipper of cargo may have no control over the vessel but he has full control in the choice of the common carrier that will transport his goods. Or the cargo owner may enter into a contract of insurance which specifically provides that the insurer answers not only for the perils of the sea but also provides for coverage of perils of the ship. There is no doubt that the term perils of the sea extends only to losses caused by sea damage, or by the violence of the elements, and does not embrace all losses happening at sea. They insure against losses from extraordinary occurrences only, such as stress of weather, winds and waves, lightning, tempests, rocks and the like. These are understood to be the perils of the sea referred in the policy, and not those ordinary perils which every vessel must encounter. Perils of the sea has been said to include only such losses as are of extraordinary nature, or arise from some overwhelming power, which cannot be guarded against by the ordinary exertion of human skill and prudence. Damage done to a vessel by perils of the sea includes every species of damages done to a vessel at sea, as distinguished from the ordinary wear and tear of the voyage, and distinct from injuries suffered by the vessel in consequence of her not being seaworthy at the outset of her voyage (as in this case). It is also the general rule that everything which happens thru the inherent vice of the thing, or by the act of the owners, master or shipper, shall not be reputed a peril, if not otherwise borne in the policy. On the contention of the petitioners that the trial court found that the loss was occasioned by the perils of the sea characterized by the storm and waves which buffeted the vessel, the records show that the court ruled otherwise. It stated: x x x The other affirmative defense of defendant Lighterage, That the supposed loss of the logs was occasioned by force majeure was not supported by the evidence. At the time Mable 10 sank, there was no typhoon but ordinary strong wind and waves, a condition which is natural and normal in the open sea. The evidence shows that the sinking of Mable 10 was due to improper loading of the logs on one side so that the barge was tilting on one side and for that it did not navigate on even keel; that it was no longer seaworthy that was why it developed leak; that the personnel of the tugboat and the barge committed a mistake when it turned loose the barge from the tugboat east of Cabuli point where it was buffeted by storm and waves, while the tugboat proceeded to west of Cabuli point where it was protected by the mountain side from the storm and waves coming from the east direction. x x x lt must be considered to be settled, furthermore, that a loss which, in the ordinary course of events, results from the natural and inevitable action of the sea, from the ordinary wear and tear of the ship, or from the negligent failure of the ships owner to provide the vessel with proper equipment to convey the cargo under ordinary conditions, is not a peril of the sea. Such a loss is rather due to what has been aptly called the peril of the ship. The insurer undertakes to insure against perils of the sea and similar perils, not against perils of the ship. As was well said by Lord Herschell in Wilson, Sons & Co. v. Owners of Cargo per the Xantho ([1887], 12 A. C., 503, 509), there must, in order to make the insurer liable, be some casualty, something which could not be foreseen as one of the necessary incidents of the adventure. The purpose of the policy is to secure an indemnity against accidents which may happen, not against events which must happen.

Page 87 of 154

INSURANCE G01 CASE DIGESTS Barratry as defined in American Insurance Law is any willful misconduct on the part of master or crew in pursuance of some unlawful or fraudulent purpose without the consent of the owners, and to the prejudice of the owners interest, (Sec. 171, U.S. Insurance Law, quoted in Vance, Handbook on Law of Insurance, 1951, p. 929.) Barratry necessarily requires a willful and intentional act in its commission. No honest error of judgment or mere negligence, unless criminally gross, can be barratry. (See Vance on Law of Insurance, p. 929 and cases cited therein.) In the case at bar, there is no finding that the loss was occasioned by the willful or fraudulent acts of the vessels crew. There was only simple negligence or lack of skill. Hence, the second assignment of error must likewise be dismissed. 119. GO TIACO V. UNION INSURANCE OF CANADA, 40 PHIL. 40 (1919) GO TIACO v. UNION INSURANCE FACTS Union Insurance Society of Canton, Ltd., issued a marine insurance policy upon a cargo of rice belonging to the Go Tiaoco Brothers, which was transported in the early days of May, 1915, on the steamship Hondagua from the port of Saigon to Cebu. On discharging the rice from one of the compartments in the after hold, upon arrival at Cebu, it was discovered that 1473 sacks had been damaged by sea water. The loss was P3,875.25. The trial court found that the inflow of the sea water during the voyage was due to a defect in one of the drain pipes of the ship and concluded that the loss was not covered by the policy of insurance. The trial court made the ff findings: The drain pipe which served as a discharge from the water closet passed down through the compartment where the rice in question was stowed and thence out to sea through the wall of the compartment, which was a part of the wall of the ship. The joint or elbow where the pipe changed its direction was of cast iron; and in course of time it had become corroded and abraded until a longitudinal opening had appeared in the pipe about one inch in length. This hole had been in existence before the voyage was begun, and an attempt had been made to repair it by filling with cement and bolting over it a strip of iron. The effect of loading the boat was to submerge the vent, or orifice, of the pipe until it was about 18 inches or 2 feet below the level of the sea. As a consequence the sea water rose in the pipe. Navigation under these conditions resulted in the washing out of the cement-filling from the action of the sea water, thus permitting the continued flow of the salt water into the compartment of rice.

Page 88 of 154

INSURANCE G01 CASE DIGESTS The court found in effect that the opening above described had resulted in course of time from ordinary wear and tear and not from the straining of the ship in rough weather on that voyage. The court also found that the repairs that had been made on the pipe were slovenly and defective and that, by reason of the condition of this pipe, the ship was not properly equipped to receive the rice at the time the voyage was begun. For this reason the court held that the ship was unseaworthy. The policy purports to insure the cargo from the following among other risks: "Perils . . . of the seas, men, of war, fire, enemies, pirates, rovers, thieves, .jettisons, . . . barratry of the master and mariners, and of all other perils, losses, and misfortunes that have or shall come to the hurt, detriment, or damage of the said goods and merchandise or any part thereof." ISSUE WON Union Insurance is liable for the loss of the Go Tiaco Brothers HELD NO - the words "all other perils, losses, and misfortunes" are to be interpreted as covering risks which are of like kind (ejusdem generis) with the particular risks which are enumerated in the preceding part of the same clause of the contract. ''According to the ordinary rules of construction, these words must be interpreted with reference to the words which immediately precede them. They were no doubt inserted in order to prevent disputes founded on nice distinctions. X x x For example, if the expression 'perils of the seas' is given its widest sense the general words have little or no effect as applied to that case. If on the other hand that expression is to receive a limited construction, as apparently it did in Cullen vs. Butler (5 M. & S., 461), and loss by perils of the seas is to be confined to loss ex marine tempestatis discrimine, the general words become most important. X x x" (Thames and Mersey Marine Insurance Co. vs. Hamilton, Fraser & Co.) - a loss which, in the ordinary course of events, results from the natural and inevitable action of the sea, from the ordinary wear and tear of the ship, or from the negligent failure of the ship's owner to provide the vessel with proper equipment to convey the cargo under ordinary conditions, is not a peril of the sea. Such a loss is rather due to what has been aptly called the "peril of the ship." The insurer undertakes to insure against perils of the sea and similar perils, not against perils of the ship. There must, in order to make the insurer liable, be "some casualty, something which could not be foreseen as one of the necessary incidents of the adventure. The purpose of the policy is to secure an indemnity against accidents which may happen, not against events which must happen." (Wilson, Sons & Co. vs. Owners of Cargo per the Xantho) - In the present case the entrance of the sea water into the ship's hold through the defective pipe already described was not due to any accident which happened during the voyage, but to the failure of the ship's owner properly to repair a defect of the existence of which he was apprised. The loss was therefore more analogous to that which directly results from simple unseaworthiness than to that which results from perils of the sea. - there is no room to doubt the liability of the shipowner for such a loss as occurred in this case. By parity of reasoning the insurer is not liable; for, generally speaking, the shipowner excepts the perils of the sea from his engagement under the bill of lading, while this is the very peril against which the insurer intends to give protection. As applied to the present case it results that the owners of the damaged rice must look to the shipowner for redress and not to the insurer. The same conclusion must be reached if the question be discussed with reference to the seaworthiness of the ship. It is universally accepted that in every contract of insurance upon Page 89 of 154

INSURANCE G01 CASE DIGESTS anything which is the subject of marine insurance, a warranty is implied that the ship shall be seaworthy at the time of the inception of the voyage. This rule is accepted in our own Insurance Law (Act No. 2427, sec. 106). It is also well settled that a ship which is seaworthy for the purpose of insurance upon the ship may yet be unseaworthy for the purpose of insurance upon the cargo (Act No. 2427, sec. 106). Disposition Decision of trial court is affirmed Petitioner filed a claim for said loss against respondent insurance company. Respondent insurance company rejected the claim alleging that assuming that spillage took place while the goods were in transit, petitioner and his agent failed to avert or minimize the loss by failing to recover spillage from the sea van, thus violating the terms of the insurance policy sued upon; and that assuming that the spillage did not occur while the cargo was in transit, the said 400 bags were loaded in bad order, and that in any case, the van did not carry any evidence of spillage. Hence, petitioner filed the complaint in the RTC of Manila against respondent insurance company seeking payment. In its answer, respondent insurance company denied all the material allegations of the complaint and raised several special defenses as well as a compulsory counterclaim. Respondent insurance company filed a third-party complaint against respondents Ben Lines and broker. Respondent broker filed its answer to the third-party complaint denying liability and arguing, among others, that the petitioner has no valid cause of action against it. Similarly, Ben Lines filed its answer denying any liability and a special defense arguing that respondent insurance company was not the proper party in interest and has no connection whatsoever with Ben Lines Containers, Ltd. and that the third-party complaint has prescribed under the applicable provisions of the Carriage of Goods by Sea Act. Respondent Ben Lines filed a motion for preliminary hearing on the affirmative defense of prescription. The trial court deferred resolution of the aforesaid motion after trial on the ground that the defense of prescription did not appear to be indubitable. After the pre-trial conference and trial on the merits, on March 31, 1986, the court a quo rendered a judgment Page 90 of 154

120.

CHOA TIEK SENG V. COURT OF APPEALS, 183 SCRA 223 (1990)

Choa Tiek Seng v Court of Appeals, 183 SCRA 223 (1990) Facts: Petitioner imported some lactose crystals from Holland. The importation involved fifteen (15) metric tons packed in 600 6-ply paper bags with polythelene inner bags, each bag at 25 kilos net. The goods were loaded at the port at Rotterdam in sea vans on board the vessel "MS Benalder' as the mother vessel, and thereafter aboard the feeder vessel "Wesser Broker V-25" of respondent Ben Lines Container, Ltd. (Ben Lines for short). The goods were insured by the respondent Filipino Merchants' Insurance Co., Inc. (insurance company for short) against all risks under the terms of the insurance cargo policy. Upon arrival at the port of Manila, the cargo was discharged into the custody of the arrastre operator respondent E. Razon, Inc. (broker for short), prior to the delivery to petitioner through his broker. Of the 600 bags delivered to petitioner, 403 were in bad order. The surveys showed that the bad order bags suffered spillage and loss.

INSURANCE G01 CASE DIGESTS dismissing the complaint, the counterclaim and the thirdparty complaint with costs against the petitioner. Hence, the appeal to the Court of Appeals by petitioner which, in due course, as aforestated, affirmed the judgment of the trial court. A motion for reconsideration of said judgment was denied by the appellate court in a resolution. Petitioner now filed this petition for review on certiorari in this Court. every specie of damage done to the ship or goods at sea by the violent action of the winds or waves. They do not embrace all loses happening on the sea. A peril whose only connection with the sea is that it arises aboard ship is not necessarily a peril of the sea; the peril must be of the sea and not merely one accruing on the sea. Moreover, the cargo in question was insured in an "against all risk policy." Insurance "against all risk" has a technical meaning in marine insurance. Under an "all risk" marine policy, there must be a general rule be a fortuitous event in order to impose liability on the insurer; losses occasioned by ordinary circumstances or wear and tear are not covered, thus, while an "all risk" marine policy purports to cover losses from casualties at sea, it does not cover losses occasioned by the ordinary circumstances of a voyage, but only those resulting from extra and fortuitous events. It has been held that damage to a cargo by high seas and other weather is not covered by an "all risk" marine policy, since it is not fortuitous, particularly where the bad weather occurs at a place where it could be expected at the time in question. The Court disagrees. In Gloren Inc. vs. Filipinas Cia. de Seguros, it was held that an all risk insurance policy insures against all causes of conceivable loss or damage, except as otherwise excluded in the policy or due to fraud or intentional misconduct on the part of the insured. It covers all losses during the voyage whether arising from a marine peril or not, including pilferage losses during the war. In the present case, the "all risks" clause of the policy sued upon reads as follows: 5. This insurance is against all risks of loss or damage to the subject matter insured but shall in Page 91 of 154

ISSUES: WON court erred in holding that an "all risks" coverage covers only losses occasioned by or resulting from "extra and fortuitous events" despite the clear and unequivocal definition of the term made and contained in the policy sued upon. Held: YES. The decision appealed from is hereby REVERSED AND SET ASIDE. Respondent Filipinas Merchants Insurance Company, Inc. is liable. There is no question that the 403 bags in damaged condition delivered and received by petitioner. Nevertheless, on the assumption that the cargo suffered damages, the appellate court ruled: Even assuming that the cargo indeed sustained damage, still the appellant cannot hold the appellee insurance company liable on the insurance policy. In the case at bar, appellant failed to prove that the alleged damage was due to risks connected with navigation. A distinction should be made between "perils of the sea" which render the insurer liable on account of the loss and/or damage brought about thereof and "perils of the ship" which do not render the insurer liable for any loss or damage. Perils of the sea or perils of navigation embrace all kinds of marine casualties, such as shipwreck, foundering, stranding, collision and

INSURANCE G01 CASE DIGESTS no case be deemed to extend to cover loss, damage, or expense proximately caused by delay or inherent vice or nature of the subject matter insured. Claims recoverable hereunder shall be payable irrespective of percentage. The terms of the policy are so clear and require no interpretation. The insurance policy covers all loss or damage to the cargo except those caused by delay or inherent vice or nature of the cargo insured. It is the duty of the respondent insurance company to establish that said loss or damage falls within the exceptions provided for by law, otherwise it is liable therefor. An "all risks" provision of a marine policy creates a special type of insurance which extends coverage to risks not usually contemplated and avoids putting upon the insured the burden of establishing that the loss was due to peril falling within the policy's coverage. The insurer can avoid coverage upon demonstrating that a specific provision expressly excludes the loss from coverage. In this case, the damage caused to the cargo has not been attributed to any of the exceptions provided for nor is there any pretension to this effect. Thus, the liability of respondent insurance company is clear. 121. FILIPINO MERCHANTS INS. V. COURT OF APPEALS, 179 SCRA 638 (1989) Marine Insurance Filipino Merchants Insurance Co., Inc v Court of Appeals FACTS: 1. The consignee of the shipment of fishmeal loaded on board the vessel SS Bougainville and unloaded at the Port of Manila on or about December 11, 1976 and seeks to recover from the defendant insurance company the amount of P51,568.62 representing damages to the said shipment.

2. The defendant brought a party against third party defendants Compagnie Maritime Des Chargues Reunis and/or E. Razon Inc seeking judgment against the third defendants. 3. Evidence shows that the plaintiff insured said shipment with the defendant insurance company under the said cargo Policy No. M2678 for the sum of P267,653.59 for the 600 metric tons of fishmeal I new gunny bags of 90 kilos each from Thailand to Manila against all risks under warehouse to warehouse terms. What was delivered was only 59.940 metric tons not 600 tons at $395.42 a ton. 4. The fishmeal in 666 new gunny bags were unloaded from the ship on December 11, 1976 at Manila unto the arrastre contractor E. Razon, Inc. and defendant's surveyor ascertained and certified that in such discharge 105 bags were in bad order condition as Page 92 of 154

INSURANCE G01 CASE DIGESTS jointly surveyed by the ship's agent and the arrastre contractor. The condition of the bad order was proven in the survey report. 5. The cargo was also surveyed by the arrastre contractor before the delivery and a total of 227 bags were in bad order condition. 6. The Filipino Merchants Insurance Company refused to pay the claim when the consignee made a formal claim which amounted to P51,568.62. 7. RTC: In favor of the consignee. Defendant was ordered to pay P51,568.62. 8. CA: Affirmed the decision of the RTC ISSUE: Whether or not the insurer is liable under the "all risks policy''? HELD: RATIONALE: A marine insurance policy providing that the insurance was to be "against all risks" must be construed as creating a special insurance and extending to other risks than are usually contemplated, and covers all losses except such as arise from the fraud of the insured. The burden of the insured, therefore, is to prove merely that the goods he transported have been lost, destroyed or deteriorated. Thereafter, the burden is shifted to the insurer to prove that the loss was due to excepted perils. To impose on the insured the burden of proving the precise cause of the loss or damage would be inconsistent with the broad protective purpose of "all risks" insurance. In the present case, there being no showing that the loss was caused by any of the excepted perils, the insurer is liable under the policy. There is no evidence presented to show that the condition of the gunny bags in which the fishmeal was packed was such that they could not hold their contents in the course of the necessary transit, much less any evidence that the bags of cargo had burst as the result of the weakness of the bags themselves. Had there been such a showing that spillage would have been a certainty, there may have been good reason to plead that there was no risk covered by the policy. OTHER TOPIC: INSURABLE INTEREST Anent the issue of insurable interest, we uphold the ruling of the respondent court that private respondent, as consignee of the goods in transit under an invoice containing the terms under "C & F Manila," has insurable interest in said goods.

Page 93 of 154

INSURANCE G01 CASE DIGESTS 2. Petitioner states in its complainants that Philipp Brothers "was the charterer of the vessel MV 'Liliana Dimitrova' which transported the shipment from Yuzhny USSR to Manila." 3. Petitioner further alleged that the caking and hardening, wetting and melting, and contamination by rust and dirt of the damaged portions of the shipment were due to the improper ventilation and inadequate storage facilities of the vessel; that the wetting of the cargo was attributable to the failure of the crew to close the hatches before and when it rained while the shipment was being unloaded in the Port of Manila 4. Moreover, in its Opposition to the Motion to Dismiss, petitioner said that "[t]he cause of action of the complaint arose from breach of contract of carriage by the vessel that was chartered by defendant Philipp Brothers 5. American jurisprudence defines charter party as a contract by which an entire ship or some principal part thereof is let by the owner to another person for a specified time or use. 6. Charter or charter parties are of two kinds. Charter of demise or bareboat and contracts of affreightment. 7. Under the demise or bareboat charter of the vessel, the charterer will generally be considered as owner for the voyage or service stipulated. The charterer mans the vessel with his own people and becomes, in effect, the owner pro hac vice, subject to liability to others for damages caused by negligence. 8. To create a demise the owner of a vessel must completely and exclusively relinquish possession, anything short of such a complete transfer is a contract of affreightment (time or voyage charter party) or not a charter party at all. 9. A contract of affreightment is in which the owner of the vessel leases part or all of its space to haul goods for others. It is a contract for a special service to be rendered by the owner of the vessel and under such contract the general Page 94 of 154

122.

PUROMINES V. COURT OF APPEALS, 220 SCRA 281 (1993)]

Puromines vs Court of Appeals Facts: 1. Puromines, Inc. (Puromines for brevity) and Makati Agro Trading, Inc. (not a party in this case) entered into a contract with private respondents Philipp Brothers Oceanic, Inc. for the sale of prilled Urea in bulk. 2. On or about May 22, 1988, the vessel M/V "Liliana Dimitrova" loaded on board at Yuzhny, USSR a shipment of 15,500 metric tons prilled Urea in bulk complete and in good order and condition for transport to Iloilo and Manila, to be delivered to petitioner. 3. 3 Bill of Ladings were issued. The shipment covered by Bill of Lading no. 2 was discharged in Iloilo safely, while those covered by Bill of Ladings Nos. 1 and 3 arrived in Manila in bad condition. 4. Damages were valued at P683, 056. 29 including additional discharging expenses. 5. Petitioner filed a complaint for breach of contract of carriage against Maritime Factors as ship-agent and owner of the vessel, and against Philipp Brothers as charterer of the vessel. 6. Philipp Brothers filed a motion to dismiss on the ground that the complaint stated no cause of action against them. Issue: 1. Whether or not petitioner has a cause of action against Philipp Brothers Held: 1. NO RULING. There is a valid arbitration clause. Court ordered for arbitration as to the issue.

INSURANCE G01 CASE DIGESTS owner retains the possession, command and navigation of the ship, the charterer or freighter merely having use of the space in the vessel in return for his payment of the charter hire. If the charter is a contract of affreightment, which leaves the general owner in possession of the ship as owner for the voyage, the rights, responsibilities of ownership rest on the owner and the charterer is usually free from liability to third persons in respect of the ship. If possession is transferred to the charterer by virtue of a demise, the charterer, and not the owner, is liable as carrier on the contract of affreightment made by himself or by the master with third persons, and is answerable for loss, damage or non-delivery of goods received for transportation. An owner who retains possession of the ship, though the hold is the property of the charterer, remains liable as carrier and must answer for any breach of duty as to the care, loading or unloading of the cargo. Assuming that in the present case, the charter party is a demise or bareboat charter, then Philipp Brothers is liable to Puromines, Inc., subject to the terms and conditions of the sales contract. On the other hand, if the contract between respondent and the owner of the vessel MV "Liliana Dimitrova" was merely that of affreightment, then it cannot be held liable for the damages caused by the breach of contract of carriage, the evidence of which is the bills of lading 123. CALTEX (PHILS.) INC. V. SULPICIO LINES, 315 SCRA 709 (1999)

Caltex vs. Sulpicio Lines (1999) Facts: 1. Dec. 19, 1987; 8:00 pm: A motor tanker MT Vector (owned and operated by Vector Shipping Corporation) carried 8,800 barrels of petroleum products of Caltex by virtue of a charter contract. From Bataan, it headed to Masbate. 2. Dec. 20, 1987 6:30 am: MV Doa Paz passenger and cargo vessel owned and operated by Sulpicio Lines, left the port of Tacloban headed for Manila with 1,493 passengers indicated in the Coast Guard Clearance. 3. Dec. 20, 1987: MT Vector collided with MV Doa Paz in the open sea between Marinduque and Oriental Mindoro, killing almost all the passengers and crew members of both ships. Only 24 survived. 4. MV Doa Paz carried an estimated 4,000 passengers most were not in the passenger manifest. 5. The Board of Marine found that the Vector Shipping Corporation was at fault and responsible for the collision with MV Doa Paz. 6. Teresita Caezal and Sotera Caezal, Sebastian Caezals wife and mother filed a complaint for Damages Arising from Breach of Contract of Carriage against Sulpicio Lines for the death of Sebastian Caezal and his 11-year old daughter Corazon Caezal. 7. Sulpicio Lines filed a 3rd party complaint against Vector Shipping Corporation and Caltex. It alleged that Caltex chartered MT Vector with gross and evident bad faith knowing fully well that MT Vector was improperly manned, ill-equipped, unseaworthy and a hazard to safe navigation. 8. The RTC dismissed the 3rd party complaint against Caltex. The CA modified the ruling by including Caltex as one of those liable for damages. Page 95 of 154

10.

11.

12.

13.

INSURANCE G01 CASE DIGESTS Issue: W/N Caltex is liable for damages for the collision between Vector and Doa Paz Held: NO. Caltex and Vector entered into a contract of affreightment, also known as a voyage charter. A charter party is a contract by which an entire ship, or some principal part thereof, is let by the owner to another person for a specified time or use; a contract of affreightment is one by which the owner of a ship or other vessel lets the whole or part of her to a merchant or other person for the conveyance of goods, on a particular voyage, in consideration of the payment of freight. A contract of affreightment may be either time charter, wherein the leased vessel is leased to the charterer for a fixed period of time, or voyage charter, wherein the ship is leased for a single voyage. In both cases, the charter-party provides for the hire of the vessel only, either for a determinate period of time or for a single or consecutive voyage, the ship owner to supply the ships store, pay for the wages of the master of the crew, and defray the expenses for the maintenance of the ship. Under a demise or bareboat charter on the other hand, the charterer mans the vessel with his own people and becomes, in effect, the owner for the voyage or service stipulated, subject to liability for damages caused by negligence. If the charter is a contract of affreightment, which leaves the general owner in possession of the ship as owner for the voyage, the rights and the responsibilities of ownership rest on the owner. The charterer is free from liability to third persons in respect of the ship. Caltex is the charterer. The charterer has no liability for damages under Philippine Maritime laws. MT Vector is a common carrier. Charter parties fall into three main categories: (1) Demise or bareboat, (2) time charter, (3) voyage charter. Does a charter party agreement turn the common carrier into a private one? We need to answer this question in order to shed light on the responsibilities of the parties. In this case, the charter party agreement did not convert the common carrier into a private carrier. The parties entered into a voyage charter, which retains the character of the vessel as a common carrier.

Is Caltex liable for damages under the Civil Code? We rule that it is not. Sulpicio argues that Caltex negligently shipped its highly combustible fuel cargo aboard an unseaworthy vessel such as the MT Vector. The charterer of a vessel has no obligation before transporting its cargo to ensure that the vessel it chartered complied with all legal requirements. The duty rests upon the common carrier simply for being engaged in public service.

The relationship between the parties in this case is governed by special laws. Because of the implied warranty of seaworthiness, shippers of goods, when transacting with common carriers, are not expected to inquire into the vessels seaworthiness, genuineness of its licenses and compliance with all maritime laws. To demand more from shippers and hold them liable in case of failure exhibits nothing but the futility of our maritime laws insofar as the protection of the public in general is concerned. By the same token, we cannot expect passengers to inquire every time they board a common carrier, whether the carrier possesses the necessary papers or that all the carriers employees are qualified. Such a practice would be an absurdity in a business where time is always of Page 96 of 154

INSURANCE G01 CASE DIGESTS the essence. Considering the nature of transportation business, passengers and shippers alike customarily presume that common carriers possess all the legal requisites in its operation.

124.

SAN MIGUEL V. HEIRS OF INGUITO, 384 SCRA 87 (2002)

San Miguel vs Heirs of Inguito GR No. 141716 July 4, 2002 Thus, the nature of the obligation of Caltex demands ordinary diligence like any other shipper in shipping his cargoes. A cursory reading of the records convinces us that Caltex had reasons to believe that MT Vector could legally transport cargo that time of the year. Topic: Marine Insurance - Insurable Interest Facts: San Miguel (SMC) entered into a Time Charter Party Agreement with Julius Ouano, doing business under the name and style J. Ouano Marine Services. SMC chartered MV Dona Roberta, owned by Ouano, for a period of 2 years, for the purpose of transporting SMC's beverage products from Mandaue City plant to various points in Visayas and Mindanao. The agreement stated that: o Ouano warrants ownership, title and interest over the vessel o that the vessel is put at SMC's disposal; SMC hires the use and service of the vessel o that the owner warrants that the vessel is seaworthy and in proper, useful and operational condition; and that SMC should immediately notify Ouano in case it finds any defect in the vessel o that there is no employer-employee relationship between Ouano (and/or the crew of the vessel) and SMC; Ouano held SMC free from all claims and liabilities arising out of the acts of the crew and the condition of the vessel o that Ouano undertook to pay all compensation of all the vessel's crew o that Ouano shall indemnify SMC for damages and losses arising from the incompetence and/or negligence of, and/or the failure to observe the Page 97 of 154

Caltex and Vector Shipping Corporation had been doing business since 1985, or for about two years before the tragic incident occurred in 1987. Past services rendered showed no reason for Caltex to observe a higher degree of diligence.

Clearly, as a mere voyage charterer, Caltex had the right to presume that the ship was seaworthy as even the Philippine Coast Guard itself was convinced of its seaworthiness. All things considered, we find no legal basis to hold petitioner liable for damages.

INSURANCE G01 CASE DIGESTS required extra-ordinary diligence by the crew; Ouano would automatically be liable for shortlanded shipment, where the value shall be withheld from Ouano's collectibles, and wrong levels, where the value shall likewise be withheld, but SMC shall reimburse Ouano if SMC's laboratory shall make a determination that the bottles were never opened after it left the plant SMC issued sailing orders to the master of the vessel, Captain Inguito, stating where to sail to, when it is expected to depart and arrive, to maintain communications, and to observe weather condition (exercise utmost precautionary measures) Inguito obtained the necessary sailing clearance from the Philippine Coast Guard. The loading was completed on schedule, but the vessel did not leave in accordance with the orders. November 12, 1990: o 4am - a typhoon was spotted near Samar moving towards the general direction of Eastern Visayas o 6am - the vessel left Mandaue o 7am - while still abeam Cawit Island off Cebu, SMC Radio Operator Moreno contacted Inguito and advised him to take shelter; Inguito replied that they would proceed since the typhoon was far away from them and that the winds were in their favor o 4pm - Moreno reiterated to Inguito the advice and pointed out that it would be difficult to take shelter after passing Balicasag Island because they were approaching open sea; Inguito refused to heed o 8pm - vessel was 38 miles southeast of Balicasag o 10pm - vessel was 25 miles approaching Sulauan Point; moments later power went out in Moreno's office 11:40pm - power resumed, Moreno made a series of calls to the vessel but he failed to get in touch with anyone November 13, 1990: o 1:15am - Inguito called Moreno over the radio and requested that he contact Rico Ouano, son of the owner, because they needed a helicopter to rescue them; vessel was 20 miles west of Sulauan Point. Inguiti requested for a helicopter to rescue them; the Chief Engineer informed Rico that they can no longer stop the water from coming into the vessel because the crew members were feeling dizzy from the petroleum fumes o 2:30am - the vessel sank; only 5 of the 25 officers and crew on board survived Ouano, in lieu of the Inguito (who died in the sea tragedy), filed a Marine Protest the heirs of Inguito filed a complaint for tort against SMC and Ouano with the RTC of Lapu-Lapu City Ouano filed a cross-claim: the proximate cause of the loss of the vessel and of its officers and crew was the fault and negligence of SMC, which had complete control and disposal of the vessel as charterer and which issued a sailing order despite being forewarned of the typhoon; he prayed for indemnification of the cost of the vessel and unrealized rentals and earning SMC: it was Ouano who had control, supervision and responsibilities over the navigation of the vessel; Ouano never initiated contact with the vessel despite knowledge of the typhoon; the proximate cause was Ouano's breach of his obligation to provide SMC with a seaworthy vessel duly manned by competent crew members; demanded the value of the cargo lost in sea RTC: the proximate cause was attributable to SMC; liable for the loss of earnings of those who died in the sea tragedy, Page 98 of 154 o

INSURANCE G01 CASE DIGESTS moral and exemplary damages for heirs of each deceased crew member; liable for the value of the total loss of the vessel and unrealized rental earnings SMC and Ouano appealed to the CA. CA - modified the decision: SMC and Ouano are jointly liable to the heirs of the deceased; the claims of SMC and Ouano against each other are dismissed If the charter is a contract of affreightment, which leaves the general owner in possession of the ship as owner for the voyage, the rights and the responsibilities of ownership rest on the owner. The charterer is free from liability to third persons in respect of the ship the charter between SMC and Ouano was a contract of affreighment, as evidenced by the the provisions of their agreement Ouano was the employer, and had command and control over the vessel and the crew SMC should free from liability for any loss or damage sustained during the voyage, unless it can be shown that the loss was due to its fault or negligence, which the evidence does not show (attempts of SMC's radio operator to advise Inguito to take shelter and efforts to keep in contact) Absent any showing of fault or negligence from SMC, Ouano had the clear duty to ensure the safe carriage and arrival of goods transported on board its vessels. Ouano also expressly warranted that the vessel was seaworthy. seaworthiness: the sufficiency of the vessel in materials, construction, equipment, officers, men, and outfit for the trade or service in which it is employed; includes the fitness of a ship for its fuel and provisions supply, the quality of its officers and crew, and its adaptability for the time of voyage proposed The CA found that the proximate cause of the sinking was the negligence of Inguito Ouano is vicariously liable for the negligent acts of his employee, Captain Inguito. Under Articles 2176 and 2180 of the Civil Code, owners and managers are responsible for damages caused by the negligence of a servant or an employee, the master or employer is presumed to be negligent either in the selection or in the supervision of that employee. This presumption may be overcome only by Page 99 of 154

Issue: Who should be held liable for the loss note: i don't know how this involves insurable interest >.< it only talks about how the owner shall indemnify the charterer Ruling: Ouano, the owner of the vessel, is liable to the heirs of the deceased, and to SMC for the loss of their goods a charter party is a contract by virtue of which the owner or the agent of a vessel binds himself to transport merchandise or persons for a fixed price. It has also been defined as a contract by virtue of which the owner or the agent of the vessel leases for a certain price the whole or a portion of the vessel for the transportation of goods or persons from one port to another 2 types of charter parties: (1) bareboat or demise: the charterer mans the vessel with his own people; (2) contract of affreightment: the owner of the vessel leases part or all of its space to haul goods for others. It is a contract for special service to be rendered by the owner of the vessel. Under such contract the ship owner retains the possession, command and navigation of the ship, the charterer or freighter merely having use of the space in the vessel in return for his payment of the charter hire. 2 types of contract of affreightment: time charter and voyage charter

INSURANCE G01 CASE DIGESTS satisfactorily showing that the employer exercised the care and the diligence of a good father of a family in the selection and the supervision of its employee. Ouano failed to present proof that he exercised the due diligence in the selection and supervision of the captain. Thus, he is vicariously liable for the loss of lives and property occasioned by the lack of care and negligence of his employee 125. CALTEX (PHILS.) INC. V. SULPICIO LINES, 315 SCRA 709 (1999)

Caltex vs Sulpicio Lines Facts: MV Vector and Caltex entered into a charter contract (contract of affreightment). On December 20, 1987, motor tanker MV Vector, carrying petroleum products of Caltex, collided in the open sea with passenger ship MV Doa Paz, causing the death of all but 25 of the latters passengers. Among those who died were Sebastian Canezal and his daughter Corazon Canezal. On March 22, 1988, the board of marine inquiry found that Vector Shipping Corporation was at fault. On February 13, 1989, Teresita Caezal and Sotera E. Caezal, Sebastian Caezals wife and mother respectively, filed with the Regional Trial Court of Manila a complaint for damages arising from breach of contract of carriage against Sulpicio Lines. Sulpicio filed a third-party complaint against Vector and Caltex. The trial court dismissed the complaint against Caltex, but the Court of Appeals included the same in the liability. Hence, Caltex filed this petition.

**as for insurable interest, Section 100, 103, 105 and 106 provide for who has an insurable interest in this case Section 100. The owner of a ship has in all cases an insurable interest in it, even when it has been chartered by one who covenants to pay him its value in case of loss; Provided, That in this case the insurer shall be liable for only that part of the loss which the insured cannot recover from the charterer Sec. 103. The owner of a ship has an insurable interest in expected freightage which according to the ordinary and probable course of things he would have earned but for the intervention of a peril insured against or other peril incident to the voyage. (freightage: the benefit which is to accrue to the owner from the use of the vessel in the voyage contemplated, ir the benefit derived from the employment of the ship) Sec. 105. One who has an interest in the thing from which profits are expected to proceed has an insurable interest in the profits. Sec. 106. The charterer of a ship has an insurable interest in it, to the extent that he is liable to be damnified by its loss.

Issue: Whether or not Caltex, as the charterer of a sea vessel, is liable for damages resulting from a collision between the chartered vessel and a passenger ship?

Ruling: Page 100 of 154

INSURANCE G01 CASE DIGESTS The Supreme Court ruled in the negative. The Supreme Court ruled that MT Vector is a common carrier. The charter party agreement did not convert the common carrier into a private carrier. The parties entered into a voyage charter, which retains the character of the vessel as a common carrier. It is imperative that a public carrier shall remain as such, notwithstanding the charter of the whole or portion of a vessel by one or more persons, provided the charter is limited to the ship only, as in the case of a time-charter or voyage charter. It is only when the charter includes both the vessel and its crew, as in a bareboat or demise that a common carrier becomes private, at least insofar as the particular voyage covering the charter-party is concerned. Indubitably, a ship-owner in a time or voyage charter retains possession and control of the ship, although her holds may, for the moment, be the property of the charterer. A common carrier is a person or corporation whose regular business is to carry passengers or property for all persons who may choose to employ and to remunerate him. 16 MT Vector fits the definition of a common carrier under Article 1732 of the Civil Code. The public must of necessity rely on the care and skill of common carriers in the vigilance over the goods and safety of the passengers, especially because with the modern development of science and invention, transportation has become more rapid, more complicated and somehow more hazardous. For these reasons, a passenger or a shipper of goods is under no obligation to conduct an inspection of the ship and its crew, the carrier being obliged by law to impliedly warrant its seaworthiness. The charterer of a vessel has no obligation before transporting its cargo to ensure that the vessel it chartered complied with all legal requirements. The duty rests upon the common carrier simply for Page 101 of 154 being engaged in "public service." The relationship between the parties in this case is governed by special laws. Because of the implied warranty of seaworthiness, shippers of goods, when transacting with common carriers, are not expected to inquire into the vessels seaworthiness, genuineness of its licenses and compliance with all maritime laws. To demand more from shippers and hold them liable in case of failure exhibits nothing but the futility of our maritime laws insofar as the protection of the public in general is concerned. Such a practice would be an absurdity in a business where time is always of the essence. Considering the nature of transportation business, passengers and shippers alike customarily presume that common carriers possess all the legal requisites in its operation.

INSURANCE G01 CASE DIGESTS 126. SAN MIGUEL V. HEIRS OF INGUITO, 384 SCRA 87 (2002) SMC Radio Operator Moreno contacted Captain Inguito 3 times through the radio and advised him to take shelter but Capt. Inquito did not heed the advice and continued to sail. At 1:15 a.m., November 13, 1990, Captain Inguito called Moreno over the radio and requested him to contact Rico Ouano, son of Julius Ouano, because they needed a helicopter to rescue them. At 2:30 a.m. of November 13, 1990, the M/V Doa Roberta sank. Out of the 25 officers and crew on board the vessel, only five survived. Shipowner Julius Ouano, in lieu of the captain who perished in the sea tragedy, filed a Marine Protest. The heirs of the deceased captain and crew, as well as the survivors filed a complaint for tort against San Miguel Corporation and Julius Ouano. Julius Ouano alleged that the proximate cause of the loss of the vessel and its officers and crew was the fault and negligence of SMC which issued the sailing order for its departure despite being forewarned of the impending typhoon. SMC argued that the proximate cause of the sinking was Ouanos breach of his obligation to provide SMC with a seaworthy vessel duly manned by competent crew members. RTC --- proximate cause of the loss of the M/V Doa Roberta was attributable to SMC. Both SMC and Ouano appealed to the Court of Appeals. SMC argued that as mere charterer, it did not have control of the vessel and that the proximate cause of the loss of the vessel and its cargo Page 102 of 154

G.R. No. 141716. July 4, 2002] SAN MIGUEL CORPORATION, petitioner, vs. HEIRS OF SABINIANO INGUITO, and JULIUS OUANO, respondents. FACTS: San Miguel Corporation entered into a Time Charter Party Agreement with Julius Ouano, owner of Ouano Marine Services. Under the terms of the agreement, SMC will charter the M/V Doa Roberta owned by Julius Ouano for a period of two years, to transport SMCs beverage products from its Mandaue City plant to various points in Visayas and Mindanao. Pertinent portions of the Time Charter Party Agreement state: 1. OWNER [i.e., Ouano] warrants his ownership, title and interest over the vessel DOA ROBERTA; 4. OWNER warrants that the vessel is seaworthy; 9. There shall be no employer-employee relations between the OWNER and/or its vessels crew on one hand and the CHARTERER on the other; 10. The OWNER shall undertake to pay all compensation of all the vessels crew; 11. The OWNER shall be responsible to and shall indemnify the CHARTERER for damages and losses arising from the incompetence and/or negligence of its crew. During the term of the charter, SMC issued sailing orders to MV Doa Roberta Captain Inguito to sail on November 12, 1990. Meanwhile, at 4:00 a.m. of November 12, 1990, typhoon Ruping was spotted.

INSURANCE G01 CASE DIGESTS was the negligence of the ship captain. For his part, Ouano complained of the reduced damages awarded to him by the trial court. CA --- SMC and Ouano solidarily liable to plaintiffs heirs and survivors, except to the heirs of Capt. Sabiniano Inguito message. Neither Ouano nor his son was available during the entire time that the vessel set out and encountered foul weather. Considering that the charter was a contract of affreightment, the shipowner had the clear duty to ensure the safe carriage and arrival of goods transported on board its vessels. More specifically, Ouano expressly warranted in the Time Charter Party that his vessel was seaworthy.

ISSUE: WHETHER THE WARRANT BY THE SHIPOWNER OF THE SEAWORTHINESS OF THE M/V DOA ROBERTA WAS VIOLATED.

WHAT IT MEANS FOR A VESSEL TO BE SEAWORTHY For a vessel to be seaworthy, it must be adequately equipped for the voyage and manned with a sufficient number of competent officers and crew.[32]

HELD: YES.

CHARTER PARTY AGREEMENT MAKES SHIPOWNER LIABLE TO SMC Under the foregoing definitions, as well as the clear terms of the Charter Party Agreement between the parties, the charterer, SMC, should be free from liability for any loss or damage sustained during the voyage,[31] unless it be shown that the same was due to its fault or negligence. The evidence does not show that SMC or its employees were amiss in their duties.

DEFINITION OF SEAWORTHINESS Seaworthiness is defined as the sufficiency of the vessel in materials, construction, equipment, officers, men, and outfit, for the trade or service in which it is employed.[33] It includes the fitness of a ship for a particular voyage with reference to its physical and mechanical condition, the extent of its fuel and provisions supply, the quality of its officers and crew, and its adaptability for the time of voyage proposed.[34]

SHIPOWNER WAS NEGLIGENT In contrast to the care exercised by Moreno, Rico Ouano tried to communicate with the captain only after receiving the S.O.S.

PROXIMATE CAUSE OF THE SINKING WAS NEGLIGENCE OF CAPTAIN INGUITO It appears that the proximate cause of the sinking of the vessel was the gross failure of the captain of the vessel to observe due Page 103 of 154

INSURANCE G01 CASE DIGESTS care and to heed SMCs advices to take shelter. He was fully apprised of typhoon Ruping and its strength. Due diligence dictates that at any time before the vessel was in distress, he should have taken shelter in order to safeguard the vessel and its crew. It is very clear that Captain Sabiniano Inguito had sufficient time within which to secure his men and the vessel. But he waited until the vessel was already in distress. In fact, there was an incident when a sailing order was issued by SMC to Inguito but he decided not to proceed with the voyage because of a tropical storm.[35] previous occasion, SMC issued a sailing order to the captain of the M/V Doa Roberta, but the vessel cancelled its voyage due to typhoon.[39] Likewise, it appears from the records that SMC issued the sailing order on November 11, 1990, before typhoon Ruping was first spotted at 4:00 a.m. of November 12, 1990.[40]

ONLY SHIPOWNER OUANO IS LIABLE FOR THE LOSSES Consequently, Ouano should answer for the loss of lives and damages suffered by the heirs of the officers and crew members who perished on board the M/V Doa Roberta, except Captain Sabiniano Inguito. The award of damages granted by the Court of Appeals is affirmed only against Ouano, who should also indemnify SMC for the cost of the lost cargo, in the total amount of P10,278,542.40.[41]

OUANO VICARIOUSLY LIABLE UNDER ARTS. 2176 AND 2180 OF THE CIVIL CODE Under Articles 2176 and 2180 of the Civil Code, owners and managers are responsible for damages caused by the negligence of a servant or an employee, the master or employer is presumed to be negligent either in the selection or in the supervision of that employee. This presumption may be overcome only by satisfactorily showing that the employer exercised the care and the diligence of a good father of a family in the selection and the supervision of its employee.[38] Ouano miserably failed to overcome the presumption of his negligence. SMC CANNOT BE LIABLE FOR THE LOSSES However, we cannot sustain the appellate courts finding that SMC was likewise liable for the losses. The contention that it was the issuance of the sailing order by SMC which was the proximate cause of the sinking is untenable. The fact that there was an approaching typhoon is of no moment. It appears that on one

(NOTE: The Time Charter Agreement between SMC and Ouano in this case was a contract of of affreightment which means that the shipowner or the captain of the vessel had command and control of the vessel at all times. This is in contrast to a bareboat or demise charter wherein it is the charterer who mans the vessel with his own people and therefore had control of the vessel.)

Page 104 of 154

INSURANCE G01 CASE DIGESTS 127. PHILIPPINE AMERICAN GENERAL INSURANCE V. CA, 273 SCRA 262 (1997) G.R. No. 116940 June 11, 1997 THE PHILIPPINE AMERICAN GENERAL INSURANCE COMPANY, INC., petitioner, vs. COURT OF APPEALS and FELMAN SHIPPING LINES, respondents. Nature: This case deals with the liability, if any, of a shipowner for loss of cargo due to its failure to observe the extraordinary diligence required by Art. 1733 of the Civil Code as well as the right of the insurer to be subrogated to the rights of the insured upon payment of the insurance claim. The shipment was insured with petitioner Philippine American General Insurance Co., Inc. (PHILAMGEN for brevity), under Marine Open Policy No. 100367-PAG.

"MV Asilda" left the port of Zamboanga in fine weather at eight o'clock in the evening of the same day. At around eight forty-five the following morning, 7 July 1983, the vessel sank in the waters of Zamboanga del Norte bringing down her entire cargo with her including the subject 7,500 cases of 1-liter Coca-Cola softdrink bottles.

On 15 July 1983 the consignee Coca-Cola Bottlers Philippines, Inc., Cebu plant, filed a claim with respondent FELMAN for recovery of damages it sustained as a result of the loss of its softdrink bottles that sank with "MV Asilda." Respondent denied the claim thus prompting the consignee to file an insurance claim with PHILAMGEN which paid its claim of P755,250.00.

Facts: On 6 July 1983 Coca-Cola Bottlers Philippines, Inc., loaded on board "MV Asilda," a vessel owned and operated by respondent Felman Shipping Lines (FELMAN for brevity), 7,500 cases of 1-liter Coca-Cola softdrink bottles to be transported from Zamboanga City to Cebu City for consignee Coca-Cola Bottlers Philippines, Inc., Cebu.

Claiming its right of subrogation PHILAMGEN sought recourse against respondent FELMAN which disclaimed any liability for the loss. Consequently, on 29 November 1983 PHILAMGEN sued the shipowner for sum of money and damages.

Issue: Whether or not the limited liability under Art. 587 of the Code of Commerce should apply.

Page 105 of 154

INSURANCE G01 CASE DIGESTS Held: from 29 November 1983, the date of judicial demand,
pursuant to Arts. 2212 and 2213 of the Civil Code.

No, Art. 587 of the Code of Commerce is not applicable to the case at bar. Simply put, the ship agent is liable for the negligent acts of the captain in the care of goods loaded on the vessel. This liability however can be limited through abandonment of the vessel, its equipment and freightage as provided in Art. 587. Nonetheless, there are exceptional circumstances wherein the ship agent could still be held answerable despite the abandonment, as where the loss or injury was due to the fault of the shipowner and the captain. The international rule is to the effect that the right of abandonment of vessels, as a legal limitation of a shipowner's liability, does not apply to cases where the injury or average was occasioned by the shipowner's own fault.

128.

DELSAN TRANSPORT V. CA, 369 SCRA 24 (2001)

Delsan Transport vs. Court of Appeals G.R. No. 127897; November 15, 2001 Facts: Caltex Philippines entered into a contract of affreightment with the petitioner, Delsan Transport Lines, Inc. for a period of one year whereby the petitioner agreed to transport Caltex industrial fuel oil from Batangas refinery to different parts of the country. Under the contract, petitioner took on board its vessel, M/T Maysun industrial oil of Caltex to be delivered to the latters Oil Terminal in Zamboanga City. The shipment was insured with the private respondent, American Home Assurance Corporation On August 14, 1986, MT Maysun set sail for Zamboanga City but unfortunately the vessel in the early morning of August 16, 1986 near Panay Gulf. The shipment was insured with the private respondent, American Home Assurance Corporation. Subsequently, private respondent paid Caltex the sum of Php.5,096,635.57. Exercising its right of subrogation under Art. 2207, NCC, the private respondent demanded from the petitioner the same amount paid to Caltex. Due to its failure to collect from the petitioner, private respondent filed a complaint with the RTC of Page 106 of 154

It must be stressed at this point that Art. 587 speaks only of situations where the fault or negligence is committed solely by the captain. Where the shipowner is likewise to be blamed, Art. 587 will not apply, and such situation will be covered by the provisions of the Civil Code on common carrier.

WHEREFORE, the petition is GRANTED. Respondent FELMAN SHIPPING LINES is ordered to pay petitioner PHILIPPINE AMERICAN GENERAL INSURANCE CO., INC., Seven Hundred Fifty-five Thousand Two Hundred and Fifty Pesos (P755,250.00) plus legal interest thereon counted

INSURANCE G01 CASE DIGESTS Makati City but the trial court dismissed the complaint, finding the vessel to be seaworthy and that the incident was due to a force majeure, thus exempting the petitioner from liability. However, the decision of the trial court was reversed by the Court of Appeals, giving credence to the report of PAGASA that the weather was normal and that it was impossible for the vessel to sink. In the absence of any explanation as to what may have caused the sinking of the vessel coupled with the finding that the same was improperly manned, the appellate court ruled that the petitioner is liable on its obligation as common carrier to herein private respondent insurance company as subrogee of Caltex. foreclose recourse against the petitioner for any liability under its contractual obligation as a common carrier. The fact of payment grants the private respondent subrogatory right which enables it to exercise legal remedies that would otherwise be available to Caltex as owner of the lost cargo against the petitioner common carrier. In order to escape liability for the loss of its cargo of industrial fuel oil belonging to Caltex, petitioner attributes the sinking of MT Maysun to fortuitous even or force majeure. But the tale of strong winds and big waves by Captain and the Chief Mate of the said vessel however, was effectively rebutted and belied by the weather report from PAGASA. Thus, as the appellate court correctly ruled, petitioners vessel, MT Maysun, sank with its entire cargo for the reason that it was not seaworthy. There was no squall or bad weather or extremely poor sea condition in the vicinity when the said vessel sank. Neither may petitioner escape liability by presenting in evidence certificates that tend to show that at the time of drydocking and inspection by the Philippine Coast Guard, the vessel MT Maysun, was fit for voyage. These pieces of evidence do not necessarily take into account the actual condition of the vessel at the time of the commencement of the voyage. As correctly observed by the Court of Appeals: At the time of dry-docking and inspection, the ship may have appeared fit. The certificates issued, however, do not negate the presumption of unseaworthiness triggered by an unexplained sinking. Of certificates issued in this regard, authorities are likewise clear as to their probative value, (thus): Page 107 of 154

Issue: Whether or not the payment made by American Home Assurance to Caltex for the insured value of the lost cargo amounted to an admission that the vessel was seaworthy, thus precluding any action of recovery against Delsan Transport?

Held: NO. Instant Petition is DENIED. Ratio: The payment made by the private respondent for the insured value of the lost cargo operates as waiver of its (private respondent) right to enforce the term of the implied warranty against Caltex under the marine insurance policy. However, the same cannot be validly interpreted as an automatic admission of the vessels seaworthiness by the private respondent as to

INSURANCE G01 CASE DIGESTS failure to rebut the presumption of fault or negligence as common carrier occasioned by the unexplained sinking of its vessel, MT Maysun, while in transit.

Seaworthiness relates to a vessels actual condition. Neither the granting of classification or the issuance of certificates established seaworthiness. (2-A Benedict on Admiralty, 7-3, Sec. 62). And also: Authorities are clear that diligence in securing certificates of seaworthiness does not satisfy the vessel owners obligation. Also securing the approval of the shipper of the cargo, or his surveyor, of the condition of the vessel or her stowage does not establish due diligence if the vessel was in fact unseaworthy, for the cargo owner has no obligation in relation to seaworthiness. (Ibid.) Additionally, the exoneration of MT Maysuns officers and crew by the Board of Marine Inquiry merely concerns their respective administrative liabilities. It does not in any way operate to absolve the petitioner common carrier from its civil liabilities. It does not in any way operate to absolve the petitioner common carrier from its civil liability arising from its failure to observe extraordinary diligence in the vigilance over the goods it was transporting and for the negligent acts or omissions of its employees, the determination of which properly belongs to the courts.

129. ORIENTAL ASSURANCE V. COURT OF APPEALS, 200 SCRA 459 (1991) Oriental Assurance vs. Court of Appeals G.R. No. 94052 August 9, 1991 FACTS:

1. Panama Sawmill shipped 1208 pieces of apitog logs to Manila and insured the logs with Oriental for the value of Php 1 million. Two barges were loaded with 610 and 598 logs. At sea, typhoons ravaged one of the barges, resulting in the loss of 497 of 598 of the logs. 2. The Insurance contract provided for indemnity under the following conditions: Warranted that this Insurance is against TOTAL LOSS ONLY. Subject to the following clauses: Civil Code Article 1250 Waiver clause Typhoon warranty clause

In the case at bar, petitioner is liable for the insured value of the lost cargo of industrial fuel oil belonging to Caltex for its

Omnibus clause.

Page 108 of 154

INSURANCE G01 CASE DIGESTS 3. Oriental didnt give an indemnity because there wasnt total loss of the shipment. 4. The sawmill filed a civil case against Oriental and the court ordered it to pay 410,000 as value for the missing logs. The CA affirmed the lower court judgment but reduced the legal interest. Hence this appeal by Oriental. Also, the insurer's liability was for "total loss only" as stipulated. A total loss may be either actual or constructive. An actual total loss under Sec 130 of the Insurance Code is caused by: (a) A total destruction of the thing insured; (b) The irretrievable loss of the thing by sinking, or by being broken up; (c) Any damage to the thing which renders it valueless to the owner for the purpose for which he held it; or (d) Any other event which effectively deprives the owner of the possession, at the port of destination, of the thing insured.

ISSUE: WON Oriental Assurance can be held liable under its marine insurance policy based on the theory of a divisible contract of insurance and, consequently, a constructive total loss? NO. HELD: Perla v CA- The terms of the contract constitute the measure of the insurer liability and compliance therewith is a condition precedent to the insured's right to recovery from the insurer. Whether a contract is entire or severable is a question of intention to be determined by the language employed by the parties. The policy in question shows that the subject matter insured was the entire shipment of 2,000 cubic meters of apitong logs. The fact that the logs were loaded on two different barges did not make the contract several and divisible as to the items insured. The logs on the two barges were not separately valued or separately insured. Only one premium was paid for the entire shipment, making for only one cause or consideration. The insurance contract must, therefore, be considered indivisible.

A constructive total loss, gives to a person insured a right to abandon and it means: SECTION 139. A person insured by a contract of marine insurance may abandon the thing insured, or any particular portion thereof separately valued by the policy, or otherwise separately insured, and recover for a total loss thereof, when the cause of the loss is a peril injured against, (a) If more than three-fourths thereof in value is actually lost, or would have to be expended to recover it from the peril; (b) If it is injured to such an extent as to reduce its value more than three-fourths

Page 109 of 154

INSURANCE G01 CASE DIGESTS The appellate court considered the cargo in one barge as separate from the other and ruled that 497 of 598 was more than of the amount lost, showing a constructive total loss. 130. PHILIPPINE HOME ASSURANCE V. COURT OF APPEALS, 257 SCRA 468 (1996) G.R. No. 106999, June 26, 1996 FACTS: The SC, however, said that although the logs were placed in two barges, they were not separately valued by the policy, nor separately insured. Of the entirety of 1,208, pieces of logs, only 497 pieces thereof were lost or 41.45% of the entire shipment. Since the cost of those 497 pieces does not exceed 75% of the value of all 1,208 pieces of logs, the shipment can not be said to have sustained a constructive total loss under Section 139(a) of the Insurance Code. Eastern Shipping Lines, Inc. (ESLI) loaded on board SS Eastern Explorer in Japan shipment of carriage to Mania and Cebu consigned to the ff: (1) William Lines, Inc. - engine parts; (2) Orca's Company - ammonium chloride; (3) Pan Oriental Match Company - bags of Glue; (4) Ding Velayo garments. While the vessel was off Okinawa, a small flame was detected on the acetylene cylinder located in the accommodation area near the engine room on the main deck level. The acetylene cylinder suddenly exploded causing death and severe injuries to the crew and instantly setting fire to the whole vessel, which forced the crew to abandon the ship. The cargoes which were saved were loaded to another vessel for delivery to their original ports of destination. ESLI charged the consignees several amounts corresponding to additional freight and salvage charges. The charges were all paid by Philippine Home Assurance Corporation (PHAC) under protest for and in behalf of the consignees. Thereafter, as subrogee of the consignees, filed a complaint against ELSI to recover the sum paid under protest on the ground that the same were actually damage directly brought by the fault and negligence of ESLI. RTC & CA: Dismissed the complaint and ruled in favor of ESLI.

ISSUE: Page 110 of 154

INSURANCE G01 CASE DIGESTS (1) Who among the carrier, consignee or insurer of the goods is liable for the additional charges or expenses incurred? Carrier. (2)Whether or not the respondent court committed an error in concluding that the expenses incurred in saving the cargo are considered general average? Yes. Consequently, respondent ESLI's claim for the contribution from the consignees at the time of the occurrence of the average turns to naught. The cargo consignees cannot be made liable to respondent carrier thus respondent carrier must refund petitioner the amount it paid under protest for additional freight and salvage charges in behalf of the consignees.

RATIO: (1) Fire may not be considered a natural disaster or calamity since it almost always arise from some act of men. In this case, it is not disputed that a small flame was detected on the acetylene cylinder and that by reason thereof the same exploded despite efforts to extinguish the fire. Moreover, the acetylene cylinder, obviously full loaded, was stored in the accommodation area near the engine room and not in a storage area considerably far and safe distance from the engine room. Therefore, there is a strong evidence that the acetylene cylinder caught fire because of the fault and negligence of respondent ESLI, its captain and its crew.

(2) As a rule, general or gross averages include all damages and expenses which are deliberately caused in order to save the vessel, its cargo, or both at the same time, from a real and known risk. While this case may technically fall within the purview of the said provision, the formalities prescribed under Art. 813 and 814 of the Code of Commerce in order to incur the expenses and cause the damage corresponding to gross average were not complied with. Page 111 of 154

INSURANCE G01 CASE DIGESTS

FIRE INSURANCE
131. PHILIPPINE HOME ASSURANCE V. COURT OF APPEALS, 257 SCRA 468 (1996) FIRE INSURANCE Phil Home Assurance v. CA FACTS ESLI (respondent) loaded to a ship certain goods like engine parts, mamonium chloride, and glue 300. . While the vessel was off Okinawa, Japan, a small flame was detected on the acetylene cylinder located in the accommodation area near the engine room on the main deck level. As the crew was trying to extinguish the fire, the acetylene cylinder suddenly exploded sending a flash of flame throughout the accommodation area, thus causing death and severe injuries to the crew and instantly setting fire to the whole superstructure of the vessel. The incident forced the master and the crew to abandon the ship. After the fire was extinguished, the cargoes which were saved were loaded to another vessel for delivery to their original ports of destination. ESLI charged the consignees additional charges or expenses incurred by the owner of the ship in the salvage operations and in the transshipment of the goods via a different carrier.

Phil Home paid under protest, and as subrogee of the consignee, filed a complaint to recover the sum paid under protest on the ground that the damages were the fault, negligence of ESLI ISSUE WON the burning of the ship is a natural disaster or calamity which would absolve ESLI from liability? NO

HELD In our jurisprudence, fire may not be considered a natural disaster or calamity since it almost always arises from some act of man or by human means. It cannot be an act of God unless caused by lightning or a natural disaster or casualty not attributable to human agency. In the case at bar, it is not disputed that a small flame was detected on the acetylene cylinder and that by reason thereof, the same exploded despite efforts to extinguish the fire. Neither is there any doubt that the acetylene cylinder, obviously fully loaded, was stored in the accommodation area near the engine room and not in a storage area considerably far, and in a safe distance, from the engine room. Moreover, there was no showing, and none was alleged by the parties, that the fire was caused by a natural disaster or calamity not attributable to human agency. On the contrary, there is strong evidence indicating that the acetylene cylinder caught fire because of the fault and negligence of respondent ESLI, its captain and its crew. Page 112 of 154

INSURANCE G01 CASE DIGESTS 132. BACHRACH V. BRITISH AMERICAN ASSURANCE , 17 PHIL. 35 (1910) E. M. BACHRACH vs. BRITISH AMERICAN ASSURANCE COMPANY G.R. No. L-5715 December 20, 1910 JOHNSON, J.: FACTS: 1. E. M. Bachrach insured against loss or damage by fire, goods, belonging to its general furniture store, such as iron and brass bedsteads, toilet tables, chairs, ice boxes, bureaus, washstands, mirrors, and sea-grass furniture from British American Assurance Company through it's agent. 2. In the policy, Co insurance was allowed which should be declared in the event of loss or claim. Also, such policy provided Permission granted for the use of gasoline not to exceed 10 gallons for the "Calalac" automobile, but only whilst contained in the reservoir of the car. 3. Fire broke out. British American Assurance Company denied the claim of Bachrach alleging the ff: a. That the plaintiff maintained a paint and varnish shop in the said building where the goods which were insured were stored. b. That the plaintiff transferred his interest in and to the property covered by the policy to H. W. Peabody & Co. to secure certain indebtedness due and owing to said company, and also that the plaintiff had transferred his interest in certain of the goods covered by the said policy to one Macke, to secure certain obligations assumed by the Page 113 of 154

the fact that the acetylene cylinder was checked, tested and examined and subsequently certified as having complied with the safety measures and standards by qualified experts before it was loaded in the vessel only shows to a great extent that negligence was present in the handling of the acetylene cylinder after it was loaded and while it was on board the ship. Indeed, had the respondent and its agents not been negligent in storing the acetylene cylinder near the engine room, then the same would not have leaked and exploded during the voyage There is no merit in the finding of the trial court to which respondent court erroneously agreed that the fire was not the fault or negligence of respondent but a natural disaster or calamity. The records are simply wanting in this regard.

WHEREFORE, the judgment appealed from is hereby REVERSED and SET ASIDE. Respondent Eastern Shipping Lines, Inc. is ORDERED to return to petitioner Philippine Home Assurance Corporation the amount it paid under protest in behalf of the consignees herein.

INSURANCE G01 CASE DIGESTS said Macke for and on behalf of the insured. That the sanction of the said defendant had not been obtained by the plaintiff, as required by the said policy. c. That the plaintiff, immediately preceding the outbreak of the alleged fire, willfully placed a gasoline can containing 10 gallons of gasoline in the upper story of said building in close proximity to a portion of said goods, wares, and merchandise, which can was so placed by the plaintiff as to permit the gasoline to run on the floor of said second story, and after so placing said gasoline, he, the plaintiff, placed in close proximity to said escaping gasoline a lighted lamp containing alcohol, thereby greatly increasing the risk of fire. 4. The lower court ruled in favor of Bachrach making British American Assurance liable. ISSUE: WoN British American Assurance liable of such claim. HELD:YES. RATIO DECIDENDI: - Keeping of inflammable oils on the premises, though prohibited by the policy, does not void it if such keeping is incidental to the business. It may be added also that there was no provision in the policy prohibiting the keeping of paints and varnishes upon the premises where the insured property was stored. If the company intended to rely upon a condition of that character, it ought to have been plainly expressed in the policy. - Furthermore, upon reading the policy of insurance issued Page 114 of 154 by the defendant to the plaintiff, it will be noted that there is no provision in said policy prohibiting the plaintiff from placing a mortgage upon the property insured. It is claimed that the execution of a chattel mortgage on the insured property violated what is known as the "alienation clause," which is now found in most policies, and which is expressed in the policies involved in cases 6496 and 6497 by a purchase imposing forfeiture if the interest in the property pass from the insured. (Cases 6496 and 6497, in which are involved other action against other insurance companies for the same loss as in the present action.) *alienation clause - forfeiture if the interest in the property pass from the insured - there is no alienation within the meaning of the insurance law until the mortgage acquires a right to take possession by default under the terms of the mortgage. No such right is claimed to have accrued in the case at bar, and the alienation clause is therefore inapplicable. - This clause has been the subject of a vast number of judicial decisions (13 Am. & Eng. Encyc. of Law, 2d ed., pp. 239 et seq.), and it is held by the great weight of authority that the interest in property insured does not pass by the mere execution of a chattel mortgage and that while a chattel mortgage is a conditional sale, there is no alienation within the meaning of the insurance law until the mortgage acquires a right to take possession by default under the terms of the mortgage. No such right is claimed to have accrued in the case at bar, and the alienation clause is therefore inapplicable.

INSURANCE G01 CASE DIGESTS o 133. TAN CHUCO V. YORKSHIRE F IRE AND LIFE INSURANCE , 14 PHIL. 346 (1909) Tan Chuco vs Yorkshire Fire and Life Insurance Company Facts: Tan Chuco files a claim under an open fire insurance policy for the alleged loss by fire of certain stock of goods insured by Yorkshire. CFI: Evidence did not sustain Yorkshires allegation that Tan Chuco or his agents had intentionally and fraudulently set the building on fire o But was of the opinion that the Tan Chuco failed to establish the value of the goods he alleges were destroyed by the fire. o He submitted fabricated written evidence and false testimony in support of his claim that the insured goods actually destroyed were worth more than the total amount of the insurance thereon. o CFI was of the opinion that the submitted inventory was not genuine and was fraudulently prepared. o Tan Chucos representatives and employees who were in the building when the fire took place, not only made no effort to extinguish the fire, or to save the goods from destruction, but also failed to save any of the books or papers connected with the business of which he was in charge ofthose could have corroborated with the data in the alleged inventory o The inventory submitted was dated January 1, not of custom to Tan Chuco who were of Chinese decent. o No explanation was offered which would account for the remarkable conduct of Tan Chucos manager in preparing an inventory two months after his employer had left for China and then instead of forwarding such inventory to his principal by mail, entrusted it for transmission to a friend who had not even left for China when the fire took place. Indication that Tan Chuco had been experiencing adverse business conditions before the fire

Issue: Whether or not Tan Chuco may claim under the fire insurance policy?

Held: NO. We think that the action of the trial court in rejecting the proof offered by Tan Chuco as to the amount of the loss must be sustained.

The contract of fire insurance being a contract of indemnity, Tan Chuco is only entitled to recover the amount of actual loss sustained by him. There being no express valuation in the policy, the judgment was properly entered against him for lack of satisfactory proof of the amount of loss. Rule: In the absence of express valuation in a fire insurance policy, the insured is only entitled to recover the amount of actual loss sustained and the burden is upon him to establish such amount.

Page 115 of 154

INSURANCE G01 CASE DIGESTS 134. MALAYAN INSURANCE V. CRUZ-ARNALDO, 154 SCRA 672 (1987) G.R. No. L-67835 October 12, 1987 Lessons Applicable: Authority to Receive Payment/Effect of Payment (Insurance) Laws Applicable: Article 64, Article 65, Section 77, Section 306 of the Insurance Code

Under Section 416 of the Insurance Code, the period for appeal is thirty days from notice of the decision of the Insurance Commission. The petitioner filed its motion for reconsideration on April 25, 1981, or fifteen days such notice, and the reglementary period began to run again after June 13, 1981, date of its receipt of notice of the denial of the said motion for reconsideration. As the herein petition was filed on July 2, 1981, or nineteen days later, there is no question that it is tardy by

FACTS:

June 7, 1981: Malayan insurance co., inc. (MICO) issued to Coronacion Pinca, Fire Insurance Policy forher property effective July 22, 1981, until July 22, 1982 October 15,1981: MICO allegedly cancelled the policy for nonpayment, of the premium and sent the corresponding notice to Pinca December 24, 1981: payment of the premium for Pinca was received by Domingo Adora, agent of MICO January 15, 1982: Adora remitted this payment to MICO,together with other payments January 18, 1982: Pinca's property was completely burned February 5, 1982: Pinca's payment was returned by MICO to Adora on the ground that her policy had been cancelled earlier but Adora refused to accept it and instead demanded for payment

four days. Insurance Commission: favored Pinca MICO appealed ISSUE: W/N MICO should be liable because its agent Adora was authorized to receive it

HELD: YES. petition is DENIED


SEC. 77. An insurer is entitled to payment of the premium as soon as the thing is exposed to the peril insured against. Notwithstanding any agreement to the contrary, no policy or contract of insurance issued by an insurance company is valid and binding unless and until the premium thereof has been paid, except in the case of a life or an industrial life policy whenever the grace period provision applies. SEC. 306. xxx xxx xxx

Any insurance company which delivers to an insurance agant or insurance broker a policy or contract of insurance shall be demmed to have authorized such agent or broker to receive on its Page 116 of 154

INSURANCE G01 CASE DIGESTS behalf payment of any premium which is due on such policy or contract of insurance at the time of its issuance or delivery or which becomes due thereon.

As for the method of cancellation, Section 65 provides as follows:

Payment to an agent having authority to receive or collect payment is equivalent to payment to the principal himself; such payment is complete when the money delivered is into the agent's hands and is a discharge of the indebtedness owing to the principal. SEC. 64. No policy of insurance other than life shall be cancelled by the insurer except upon prior notice thereof to the insured, and no notice of cancellation shall be effective unless it is based on the occurrence, after the effective date of the policy, of one or more of the following:

SEC. 65. All notices of cancellation mentioned in the preceding section shall be in writing, mailed or delivered to the named insured at the address shown in the policy, and shall state (a) which of the grounds set forth in section sixty-four is relied upon and (b) that, upon written request of the named insured, the insurer will furnish the facts on which the cancellation is based.

A valid cancellation must, therefore, require concurrence of the following conditions: (1) There must be prior notice of cancellation to the insured; (2) The notice must be based on the occurrence, after the effective date of the policy, of one or more of the grounds mentioned; (3) The notice must be (a) in writing, (b) mailed, or delivered to the named insured, (c) at the address shown in the policy; (4) It must state (a) which of the grounds mentioned in Section 64 is relied upon and (b) that upon written request of the insured, the insurer will furnish the facts on which the cancellation is based.

(a) non-payment of premium; (b) conviction of a crime arising out of acts increasing the hazard insured against; (c) discovery of fraud or material misrepresentation; (d) discovery of willful, or reckless acts or commissions increasing the hazard insured against; (e) physical changes in the property insured which result in the property becoming uninsurable;or (f) a determination by the Commissioner that the continuation of the policy would violate or would place the insurer in violation of this Code.

All MICO's offers to show that the cancellation was communicated to the insured is its employee's testimony that the said cancellation was sent "by mail through our mailing section." without more It stands to reason that if Pinca had really received the said notice, she would not have made payment on the original policy on December 24, 1981. Instead, she would have asked for a Page 117 of 154

INSURANCE G01 CASE DIGESTS new insurance, effective on that date and until one year later, and so taken advantage of the extended period. Incidentally, Adora had not been informed of the cancellation either and saw no reason not to accept the said payment Although Pinca's payment was remitted to MICO's by its agent on January 15, 1982, MICO sought to return it to Adora only on February 5, 1982, after it presumably had learned of the occurrence of the loss insured against on January 18, 1982 make the motives of MICO highly suspicious

Page 118 of 154

INSURANCE G01 CASE DIGESTS

CASUALTY INSURANCE
135. GUINGON V. DEL MONTE, 20 SCRA 1043 (1967) Guingon v. Del Monte 20 SCRA 1043 (1967) Casualty Insurance

2. W/N the heirs can sue the insurer and insured jointly? - YES

RATIO: Affirmed in toto. 1. YES

FACTS: 1. Julio Aguilar owner and operator of several jeepneys insured them with Capital Insurance & Surety Co., Inc. 2. February 20, 1961: Along the intersection of Juan Luna and Moro streets, City of Manila, the jeepneys operated by Aguilar driven by Iluminado del Monte and Gervacio Guingon bumped and Guingon died some days after 3. Iluminado del Monte was charged with homicide thru reckless imprudence and was penalized 4 months imprisonment 4. The heirs of Gervacio Guingon filed an action for damages praying that P82,771.80 be paid to them jointly and severally by the driver del Monte, owner and operator Aguilar, and the Capital Insurance & Surety Co., Inc. 5. CFI: Iluminado del Monte and Julio Aguilar jointly and severally to pay plaintiffs the sum of P8,572.95 as damages for the death of their father, plus P1,000.00 for attorney's fees plus costs 6. Capital Insurance and Surety Co., Inc. is hereby sentenced to pay P5,000 plus P500 as attorney's fees and costs to be applied in partial satisfaction of the judgment rendered against Iluminado del Monte and Julio Aguilar in this case ISSUE: 1. W/N there a stipulation pour autriu to enable that will enable the heirs to sue against Capital Insurance and Surety Co., Inc.? - YES

policy: the insurer agreed to indemnify the insured "against all sums . . . which the Insured shall become legally liable to pay in respect of: a. death of or bodily injury to any person . . . ." indemnity against liability TEST: Where the contract provides for indemnity against liability to third persons, then third persons to whom the insured is liable, CAN sue the insurer. Where the contract is for indemnity against actual loss or payment, then third persons CANNOT proceed against the insurer, the contract being solely to reimburse the insured for liability actually discharged by him thru payment to third persons, said third persons' recourse being thus limited to the insured alone.

2. YES

policy: expressly disallows suing the insurer as a co-defendant of the insured in a suit to determine the latter's liability no action close: suit and final judgment be first obtained against the insured; that only "thereafter" can the person injured recover on the policy Sec. 5 of Rule 2 on "Joinder of causes of action" and Sec. 6 of Rule 3 on "Permissive joinder of parties" cannot be superseded, at least with respect to third persons not a party to the contract, as herein, by a "no action" clause in the contract of insurance.

Page 119 of 154

INSURANCE G01 CASE DIGESTS 136. DE LA CRUZ V. CAPITAL INSURANCE , 17 SCRA 559 (1966) HELD: YES The terms "accident" and "accidental", as used in insurance contracts, have not acquired any technical meaning, and are construed by the courts in their ordinary and common acceptation. Thus, the terms have been taken to mean that which happen by chance or fortuitously, without intention and design, and which is unexpected, unusual, and unforeseen. An accident is an event that takes place without one's foresight or expectation an event that proceeds from an unknown cause, or is an unusual effect of a known cause and, therefore, not expected. Even if we take appellant's theory, the death of the insured in the case at bar would still be entitled to indemnification under the policy. The generally accepted rule is that, death or injury does not result from accident or accidental means within the terms of an accident-policy if it is the natural result of the insured's voluntary act, unaccompanied by anything unforeseen except the death or injury. There is no accident when a deliberate act is performed unless some additional, unexpected, independent, and unforeseen happening occurs which produces or brings about the result of injury or death. In other words, where the death or injury is not the natural or probable result of the insured's voluntary act, or if something unforeseen occurs in the doing of the act which produces the injury, the resulting death is within the protection of policies insuring against death or injury from accident. In the present case, while the participation of the insured in the boxing contest is voluntary, the injury was sustained when he slid, giving occasion to the infliction by his opponent of the blow that threw him to the ropes of the ring. Without this unfortunate incident, that is, the unintentional slipping of the deceased, perhaps he could Page 120 of 154

INSURANCE: CASUALTY INSURANCE SIMON DE LA CRUZ vs CAPITAL INSURANCE and SURETY Co., Inc. FACTS: Eduardo de la Cruz, a mucker in Itogon-Suyoc Mines, Inc. was the holder of an accident insurance policy underwritten by Capital Insurance for the period of November 13, 1956 November 12, 1957. Itugon sponsored a boxing contest as part of their celebration of New Year. Eduardo participated as a nonprofessional boxer. In the course of his bout with another person, another non-professional, of the same height, weight, and size, Eduardo slipped and was hit by his opponent on the left part of the back of his head. This caused Eduardo to fall with his head hitting the rope of the ring. He died the following day. The cause of his death was hemmorhage, intracranial, left. The father, Simon, was named beneficiary and filed a claim with the insurance company. Insurer denied. CFI (Specific Performance) ruled in favour of Simon. Insurer contends that while the death of the insured was due to head injury, said injury was sustained by his voluntary participation in the contest. He cannot be considered to have met his death by accidental means. In other words what is required to be accidental is the means that caused the death and not the death itself.

ISSUE: WON Eduardos death is an accident within the meaning of the policy despite the fact that he voluntarily participated in the boxing contest.

INSURANCE G01 CASE DIGESTS not have received that blow in the head and would not have died. The fact that boxing is attended with some risks of external injuries does not make any injuries received in the course of the game not accidental. In boxing as in other equally physically rigorous sports, such as basketball or baseball, death is not ordinarily anticipated to result. If, therefore, it ever does, the injury or death can only be accidental or produced by some unforeseen happening or event as what occurred in this case. As to the policy provision enumerating sports events which are excluded from coverage, death or disablement resulting from engagement in boxing contests was not declared outside of the protection of the insurance contract. Failure of the defendant insurance company to include death resulting from a boxing match or other sports among the prohibitive risks leads inevitably to the conclusion that it did not intend to limit or exempt itself from liability for such death.

137. PAN MALAYAN V. COURT OF APPEALS, 184 SCRA 54 (1990) PAN MALAYAN INSURANCE vs CA GR 81026, 184 SCRA 54; April 3, 1990 Topic: Casualty Insurance: Accident and health insurance Facts: Pan Malayan insured a Lancer car registered in the name of Canlubang Automotive Resources. On May 26, 1985 the car was hit by a pick-up truck due to the carelessness, recklessness and imprudence of its driver. The insured car suffered damages, whose repairs Pan Malayan paid for. Thus it was subrogated in the rights of the owner against the pick-up drivers employer, the private respondent Erlinda Fabie. Despite demands, Fabie refused and failed to pay. Pan Malayan alleged that the damage to the insured car was covered under the own damage coverage of the policy, and that its driver was authorized at the time of the accident. The respondents argued that payment under the "own damage" clause of the insurance policy precluded subrogation under Article 2207 of the Civil Code, since indemnification there was made on the assumption that there was no wrongdoer or no third party at fault. Pan Malayan alleged in its complaint that, pursuant to a motor vehicle insurance policy, it had indemnified Canlubang for the damage to the insured car resulting from a traffic accident allegedly caused by the negligence of the driver of private respondent, Erlinda Fabie. It thus claims that its cause of action against private respondents was anchored upon Article 2207 of the Civil Code. The RTC dismissed Pan Malayans complaint. The CA affirmed. Issue:

Page 121 of 154

INSURANCE G01 CASE DIGESTS 1. Was the insurers indemnity under the own damage clause of its policy an admission that its insured/ client caused the damage? 2. Given this admission, was the insurer subrogated? Held: 1. NO 2. YES Rationale: (Only #1) It must be emphasized that the lower court's ruling that the "own damage" coverage under the policy impliesdamage to the insured car caused by the assured itself, instead of third parties, proceeds from an incorrect comprehension of the phrase "own damage" as used by the insurer. When PANMALAY utilized the phrase "own damage" a phrase which, incidentally, is not found in the insurance policy to define the basis for its settlement of CANLUBANG's claim under the policy, it simply meant that it had assumed to reimburse the costs for repairing the damage to the insured vehicle. It is in this sense that the so-called "own damage" coverage under Section III of the insurance policy is differentiated from Sections I and IV-1 which refer to "Third Party Liability" coverage (liabilities arising from the death of, or bodily injuries suffered by, third parties) and from Section IV-2 which refer to "Property Damage" coverage (liabilities arising from damage caused by the insured vehicle to the properties of third parties). Neither is there merit in the Court of Appeals' ruling that the coverage of insured risks under Section III-1 of the policy does not include to the insured vehicle arising from collision or overturning due to the negligent acts of the third party. Not only does it stem from an erroneous interpretation of the provisions of the section, but it also violates a fundamental rule on the interpretation of property insurance contracts. 1. It is a basic rule in the interpretation of contracts that the terms of a contract are to be construed according to the sense and meaning of the terms which the parties thereto have used. In the case of property insurance policies, the evident intention of the contracting parties, i.e., the insurer and the assured, determine the import of the various terms and provisions embodied in the policy. It is only when the terms of the policy are ambiguous, equivocal or uncertain, such that the parties themselves disagree about the meaning of particular provisions, that the courts will intervene. In such an event, the policy will be construed by the courts liberally in favor of the assured and strictly against the insurer. Section III-1 of the insurance policy which refers to the conditions under which the insurer PANMALAY is liable to indemnify the assured CANLUBANG against damage to or loss of the insured vehicle, reads as follows: SECTION III LOSS OR DAMAGE 1. The Company will, subject to the Limits of Liability, indemnify the Insured against loss of or damage to the Scheduled Vehicle and its accessories and spare parts whilst thereon: (a) by accidental collision or overturning, or collision or overturning consequent upon mechanical breakdown or consequent upon wear and tear; (b) by fire, external explosion, self ignition or lightning or burglary, housebreaking or theft; Page 122 of 154

INSURANCE G01 CASE DIGESTS (c) by malicious act; (d) whilst in transit (including the processes of loading and unloading) incidental to such transit by road, rail, inland, waterway, lift or elevator. It cannot be said that the meaning given by PANMALAY and CANLUBANG to the phrase "by accidental collision or overturning" found in the first paint of sub-paragraph (a) is untenable. Although the terms "accident" or "accidental" as used in insurance contracts have not acquired a technical meaning, the Court has on several occasions defined these terms to mean that which takes place "without one's foresight or expectation, an event that proceeds from an unknown cause, or is an unusual effect of a known cause and, therefore, not expected". Certainly, it cannot be inferred from jurisprudence that these terms, without qualification, exclude events resulting in damage or loss due to the fault, recklessness or negligence of third parties. The concept "accident" is not necessarily synonymous with the concept of "no fault". It may be utilized simply to distinguish intentional or malicious acts from negligent or careless acts of man. Moreover, a perusal of the provisions of the insurance policy reveals that damage to, or loss of, the insured vehicle due to negligent or careless acts of third parties is not listed under the general and specific exceptions to the coverage of insured risks which are enumerated in detail in the insurance policy itself. 138. SUN INSURANCE V. COURT OF APPEALS, 211 SCRA 554 (1990)

SUN INSURANCE OFFICE, LTD., petitioner, vs. THE HON. COURT OF APPEALS and NERISSA LIM, respondents. CRUZ, J.: July 17, 1992 Facts 1. Sun life issued a personal accident policy to Felix Lim. 2. 2 months later, he died. His wife as the beneficiary sought payment of the policy but was rejected. a. However, despite the rejection of the claim, Sun life found that there was neither suicide nor an accident. 3. The eyewitness to his death, his secretary, testified that he was playing with his handgun from which he had previously removed his magazine. He pointed it to his temple and then, an explosion occurred. Lim fell on the floor dead. 4. RTC ruled in favor of the wife. Sun life should pay the policy plus damages. 5. CA affirmed. Issue Is the death of Lim an accident in which the beneficiary may claim payment of the policy? YES (To restate: Does Lims death fall under the exceptions in which the insurer would not be liable? NO) Page 123 of 154

INSURANCE G01 CASE DIGESTS Ruling: The Court believes that Lims death was an ACCIDENT. o The term, when used in an insurance contract, is to be construed and considered according to the ordinary understanding and common usage and speech of people generally. o An accident is an event which happens without any human agency or, if happening through human agency, an event which, under the circumstances, is unusual to and not expected by the person to whom it happens. It has also been defined as an injury which happens by reason of some violence or CASUALTY to the injured without his design, consent, or voluntary co-operation. o As the secretary testified, Lim had removed the magazine from the gun and believed it was no longer dangerous. He expressly assured her that the gun was not loaded. It is submitted that Lim did not willfully expose himself to needless peril when he pointed the gun to his temple because the fact is that he thought it was not unsafe to do so. The act was precisely intended to assure Nalagon that the gun was indeed harmless. o Lim did not know that the gun he placed to his head was loaded. Lim was unquestionably negligent and that negligence cost him his own life. But it should not prevent his widow from recovering from the insurance policy he obtained precisely against accident. There is nothing in the policy that relieves the insurer of the responsibility to pay the indemnity agreed upon if the insured is shown to have contributed to his own accident. Indeed, most accidents are caused by negligence. There are only four exceptions expressly made in the contract to relieve the insurer from liability, and none of these exceptions is applicable in the case at bar. **
** Exceptions The Company shall not be liable in respect of 1. bodily injury a) sustained i) while the Insured Person is engaging in (or practicing for or taking part in training peculiar to) any of the Excluded Activities. ii) by any person before such person attains the Lower Age Limit or after the expiry of the Period of Insurance during which such person attains the Upper Age Limit. b) consequent upon i) the Insured Person committing or attempting to commit suicide or wilfully exposing himself to needless peril except in an attempt to save human life. ii) war, invasion, act of foreign enemy, hostilities (whether war be declared or not) civil war, rebellion, revolution, insurrection, or military or usurped power. 2. bodily injury or Death Disablement or Medical Expenses consequent upon or contributed to by the Insured Person a) having taken a drug unless the Insured proves that the drug was taken in accordance with proper medical prescription and directions and not for treatment of drug addiction. b) suffering from pre-existing physical or mental defect or infirmity which had not been declared to and accepted in writing by the Company. 3. Death Disablement or Medical Expenses consequent upon or contributed to by the Insured Person being pregnant or suffering from sickness or disease not resulting from bodily injury or suffering from bodily injury due to a gradually operating cause.

Page 124 of 154

INSURANCE G01 CASE DIGESTS


4. Risks of Murder and Assault.

The award of damages is deleted since there is no evidence to show that there was any wrongful act done by the insurer, Sun Life.

Were the injuries sustained by the deceased intentionally afflicted, thus not covered by the accidental death benefit clause? HELD Yes. Whatever the true intent of the robbers may be, the act itself of inflicting the injuries was intentional. The exception clause does not speak of the purpose of the one causing the injury, only of the fact that such injuries have been "intentionally" inflicted, to distinguish them from injuries which, although received at the hands of a third party, are purely accidental. Examples would be when a gun which discharges while being cleaned and kills a bystander; or a hunter who shoots at his prey and hits a person instead; or an athlete in a competitive game involving physical effort who collides with an opponent and fatally injures him as a result. In Calanoc vs. CA, where a shot was fired and it turned out afterwards that the watchman was hit in the abdomen, the wound causing his death, the Court held that it could not be said that the killing was intentional for there was the possibility that the malefactor had fired the shot to scare the people around for his own protection and not necessarily to kill or hit the victim. It is not the same with the case at bar, for while a single shot fired from a distance, and by a person who was not even seen aiming at the victim, could indeed have been fired without the intent to kill or injure, nine wounds inflicted with bladed weapons at close range cannot conceivably be considered as innocent insofar as such intent is concerned.

SC affirms CA. 139. BIAGTAN V. INSULAR LIFE, 144 SCRA 58 (1972)

Biagtan vs Insular Life (1972) FACTS Juan Biagtan was insured with Insular for P5k with a supplementary contract Accidental Death Benefit clause for another P5k if "the death of the Insured resulted directly from bodily injury effected solely through external and violent means sustained in an accident . . . and independently of all other causes." The clause, however, expressly provided that it would not apply where death resulted from an injury "intentionally inflicted by a third party." One night, robbers entered their house. Juan was stabbed 9 times with a knife and then died. The robbers were convicted of robbery with homicide. The family now claims for the additional P5k from Insular, under the Accidental Death Benefit clause. Insular refused saying that the death resulted from injuries intentionally inflicted by 3rd parties and was therefore not covered. The family sued. CFI ruled in favor of the family. ISSUE

Page 125 of 154

INSURANCE G01 CASE DIGESTS Insular won and the family cannot claim the additional 5k under the ADB clause because the injury was intentionally caused by a person, i.e. not accidental. associated words and concepts in the policy plainly exclude the accidental death from the coverage of the policy only where the injuries are self-inflicted or attended by some proscribed act of the insured or are incurred in some expressly excluded calamity such as riot, war or atomic explosion.- The untenability of insurer's claim that the insured's death fell within the exception is further heightened by the stipulated fact that two other insurance companies which likewise covered the insured for much larger sums under similar accidental death benefit clauses promptly paid the benefits thereof to plaintiffs beneficiaries.

Justice Teehankee dissents, saying that Calanoc v. CA is controlling because the insurance company was unable to prove that the killing was intentional. Burden of proof is with the insurance company. Insurance, being contracts of adhesion, must be construed strictly against insurance company in cases of ambiguity. The supplementary contract enumerated exceptions. The only exception which is not susceptible of classification is that provided in paragraph 5(e), the very exception which is currently the subject of contention, which would also except injuries "inflicted intentionally by a third party, either with or without provocation on the part of the insured, and whether or not the attack or the defense by the third party was caused by a violation of the law by the insured." The ambiguous clause conflicts with all the other four exceptions in the same paragraph 5 particularly that immediately preceding it in item (d) which excepts injuries received where the insured has violated the law or provoked the injury, while this clause, construed as the insurance company now claims, would seemingly except also all other injuries, intentionally inflicted by a third party, regardless of any violation of law or provocation by the insured, and defeat the very purpose of the policy of giving the insured double indemnity in case of accidental death by "external and violent means" in the very language of the policy.' It is obvious from the very classification of the exceptions and applying the rule of noscitus a sociis, that the double-indemnity policy covers the insured against accidental death, whether caused by fault, negligence or intent of a third party which is unforeseen and unexpected by the insured. All the

140.

CALANOC V. COURT OF APPEALS, 98 PHIL. 79 (1955)

Calanoc v CA Facts: The circumstances surrounding the death of Melencio Basilio show that when he was killed at about seven oclock in the night of January 25, 1951, he was on duty as watchman of the Manila Auto Supply at the corner of Avenida Rizal and Zurbaran; that it turned out that Atty. Antonio Ojeda who had his residence at the corner of Zurbaran and Oroquieta, a block away from Basilios station, had come home that night and found that his house was well-lighted, but with the windows closed; that getting suspicious that there were culprits in his house, Atty. Ojeda retreated to look for a policeman and finding Basilio in khaki uniform, asked him to accompany him to the house with the latter refusing on the ground that he was not a policeman, but suggesting that Atty. Ojeda should ask the traffic policeman on duty at the corner of Rizal Avenue and Zurbaran; that Atty. Ojeda went to the traffic policeman at said Page 126 of 154

INSURANCE G01 CASE DIGESTS corner and reported the matter, asking the policeman to come along with him, to which the policeman agreed; that on the way to the Ojeda residence, the policeman and Atty. Ojeda passed by Basilio and somehow or other invited the latter to come along; that as the tree approached the Ojeda residence and stood in front of the main gate which was covered with galvanized iron, the fence itself being partly concrete and partly adobe stone, a shot was fired; that immediately after the shot, Atty. Ojeda and the policeman sought cover; that the policeman, at the request of Atty. Ojeda, left the premises to look for reinforcement; that it turned out afterwards that the special watchman Melencio Basilio was hit in the abdomen, the wound causing his instantaneous death; that the shot must have come from inside the yard of Atty. Ojeda, the bullet passing through a hole waist-high in the galvanized iron gate; that upon inquiry Atty. Ojeda found out that the savings of his children in the amount of P30 in coins kept in his aparador contained in stockings were taken away, the aparador having been ransacked; that a month thereafter the corresponding investigation conducted by the police authorities led to the arrest and prosecution of four persons in Criminal Case No. 15104 of the Court of First Instance of Manila for Robbery in an Inhabited House and in Band with Murder.

Ratio: The circumstances of Basilios death cannot be taken as purely intentional on the part of Basilio to expose himself to the danger. There is no proof that his death was the result of intentional killing because there is the possibility that the malefactor had fired the shot merely to scare away the people around. While as a general rule "the parties may limit the coverage of the policy to certain particular accidents and risks or causes of loss, and may expressly except other risks or causes of loss therefrom", however, it is to be desired that the terms and phraseology of the exception clause be clearly expressed so as to be within the easy grasp and understanding of the insured, for if the terms are doubtful or obscure the same must of necessity be interpreted or resolved against the one who has caused the obscurity. (Article 1377, new Civil Code) And so it has been generally held that the "terms in an insurance policy, which are ambiguous, equivocal, or uncertain . . . are to be construed strictly and most strongly against the insurer, and liberally in favor of the insured so as to effect the dominant purpose of indemnity or payment to the insured, especially where a forfeiture is involved", and the reason for this rule is that he "insured usually has no voice in the selection or arrangement of the words employed and that the language of the contract is selected with great care and deliberation by experts and legal advisers employed by, and acting exclusively in the interest of, the insurance company." Insurance is, in its nature, complex and difficult for the layman to understand. Policies are prepared by experts who know and can Page 127 of 154

Issue: Whether or not the Philippine American Life Insurance Co. liable to the petitioner for the amount covered by the supplemental contract?

Held: Yes.

INSURANCE G01 CASE DIGESTS anticipate the bearings and possible complications of every contingency. So long as insurance companies insist upon the use of ambiguous, intricate and technical provisions, which conceal rather than frankly disclose, their own intentions, the courts must, in fairness to those who purchase insurance, construe every ambiguity in favor of the insured. An insurer should not be allowed, by the use of obscure phrases and exceptions, to defeat the very purpose for which the policy was procured. Applying this to the situation, the ambiguous or obscure terms in the insurance policy are to be construed strictly against the insurer and liberally in favor of the insured party. The reason is to ensure the protection of the insured since these insurance contracts are usually arranged and employed by experts and legal advisers acting exclusively in the interest of the insurance company. As long as insurance companies insist upon the use of ambiguous, intricate and technical provisions, which conceal their own intentions, the courts must, in fairness to those who purchase insurance, construe every ambiguity in favor of the insured.

141. PHIL AM CARE HEALTH SYSTEMS V. COURT OF APPEALS, 379 SCRA 356 (2002) PHILAMCARE HEALTH SYSTEMS, INC. V CA (TRINOS) 379 SCRA 357 YNARES-SANTIAGO; March 18, 2002

NATURE Petition for review of CA decision

FACTS - Ernani TRINOS, deceased husband of respondent Julita, applied for a health care coverage with Philamcare Health Systems, Inc . In the standard application form, he answered no to the question: Have you or any of your family members ever consulted or been treated for high blood pressure, heart trouble, diabetes, cancer, liver disease, asthma or peptic ulcer? (If Yes, give details). - The application was approved for period of one year; upon termination, it was extended for another 2 years. Amount of coverage was increased to a maximum sum of P75T per disability. - During this period, Ernani suffered a HEART ATTACK and was confined at the Manila Medical Center (MMC) for one month. While her husband was in the hospital, Julita tried to claim the hospitalization benefits. Page 128 of 154

INSURANCE G01 CASE DIGESTS - Petitioner treated the Health Care Agreement (HCA) as void since there was a concealment regarding Ernanis medical history. Doctors at the MMC allegedly discovered at the time of his confinement, he was hypertensive, diabetic and asthmatic. Julita then paid the hospitalization expenses herself, amounting to about P76T. - After her husband died, Julita instituted action for damages against Philamcare and its Pres. After trial, the lower court ruled in her favor and ordered Philamcare to reimburse medical and hospital coverage amounting to P76T plus interest, until fully paid; pay moral damages of P10T; pay exemplary damages of P10T; attys fees of P20T. - CA affirmed the decision of the trial court but deleted all awards for damages and absolved petitioner Reverente. (4) It is not an insurance company, governed by Insurance Commission, but a Health Maintenance Organization under the authority of DOH. (5) Trinos concealed a material fact in his application. (6) Julita was not the legal wife since at the time of their marriage, the deceased was previously married to another woman who was still alive.*

ISSUES 1. WON a health care agreement is an insurance contract (If so, incontestability clause under the Insurance Code is applicable) 2. WON the HCA can be invalidated on the basis of alleged concealment

Petitioners Claims (1) Agreement grants living benefits such as medical check-ups and hospitalization which a member may immediately enjoy so long as he is alive upon effectivity of the agreement until its expiration. (2) Only medical and hospitalization benefits are given under the agreement without any indemnification, unlike in an insurance contract where the insured is indemnified for his loss. (3) HCAs are only for a period of one year; therefore, incontestability clause does not apply, as it requires effectivity period of at least 2 yrs. HELD

YES

Ratio Every person has an insurable interest in the life and health of himself1. The health care agreement was in the nature of non-life
1 Sec.10. Every person has an insurable interest in the life and health:

(1) of himself, of his spouse and of his children;

Page 129 of 154

INSURANCE G01 CASE DIGESTS insurance, which is primarily a contract of indemnity. Once the member incurs hospital, medical or any other expense arising from sickness, injury or other stipulated contingent, the health care provider must pay for the same to the extent agreed upon under the contract. (e) In consideration of the insurers promise, the insured pays a premium.

2. NO Ratio Where matters of opinion or judgment are called for, answers made in good faith and without intent to deceive will not avoid a policy even though they are untrue; since in such case the insurer is not justified in relying upon such statement, but is obligated to make further inquiry. Reasoning - The fraudulent intent on the part of the insured must be established to warrant rescission of the insurance contract. The right to rescind should be exercised previous to the commencement of an action on the contract. No rescission was made. Besides, the cancellation of health care agreements as in insurance policies requires: (a) Prior notice of cancellation to insured; (b) Notice must be based on the occurrence after effective date of the policy of one or more of the grounds mentioned; (c) Must be in writing, mailed or delivered to the insured at the address shown in the policy; (d) Must state the grounds relied upon provided in Section 64 of the Insurance Code and upon request of insured, to furnish facts on which cancellation is based. Page 130 of 154

Reasoning - A contract of insurance2 is an agreement whereby one undertakes for a consideration to indemnify another against loss, damage or liability arising from an unknown or contingent event. - An insurance contract exists where the following elements concur: (a) The insured has an insurable interest; (b) The insured is subject to a risk of loss by the happening of the peril; (c) The insurer assumes the risk; (d) Such assumption of risk is part of a general scheme to distribute actual losses among a large group of persons bearing a similar risk; and

(2) of any person on whom he depends wholly or in part for education or support, or in whom he has a pecuniary interest; (3) of any person under a legal obligation to him for the payment of money, respecting property or service, of which death or illness might delay or prevent the performance; and (4) of any person upon whose life any estate or interest vested in him depends.
2 Section 2 (1) of the Insurance Code

INSURANCE G01 CASE DIGESTS - These conditions have not been met. When the terms of insurance contract contain limitations on liability, courts should construe them in such a way as to preclude insurer from non-compliance of obligation. Being a contract of adhesion, terms of an insurance contract are to be construed strictly against the party which prepared it the insurer. - Also, Philamcare had 12 months from the date of issuance of the Agreement within which to contest the membership of the patient if he had previous ailment of asthma, and six months from the issuance of the agreement if the patient was sick of diabetes or hypertension. * The health care agreement is in the nature of a contract of indemnity. Hence, payment should be made to the party who incurred the expenses. It is clear that respondent paid all the hospital and medical bills; thus, she is entitled to reimbursement. 142. FORTUNE INSURANCE V. COURT OF APPEALS, 244 SCRA 308 (1996)

FORTUNE VS CA (1995) G.R. No. 115278 May 23, 1995 Petitioner: FORTUNE INSURANCE AND SURETY CO., INC. (Fortune) Respondent: PRODUCERS BANK OF THE PHILIPPINES (PBP) FACTS: > PBP filed against Fortune a complaint for recovery of the sum of P725,000.00 under the policy issued by Fortune. The money was allegedly lost during a robbery of Producer's armored vehicle while it was in transit to transfer the money from its Pasay City Branch to its head office in Makati along Taft Avenue. >The armored car was driven by Benjamin Magalong escorted by Security Guard Saturnino Atig. >Driver Magalong was assigned by PRC Management Systems with the PBP by virtue of an Agreement and Atiga was assigned by Unicorn Security Services, Inc. by virtue of a contract of Security Service. >After an investigation conducted by the Pasay police authorities, the driver Magalong and guard Atiga were charged, together with Edelmer Bantigue, Reynaldo Aquino and John Doe, with violation of P.D. 532 (Anti-Highway Robbery Law) before the Fiscal of Pasay City. >Demands were made by PBP but Fortune refused to pay as the loss is excluded from the coverage of the insurance policy which is Page 131 of 154

Disposition Petition DENIED.

INSURANCE G01 CASE DIGESTS stipulated under "General Exceptions" Section (b) which reads as follows: GENERAL EXCEPTIONS The company shall not be liable under this policy in report of xxx xxx xxx (b) any loss caused by any dishonest, fraudulent or criminal act of the insured or any officer, employee, partner, director, trustee or authorized representative of the Insured whether acting alone or in conjunction with others. . . . 8. The plaintiff opposes the contention of the defendant and contends that Atiga and Magalong are not its "officer, employee, . . . trustee or authorized representative . . . at the time of the robbery. >RTC & CA: held that there should be recovery. The trial court ruled that Magalong and Atiga were not employees or representatives of Producers. The wages and salaries of both Magalong and Atiga are presumably paid by their respective firms, which alone wields the power to dismiss them. Neither is the Court prepared to accept the proposition that driver Magalong and guard Atiga were the "authorized representatives" of plaintiff. ISSUE: W/N the recovery in the policy is precluded under the general exceptions clause? Fortune is exempt from liability under the general exceptions clause of the insurance policy. >It should be noted that the insurance policy entered into by the parties is a theft or robbery insurance policy which is a form of casualty insurance (Section 174 of the Insurance Code). Other than what is mentioned in the provision, the rights and obligations of the parties must be determined by the terms of their contract, taking into consideration its purpose and always in accordance with the general principles of insurance law. >The purpose of the exception is to guard against liability should the theft be committed by one having unrestricted access to the property. In such cases, the terms specifying the excluded classes are to be given their meaning as understood in common speech. The terms "service" and "employment" are generally associated with the idea of selection, control, and compensation. >A contract of insurance is a contract of adhesion, thus any ambiguity therein should be resolved against the insurer, or it should be construed liberally in favor of the insured and strictly against the insurer. Limitations of liability should be regarded with extreme jealousy and must be construed in such a way, as to preclude the insurer from non-compliance with its obligation. >If the terms of the contract are clear and unambiguous, there is no room for construction and such terms cannot be enlarged or diminished by judicial construction. HELD: YES.

Page 132 of 154

INSURANCE G01 CASE DIGESTS >An insurance contract is a contract of indemnity. It is settled that the terms of the policy constitute the measure of the insurer's liability. In the absence of statutory prohibition to the contrary, insurance companies have the same rights as individuals to limit their liability and to impose whatever conditions they deem best upon their obligations not inconsistent with public policy. >It was clear that Fortunes intention is to exclude and exempt from protection and coverage losses arising from dishonest, fraudulent, or criminal acts of persons granted or having unrestricted access to Producers' money or payroll. When it used then the term "employee," it must have had in mind any person who qualifies as such as generally and universally understood, or jurisprudentially established in the light of the four standards in the determination of the employer-employee relationship, or as statutorily declared even in a limited sense as in the case of Article 106 of the Labor Code which considers the employees under a "labor-only" contract as employees of the party employing them and not of the party who supplied them to the employer. >Fortune claims that Producers' contracts with PRC Management Systems and Unicorn Security Services are "labor-only" contracts. But even granting for the sake of argument that these contracts were not "labor-only" contracts, and PRC Management Systems and Unicorn Security Services were truly independent contractors, we are satisfied that Magalong and Atiga were, in respect of the transfer of Producer's money from its Pasay City branch to its head office in Makati, its "authorized representatives" who served as such with its teller Maribeth Alampay. >Producers entrusted the three with the specific duty to safely transfer the money to its head office, with Alampay to be responsible for its custody in transit; Magalong to drive the armored vehicle which would carry the money; and Atiga to provide the needed security for the money, the vehicle, and his two other companions. In short, for these particular tasks, the three acted as agents of Producers. A "representative" is defined as one who represents or stands in the place of another; one who represents others or another in a special capacity, as an agent, and is interchangeable with "agent."

143. PERLA COMPANIA SEGUROS V. RAMOLETE , 203 SCRA 487 (1991) PERLA COMPANIA DE SEGUROS, INC., petitioner, vs. HON. JOSE R. RAMOLETE, PRIMITIVA Y. PALMES, HONORATO BORBON, SR., OFFICE OF THE PROVINCIAL SHERIFF, PROVINCE OF CEBU, respondents. FACTS: A Cimarron PUJ owned and registered in the name of Nelia Enriquez, and driven by Cosme Casas, collided with a private jeep owned by the late Calixto Palmes (husband of private respondent Primitiva Palmes) who was then driving the private jeep. The impact of the collision was such that the private jeep was flung away to a distance of about thirty (30) feet and then fell on its right side pinning down Calixto Palmes. He died as a result of cardio-respiratory arrest due to a crushed chest. It also caused physical injuries on the part of Adeudatus Borbon who was then only two (2) years old. Private respondents Primitiva Palmes (widow of Calixto Palmes) and Honorato Borbon, Sr. (father of minor Adeudatus Borbon) filed a complaint with the CFI Judge Jose

Page 133 of 154

INSURANCE G01 CASE DIGESTS R. Ramolete ordered the Borbon claim excluded from the complaint it being distinct and separate from that of Palmes CFI: in favor of Palmes Judgment became final and executory, writ of execution issued but returned unsatisfied The judgment debtor Nelia Enriquez was summoned. She declared under oath that the Cimarron PUJ registered in her name was covered by a third-party liability insurance policy issued by petitioner Perla. Palmes filed a motion for garnishment. The Judge issued an Order directing the Provincial Sheriff or his deputy to garnish the third-party liability insurance policy. Perla filed a MR alleging that the writ was void on the ground that it (Perla) was not a party to the case and that jurisdiction over its person had never been acquired by the trial court Thus certiorari and prohibition alleging grave abuse of discretion on the part of respondent Judge Ramolete in ordering garnishment of the third-party liability insurance contract issued by petitioner Perla in favor of the judgment debtor, Nelia Enriquez. ISSUE: Can Perla be made liable? - YES RATIO: In the present case, there can be no doubt, therefore, that the trial court actually acquired jurisdiction over petitioner Perla when it was served with the writ of garnishment of the third-party liability insurance policy it had issued in favor of judgment debtor Nelia Enriquez. Perla cannot successfully evade liability thereon by such a contention. Every interest which the judgment debtor may have in property may be subjected to execution. judgment debtor Nelia Enriquez Page 134 of 154 clearly had an interest in the proceeds of the third-party liability insurance contract. In a third-party liability insurance contract, the insurer assumes the obligation of paying the injured third party to whom the insured is liable. The insurer becomes liable as soon as the liability of the insured to the injured third person attaches. Prior payment by the insured to the injured third person is not necessary in order that the obligation of the insurer may arise. From the moment that the insured became liable to the third person, the insured acquired an interest in the insurance contract, which interest may be garnished like any other credit.

INSURANCE G01 CASE DIGESTS 144. SHAFER V. JUDGE OF RTC, 167 SCRA 386 (1986) Upon motion, petitioner was granted leave by the former presiding judge of the trail court to file a third party complaint against the herein private respondent, Makati Insurance Company, Inc. Said insurance company, however, moved to vacate the order granting leave to petitioner to file a third party complaint against it and/or to dismiss the same. The court a quo issued an order dismissing the third party complaint on the ground that it was premature, based on the premise that unless the accused (herein petitioner) is found guilty and sentenced to pay the offended party (Poblete Sr.) indemnity or damages, the third party complaint is without cause of action. The court further stated that the better procedure is for the accused (petitioner) to wait for the outcome of the criminal aspect of the case to determine whether or not the accused, also the third party plaintiff, has a cause of action against the third party defendant for the enforcement of its third party liability (TPL) under the insurance contract. Petitioner moved for reconsideration of said order, but the motion was denied; hence, this petition.

Shafer v Hon Judge of RTC Facts: Petitioner Sherman Shafer obtained a private car policy over his Ford Laser car from Makati Insurance Company, Inc., for third party liability (TPL).> During the effectivity of the policy, an information for reckless imprudence resulting in damage to property and serious physical injuries was filed against petitioner. The information reads as follows: the above-named accused, being then the driver and in actual physical control of a Ford Laser car did then and there wilfully, unlawfully and criminally drive, operate and manage the said Ford Laser car in a careless, reckless and imprudent manner without exercising reasonable caution, diligence and due care to avoid accident to persons and damage to property and in disregard of existing traffic rules and regulations, causing by such carelessness, recklessness and imprudence the said Ford Laser car to hit and bump a Volkswagen car and as a result thereof one Jovencio Poblete, Sr. who was on board of the said Volkswagen car sustained physical injurieswhich injuries causing deformity on the face." The owner of the damaged Volkswagen car filed a separate civil action against petitioner for damages, while Jovencio Poblete, Sr., who was a passenger in the Volkswagen car when allegedly hit and bumped by the car driven by petitioner, did not reserve his right to file a separate civil action for damages. Instead, in the course of the trial in the criminal case, Poblete, Sr. testified on his claim for damages for the serious physical injuries which he claimed to have sustained as a result of the accident.

ISSUE: WON the court a quo erred in dismissing petitioner's third party complaint on the ground that petitioner had no cause of action yet against the insurance company (third party defendant). HELD: YES There is no need on the part of the insured to wait for the decision of the trial court finding him guilty of reckless imprudence. The occurrence of the injury to the third party immediately gave rise to the liability of the insurer under its policy Compulsory Motor Vehicle Liability Insurance (third party liability, or TPL) is primarily intended to provide compensation for the Page 135 of 154

INSURANCE G01 CASE DIGESTS death or bodily injuries suffered by innocent third parties or passengers as a result of a negligent operation and use of motor vehicles. The victims and/or their dependents are assured of immediate financial assistance, regardless of the financial capacity of motor vehicle owners. The liability of the insurance company under the Compulsory Motor Vehicle Liability Insurance is for loss or damage. Where an insurance policy insures directly against liability, the insurer's liability accrues immediately upon the occurrence of the injury or event upon which the liability depends, and does not depend on the recovery of judgment by the injured party against the insured. The injured for whom the contract of insurance is intended can sue directly the insurer. The general purpose of statutes enabling an injured person to proceed directly against the insurer is to protect injured persons against the insolvency of the insured who causes such injury, and to give such injured person a certain beneficial interest in the proceeds of the policy, and statutes are to be liberally construed so that their intended purpose may be accomplished. It has even been held that such a provision creates a contractual relation which inures to the benefit of any and every person who may be negligently injured by the named insured as if such injured person were specifically named in the policy. In the event that the injured fails or refuses to include the insurer as party defendant in his claim for indemnity against the insured, the latter is not prevented by law to avail of the procedural rules intended to avoid multiplicity of suits. Not even a "no action" clause under the policy-which requires that a final judgment be first obtained against the insured and that only thereafter can the person insured recover on the policy can prevail over the Rules of Court provisions aimed at avoiding multiplicity of suits. A third party complaint is a device allowed by the rules of procedure by which the defendant can bring into the original suit a party against whom he will have a claim for indemnity or remuneration as a result of a liability established against him in the original suit. Third party complaints are allowed to minimize the number of lawsuits and avoid the necessity of bringing two (2) or more actions involving the same subject matter. They are predicated on the need for expediency and the avoidance of unnecessary lawsuits. If it appears probable that a second action will result if the plaintiff prevails, and that this result can be avoided by allowing the third party complaint to remain, then the motion to dismiss the third party complaint should be denied. Respondent insurance company's contention that the third party complaint involves extraneous matter which will only clutter, complicate and delay the criminal case is without merit. An offense causes two (2) classes of injuries the first is the social injury produced by the criminal act which is sought to be repaired thru the imposition of the corresponding penalty, and the second is the personal injury caused to the victim of the crime, which injury is sought to be compensated thru indemnity, which is civil in nature. In the instant case, the civil aspect of the offense charged, i.e., serious physical injuries allegedly suffered by Jovencio Poblete, Sr., was impliedly instituted with the criminal case. Petitioner may thus raise all defenses available to him insofar as the criminal and civil aspects of the case are concerned. The claim of petitioner for payment of indemnity to the injured third party, under the insurance policy, for the alleged bodily injuries caused to said third party, arose from the offense charged in the criminal case, from which the injured (Jovencio Poblete, Sr.) has sought to recover civil damages. Hence, such claim of petitioner against the insurance company cannot be regarded as not related to the criminal action.

Page 136 of 154

INSURANCE G01 CASE DIGESTS

145. MALAYAN INSURANCE V. CA, VALLEJOS ET AL .,, 165 SCRA 136 (1998)

CASUALTY INSURANCE 144. MALAYAN INSURANCE CO., INC.,, vs. THE HON. COURT OF APPEALS, MARTIN C. VALLEJOS,

4. PANTRANCO claimed that the jeep of Sio Choy was operated at an excessive speed and bumped the PANTRANCO bus. It prayed that it be absolved from any and all liability. Defendant Sio Choy and the Insurance Company denied the liability, claiming that the fault in the accident was solely imputable to the PANTRANCO.

5. SIO CHOY'S CONTENTION FACTS 1. Petitioner Malayan issued in favor of the respondent Sio Choy Private Comprehensive Car Comprehensive covering a Willys Jeep. The insurance coverage was for own damage not exceeding P600.00 and third party liability in the amount of P20,000 alleged that he had actually paid the Matrin Vallejos, the amount of P5,000.00 for the hospitalization and other expenses He alleged that Malayan had issued in his favor a private car comprehensive car, wherein the insurance company obligated itself to indemnify Sio Choy, as insured for the damge to his motor vehicle as well as the liability to third persin arrising to any vehicular accident (which the policy is in full force and effect when the vehicular accident complained of occurred. He prayed that he be reimbursed by the insurance company for the amount that he may be ordered to pay File a third party complaint against San Leon, since the driver was its employee 6. RTC ruled in favor of the Martin Vallejos and against Sio Choy and Malayan Insurance and third party defendant San Leon Rice Page 137 of 154

2. During the effectivity of the insurance policy, the insured jeep while being driven by Juan Campollo, an employee of the respondent San Leon Rice Mill collided with passenger bus belonging to the respondent PANTRANCO at the national highway causing damage to the insured vehicle and injuries to the driver and Respondent Martin Vallejos

3. RESPONDENT VALLEJOS filed an action for damages afaist Sio Roy Malayan and PANTRANCO before the RTC.

INSURANCE G01 CASE DIGESTS CA affirmed the decision. They are jointly and severally liable for the damages of Martin Vallejos. However, San Leon Rice Mill has no obligation to indemnify or reimburse the petitioner insurance company since it is not a privy to the contract of insurance between Sio Choy and the insurance company. It must be observed that Respondent Sio Choy is made liable to Vallejos as the owner of the ill- fated Will Jeep pursuant to Article 2184 of the Civil Code. Art. 2184. In motor vehicle mishaps, the owner is solidarily liable with his driver, if the former, who was in the vehicle, could have, by the use of due diligence, prevented the misfortune it is disputably presumed that a driver was negligent, x x x The basis of liability of Respondent San Leon to Vallejos, the former being the employer of the driver of the Willys Jeep at the time of the motor vehicle mishap is Article 2180 of the Civil Code Art. 2180. The obligation imposed by article 2176 is demandable not only for one's own acts or omissions, but also for those of persons for whom one is responsible. Employers shall be liable for the damages caused by their employees and household helpers acting within the scope of their assigned tasks, even though the former are not engaged ill any business or industry. Sio Choy and San Leon Rice Mill are the principal tortfeasors who are primarily liable to Vallejos. The law states that the responsibility of two or more persons who are liable for quasi-delict is solidarily. ON THE OTHER HAND, the basis of petitioner's liability is its insurance contract with respondent Sio Choy. If petitioner is adjudged to pay respondent Vallejos in the amount of not more than P20,000.00, this is on account of its being the insurer of respondent Sio Choy under the third party liability clause included in the private car comprehensive policy existing between petitioner Page 138 of 154

ISSUES: (1) Whether the trial court, as upheld by the Court of Appeals, was correct in holding petitioner and respondents Sio Choy and San Leon Rice Mill, Inc. "solidarily liable" to respondent Vallejos; and

(2) Whether petitioner is entitled to be reimbursed by respondent San Leon Rice Mill, Inc. for whatever amount petitioner has been adjudged to pay respondent Vallejos on its insurance policy.

RATIONALE: In order to determine the alleged liability of respondent San Leon Rice Mill to Malayan, it is important to determine first the nature or basis of the liability of Malayan to Respondent Vallejos, as compared to that of respondents Sio and San Leon Rice Mill. 1. No."ONLY" RESPONDENTS SIO CHOY AND SAN LEON RICE ARE SOLIDARILY LIABLE TO VALLEJOS. MALAYAN IS NOT INCLUDED.

INSURANCE G01 CASE DIGESTS and respondent Sio Choy at the time of the complained vehicular accident 2. Yes. Malayan has the right to be reimbursed. There is a right of Subrogation. Petitioner, upon paying respondent Vallejos the amount of riot exceeding P20,000.00, shall become the subrogee of the insured, the respondent Sio Choy; as such, it is subrogated to whatever rights the latter has against respondent San Leon Rice Mill, Inc. Article 1217 of the Civil Code gives to a solidary debtor who has paid the entire obligation the right to be reimbursed by his co-debtors for the share which corresponds to each. In accordance with Article 1217, petitioner, upon payment to respondent Vallejos and thereby becoming the subrogee of solidary debtor Sio Choy, is entitled to reimbursement from respondent San Leon Rice Mill, Inc. SC hold that only respondents Sio Choy and San Leon Rice Mill, Inc. are solidarily liable to the respondent Martin C. Vallejos for the amount of P29,103.00. Vallejos may enforce the entire obligation on only one of said solidary debtors. If Sio Choy as solidary debtor is made to pay for the entire obligation (P29,103.00) and petitioner, as insurer of Sio Choy, is compelled to pay P20,000.00 of said entire obligation, petitioner would be entitled, as subrogee of Sio Choy as against San Leon Rice Mills, Inc., to be reimbursed by the latter in the amount of P14,551.50 (which is 1/2 of P29,103.00 ) 146. HEIRS OF G.Y. POE V. MALAYAN INSURANCE , 584 SCRA 152 (2009) Heirs of GY Poe vs Malayan Insurance Facts: 1. George Y. Poe (George) while waiting for a ride to work in front of Capital Garments Corporation, Ortigas Avenue Extension, Barangay Dolores, Taytay, Rizal, was run over by a ten-wheeler Isuzu hauler truck 2. The truck was owned by Rhoda Santos (Rhoda), and then being driven by Willie Labrador (Willie). 3. To seek redress for Georges untimely death, his heirs and herein petitioners, namely, his widow Emercelinda, and their children Flerida and Fernando, filed with the RTC a Complaint for damages 4. Malayan Insurance was named respondent in this case since they are the insurer of Rhoda Santos over the subject truck. 5. Under the insurance policy of the said truck Malayan binds itself, among others, to be liable for damages as well as any bodily injury to third persons which may be caused by the operation of the insured vehicle. 6. In their answer, Malayan insurance argued that their liability will only attach if there is a judicial pronouncement that the insured Rhoda and her driver are liable. 7. They of course argued that the accident was caused by the negligence of George. 8. RTC Ruled that Rhoda and Malayan are liable solidarily. 9. CA Set aside the RTC decision. Issue: 1. Whether or not Malayan Insurance should be held solidarily liable with Rhoda. Held: Page 139 of 154

INSURANCE G01 CASE DIGESTS 1. Court rundle that it is up to petitioner to hold them either solidary or joint. Nevertheless, Malayan IS LIABLE 2. Respondent MICI does not deny that it is the insurer of the truck. Nevertheless, it asserts that its liability is limited, and it should not be held solidarily liable with Rhoda for all the damages awarded to petitioners. 3. It is settled that where the insurance contract provides for indemnity against liability to third persons, the liability of the insurer is direct and such third persons can directly sue the insurer. 4. The direct liability of the insurer under indemnity contracts against third party liability does not mean, however, that the insurer can be held solidarily liable with the insured and/or the other parties found at fault, since they are being held liable under different obligations. 5. The liability of the insured carrier or vehicle owner is based on tort, in accordance with the provisions of the Civil Code 6. The liability of the insurer arises from contract, particularly, the insurance policy. 7. The third-party liability of the insurer is only up to the extent of the insurance policy and that required by law; and it cannot be held solidarily liable for anything beyond that amount. 8. In the case of Vda de Maglana vs Colsolacion if indeed Malayan have limited liability under the insurance policy petitioners would have had the option either (1) to claim the amount awarded to them from respondent Malayan, up to the extent of the insurance coverage, and the balance from Rhoda; or (2) to enforce the entire judgment against Rhoda, subject to reimbursement from respondent MICI to the extent of the insurance coverage. 9. However, Malayan failed to present the insurance contract to prove their limited liability. 10. As such there is no more difference in the amounts of damages which petitioners can recover from Rhoda or respondent Malayan; petitioners can recover the said amounts in full from either of them, thus, making their liabilities solidary or joint and several.

147.

VDA. DE MAGLANA VS. CONSOLACION, 212 SCRA 268 (1992)

Vda. De Magdalena vs. Consolacion and AFISCO Insurance Corp (1992) Facts: 1. Lope Maglana was an employee of the Bureau of Customs whose work station was at Lasa, Davao City. On December 20, 1978, early morning, Lope Maglana was on his way to his work, driving a motorcycle, he met an accident that resulted to his death. He died on the spot. 2. The PUJ jeep that bumped the deceased was driven by Pepito Into, operated and owned by defendant Destrajo. 3. From the investigation conducted by the traffic investigator, the PUJ jeep was overtaking another passenger jeep that was going towards the city poblacion. While overtaking, the PUJ jeep of defendant Destrajo running abreast with the overtaken jeep, bumped the motorcycle driven by the deceased who was going towards the direction of Lasa, Davao City. The point of impact was on the lane of the motorcycle and the deceased was thrown from the road and met his untimely death. 4. Maglanas heirs filed an action for damages and attorneys fees against Destrajo and AFISCO Insurance Corporation. An information for homicide thru reckless imprudence was also filed against Pepito Into. 5. The CFI ruled that Destrajo had not exercised sufficient diligence as the operator of the jeepney, ordering him to pay the heirs for damages. AFISCO Insurance Company was ordered to reimburse Destrajo whatever amounts the Page 140 of 154

INSURANCE G01 CASE DIGESTS latter shall have paid only up to the extent of its insurance coverage. 6. Petitioner heirs filed an MR, contending that AFISCO should not merely be held secondarily liable because the Insurance Code provides that the insurer's liability is "direct and primary and/or jointly and severally with the operator of the vehicle, although only up to the extent of the insurance coverage." Hence, they argued that the P20,000.00 coverage of the insurance policy issued by AFISCO, should have been awarded in their favor. 7. AFISCO argued that since the Insurance Code does not expressly provide for a solidary obligation, the presumption is that the obligation is joint. 8. Petitioners reassert their position that the insurance company is directly and solidarily liable with the negligent operator up to the extent of its insurance coverage. Issue: W/N the insurance company is directly and solidarily liable with the negligent operator up to the extent of its insurance coverage Held: Directly liable with the operator, YES. Solidarily liable with the operator, NO. The particular provision of the insurance policy on which petitioners base their claim is as follows: Sec. 1 LIABILITY TO THE PUBLIC 1. The Company will, subject to the Limits of Liability, pay all sums necessary to discharge liability of the insured in respect of (a) death of or bodily injury to any THIRD PARTY .... 3. In the event of the death of any person entitled to indemnity under this Policy, the Company will, in respect of the liability incurred to such person indemnify his personal representatives in terms of, and subject to the terms and conditions hereof.

The above-quoted provision leads to no other conclusion but that AFISCO can be held directly liable by petitioners. As this Court ruled in Shafer vs. Judge, RTC of Olongapo City "where an insurance policy insures directly against liability, the insurer's liability accrues immediately upon the occurrence of the injury or even upon which the liability depends, and does not depend on the recovery of judgment by the injured party against the insured." The underlying reason behind the third party liability (TPL) of the Compulsory Motor Vehicle Liability Insurance is "to protect injured persons against the insolvency of the insured who causes such injury, and to give such injured person a certain beneficial interest in the proceeds of the policy." Since petitioners had received from AFISCO the sum of P5,000.00 under the no-fault clause, AFISCO's liability is now limited to P15,000.00. However, we cannot agree that AFISCO is likewise solidarily liable with Destrajo. In Malayan Insurance Co., Inc. v. Court of Appeals, this Court had the opportunity to resolve the issue as to the nature of the liability of the insurer and the insured vis-a-vis the third party injured in an accident. We categorically ruled thus: While it is true that where the insurance contract provides for indemnity against liability to third persons, such third persons can directly sue the insurer, however, the direct liability of the insurer under indemnity contracts against third party liability does not mean that the insurer can be held solidarily liable with the insured and/or the other parties found at fault. The liability of the insurer is based on contract; that of the insured is based on tort.

Page 141 of 154

INSURANCE G01 CASE DIGESTS The Court then proceeded to distinguish the extent of the liability and manner of enforcing the same in ordinary contracts from that of insurance contracts. While in solidary obligations, the creditor may enforce the entire obligation against one of the solidary debtors, in an insurance contract, the insurer undertakes for a consideration to indemnify the insured against loss, damage or liability arising from an unknown or contingent event. To make the insurer solidarily liable with the latters entire obligation beyond the sum limited in the insurance contract would result in an evident breach of the concept of solidary obligation. Similarly, petitioners herein cannot validly claim that AFISCO, whose liability under the insurance policy is P20,000.00, can be held solidarily liable with Destrajo for the total amount of P53,901.70 in accordance with the decision of the lower court. Since under both the law and the insurance policy, AFISCO's liability is only up to P20,000.00, the second paragraph of the dispositive portion of the decision in question may have unwittingly sown confusion among the petitioners and their counsel. What should have been clearly stressed as to leave no room for doubt was the liability of AFISCO under the explicit terms of the insurance contract. In fine, we conclude that the liability of AFISCO based on the insurance contract is direct, but not solidary with that of Destrajo which is based on Article 2180 of the Civil Code. As such, petitioners have the option either to claim the P15,000 from AFISCO and the balance from Destrajo or enforce the entire judgment from Destrajo subject to reimbursement from AFISCO to the extent of the insurance coverage. 148. GSIS V. COURT OF APPEALS, 308 SCRA 559 (1999)

GSIS vs CA GR No. 101439 June 21, 1999 Facts: National Food Authority (NFA, formerly National Grains Authority) owned a Chevrolet truck which was insured against liabilities for death of and injuries to third persons with GSIS (third-person liability). The truck, driven by Corbeta, collided with a public utility vehicle, a Toyota Tamaraw, owned and operated by Victor Uy, under the name of "Victory Line." The tamaraw was a total wreck and all the collision victims were its passengers. 5 passengers dies, while 10 sustained bodily injuries. 3 cases were filed with CFU of Agusan del Norte and Butuan City: 1. quasi-delict, damages and attorney's fees by Uy; 2. damages by Taer, an injured passenger, against Uy and insurer Mabuhay Insurance (MIGC), to which Uy filed a cross-claim against MIGC and third-party complaint against Corbeta and NFA; 3. actions by heirs and victims of the deceased for damages due to quasi-delict against NFA and Corbeta, against GSIS as insurer of the truck, against Uy for breach of contract of carriage, against MIGC as insurer of the tamaraw. RTC: Corbeta's negligence was the proximate cause of the collision; the truck which crossed over to the other lane was speeding because after the collision, its left front wheel was detached and fell into a ravine. RTC: first case - Uy was awarded 109,100 for damages; 2nd case - dismissed against Uy, ordered MIGC, Corbeta and NFA to pay Taer; 3rd case - damages are also awarded to the heirs and victims. Page 142 of 154

INSURANCE G01 CASE DIGESTS CA affirmed the ruling of RTC who may be negligently injured by the named insured as if such injured person were specifically named in the policy. However, although the victim may proceed directly against the insurer for indemnity, the third party liability is only up to the extent of the insurance policy and those required by law. While it is true that where the insurance contract provides for indemnity against liability to third persons, and such third persons can directly sue the insurer, the direct liability of the insurer under indemnity contracts against third party liability does not mean that the insurer can be held liable in solidum with the insured and/or the other parties found at fault. For the liability of the insurer is based on contract; that of the insured carrier or vehicle owner is based on tort. The liability of GSIS based on the insurance contract is direct, but not solidary with that of the NFA. The latters liability is based separately on Article 2180 of the Civil Code. As therein provided, the maximum indemnity for death was twelve thousand (P12,000.00) pesos per victim. The schedules for medical expenses were also provided by said IMC, specifically in paragraphs (C) to (G). Consequently, heirs of the victims who died in the May 9, 1979 vehicular incident, could proceed (1) against GSIS for the indemnity of P12,000 for each dead victim, and against NFA and Guillermo Corbeta for any other damages or expenses claimed; or (2) against NFA and Corbeta to pay them all their claims in full.

Issue: 1. Whether GSIS should be held solidarily liable with the negligent insured/owner-operator of the Chevrolet truck for damages awarded to private respondents which are beyond the limitations of the insurance policy and the Insurance Memorandum Circular No. 578 2. Whether the private respondents have no cause of action against the petitioner, allegedly for failure of the victims to file an insurance claim within six (6) months from the date of the accident Ruling: 1. The liability of GSIS is direct, but not solidary with that of the owner and negligent driver, and its liability is only to the extent of the insurance policy and those required by law. Compulsory Motor Vehicle Liability Insurance (third party liability, or TPL) is primarily intended to provide compensation for the death or bodily injuries suffered by innocent third parties or passengers as a result of a negligent operation and use of motor vehicles. The victims and/or their dependents are assured of immediate financial assistance, regardless of the financial capacity of motor vehicle owners. The injured for whom the contract of insurance is intended can sue directly the insurer. The general purpose of statutes enabling an injured person to proceed directly against the insurer is to protect injured persons against the insolvency of the insured who causes such injury, and to give such injured person a certain beneficial interest in the proceeds of the policy, and statutes are to be liberally construed so that their intended purpose may be accomplished. It has even been held that such a provision creates a contractual relation which inures to the benefit of any and every person

2. Private respondents have a cause of action. Evidence shows that a notice of loss was sent to GSIS on September 7, 1979, almost 4 months after the incident (May 9, 1979). Likewise, GSIS did not deny receipt of notice of claim during the trial, and it would be too late to state otherwise. Page 143 of 154

INSURANCE G01 CASE DIGESTS A delay in reporting the loss by the insured and/or by the beneficiaries must be promptly raised by the insurer in objecting to the claims. The insurer failed to object to the presentation of the proof of loss before the trial court. The defense of delay should have been promptly interposed. Moreover, the petitioner merely waited for the victims or beneficiaries to file their complaint. As matters stand now, the defense of laches or prescription is deemed waived because of petitioners failure to raise it not only before but also during the hearing. rendered a favorable response for the respondent. The appellate court ruled likewise. Petitioner filed this petition alleging grave abuse of discretion on the part of the appellate court in applying the principle of "expresso unius exclusio alterius" in a personal accident insurance policy, since death resulting from murder and/or assault are impliedly excluded in said insurance policy considering that the cause of death of the insured was not accidental but rather a deliberate and intentional act of the assailant. Therefore, said death was committed with deliberate intent which, by the very nature of a personal accident insurance policy, cannot be indemnified. ISSUE: Whether or not the insurer is liable for the payment of the insurance premiums? HELD: FINMAN GENERAL ASSURANCE CORPORATION vs.THE HONORABLE COURT OF APPEALS FACTS: On October 22, 1986, deceased, Carlie Surposa was insured with petitioner Finman General Assurance Corporation with his parents, spouses Julia and Carlos Surposa, and brothers Christopher, Charles, Chester and Clifton, all surnamed, Surposa, as beneficiaries. While said insurance policy was in full force and effect, the insured, Carlie Surposa, died on October 18, 1988 as a result of a stab wound inflicted by one of the three (3) unidentified men. Private respondent and the other beneficiaries of said insurance policy filed a written notice of claim with the petitioner insurance company which denied said claim contending that murder and assault are not within the scope of the coverage of the insurance policy. Private respondent filed a complaint with the Insurance Commission which Yes, the insurer is still liable. Contracts of insurance are to be construed liberally in favor of the insured and strictly against the insurer. Thus ambiguity in the words of an insurance contract should be interpreted in favor of its beneficiary. The terms "accident" and "accidental" as used in insurance contracts have not acquired any technical meaning, and are construed by the courts in their ordinary and common acceptation. Thus, the terms have been taken to mean that which happen by chance or fortuitously, without intention and design, and which is unexpected, unusual, and unforeseen. Where the death or injury is not the natural or probable result of the insured's voluntary act, or if something unforeseen occurs in the doing of the act which produces the injury, the resulting death is within the protection of the policies insuring against death or injury from accident. In the case at bar, it cannot be pretended that Carlie Surposa died in the course of an assault or murder as a result of his voluntary act Page 144 of 154

149. FINMAN GEN. ASSURANCE V. COURT OF APPEALS, 213 SCRA 493 (1992)

INSURANCE G01 CASE DIGESTS considering the very nature of these crimes. Neither can it be said that where was a capricious desire on the part of the accused to expose his life to danger considering that he was just going home after attending a festival. Furthermore, the personal accident insurance policy involved herein specifically enumerated only ten (10) circumstances wherein no liability attaches to petitioner insurance company for any injury, disability or loss suffered by the insured as a result of any of the stimulated causes. The principle of " expresso unius exclusio alterius" the mention of one thing implies the exclusion of another thing is therefore applicable in the instant case since murder and assault, not having been expressly included in the enumeration of the circumstances that would negate liability in said insurance policy cannot be considered by implication to discharge the petitioner insurance company from liability for, any injury, disability or loss suffered by the insured. Thus, the failure of the petitioner insurance company to include death resulting from murder or assault among the prohibited risks leads inevitably to the conclusion that it did not intend to limit or exempt itself from liability for such death.

Page 145 of 154

INSURANCE G01 CASE DIGESTS

SURETYSHIP
150. PHILIPPINE PRYCE ASSURANCE V. COURT OF APPEALS, 230 SCRA 164 (1994) G.R. No. 107062 February 21, 1994 PHILIPPINE PRYCE ASSURANCE CORPORATION, petitioner, vs. THE COURT OF APPEALS, (Fourteenth Division) and GEGROCO, INC., respondents. NOCON, J.: FACTS: --Petitioner, Interworld Assurance Corporation (now Philippine Pryce Assurance Corporation), was sued for collection of sum of money by respondent Gegroco, Inc. --The complaint alleged that Pryce issued two surety bonds in behalf of its principal Sagum General Merchandise for P500,000.00 and P1,000,000.00 respectively. --Pryce admitted having executed the said bonds, but denied liability because allegedly 1) the checks which were to pay for the premiums bounced and were dishonored hence there is no contract to speak of between Pryce and its supposed principal Sagum; and 2) the bonds were merely to guarantee payment of its principal's obligation, thus, there is a benefit of excussion (a right under Art. 2066 which only a guarantor may invoke against the creditor wherein the guarantor will point out to the creditor all the debtors properties in the Philippines sufficient to cover amount of debt).

(NOTE: Article 2047. By guaranty, a person, called the guarantor, binds himself to the creditor to fulfill the obligation of the principal debtor in case the latter should fail to do so. If a person binds himself solidarily with the principal debtor, the provisions of Section 4, Chapter 3, Title I of this Book (Obligations & Contracts) shall be observed. In such case, the contract is called a suretyship.) --After the issues had been joined, the case was set for pre-trial conference. --Pryce failed to appear during the pre-trial that was reset 3 times because of the non-appearance of Pryce and/or its counsel during the scheduled pre-trials. --Pryce also failed to pay the docket fees for the Third-Party Complaint it filed against his principal Sagum. --Pryce was considered as in default and respondent was allowed to present evidence ex-parte. RTC---ruled in favor of respondent Gregoco. CA---affirmed the RTC ruling. ISSUE: Whether or not there was a contract of suretyship between Pryce and its principal Sagum which makes Pryce solidarily liable to Gregoco HELD: YES. We do not find any reversible error in the conclusion reached by the court a quo. The Insurance Code states that: Sec. 177. The surety is entitled to payment of the premium as soon as the contract of suretyship or bond is perfected and delivered to the obligor. No Page 146 of 154

INSURANCE G01 CASE DIGESTS contract of suretyship or bonding shall be valid and binding unless and until the premium therefor has been paid, except where the obligee has accepted the bond, in which case the bond becomes valid and enforceable irrespective of whether or not the premium has been paid by the obligor to the surety. . . . (emphasis added) The above provision outrightly negates petitioner's first defense. In a desperate attempt to escape liability, petitioner further asserts that the above provision is not applicable because the respondent allegedly had not accepted the surety bond, hence could not have delivered the goods to Sagum Enterprises. This statement clearly intends to muddle the facts as found by the trial court and which are on record. In the first place, petitioner, in its answer, admitted to have issued the bonds subject matter of the original action.19 Secondly, the testimony of Mr. Leonardo T. Guzman, witness for the respondent, reveals the following: Q. What are the conditions and terms of sales you extended to Sagum General Merchandise? A. First, we required him to submit to us Surety Bond to guaranty payment of the spare parts to be purchased. Then we sell to them on 90 days credit. Also, we required them to issue post-dated checks. Q. Did Sagum General merchandise comply with your surety bond requirement? A. Yes. They submitted to us and which we have accepted two surety bonds. Q Will you please present to us the aforesaid surety bonds? A. Interworld Assurance Corp. Surety Bond No. 0029 for P500,000 dated July 24, 1987 and Interworld Assurance Corp. Surety Bond No. 0037 for P1,000.000 dated October 7, 1987. 20 Likewise attached to the record are exhibits consisting of delivery invoices addressed to Sagum General Merchandise proving that parts were purchased, delivered and received. On the other hand, petitioner's defense that it did not have authority to issue a Surety Bond when it did is an admission of fraud committed against respondent. No person can claim benefit from the wrong he himself committed. A representation made is rendered conclusive upon the person making it and cannot be denied or disproved as against the person relying thereon. 22 Petition is DISMISSED for lack of merit. SO ORDERED.

Page 147 of 154

INSURANCE G01 CASE DIGESTS 151. AFP GENERAL INSURANCE V. MOLINA, 556 SCRA 630 (2008) On April 6, 1998, the NLRC affirmed with modification the decision of the Labor Arbiter. The NLRC found the herein private respondents constructively dismissed and ordered Radon Security to pay them their separation pay, in lieu of reinstatement with backwages, as well as their monetary benefits limited to three years, plus attorneys fees equivalent to 10% of the entire amount, with Radon Security and Ever Emporium, Inc. adjudged jointly and severally liable.

AFP General Insurance Corporation vs. Molina 556 SCRA 630 2008 Nature: This is a petition for review on certiorari of the Decision dated August 20, 2001 of the Court of Appeals in CA-G.R. SP No. 58763, which dismissed herein petitioners special civil action for certiorari. Before the appellate court, petitioner AFP General Insurance Corporation (AFPGIC) sought to reverse the Resolution dated October 5, 1999 of the National Labor Relations Commission (NLRC) in NLRC NCR CA-011705-96 for having been issued with grave abuse of discretion. Facts: The private respondents are the complainants in a case for illegal dismissal, docketed as NLRC NCR Case No. 02-00672-90, filed against Radon Security & Allied Services Agency and/or Raquel Aquias and Ever Emporium, Inc. In his Decision dated August 20, 1996, the Labor Arbiter ruled that the private respondents were illegally dismissed and ordered Radon Security to pay them separation pay, backwages, and other monetary claims. Radon Security appealed the Labor Arbiters decision to public respondent NLRC and posted a supersedeas bond, issued by herein petitioner AFPGIC as surety.

Radon Security duly moved for reconsideration, but this was denied by the NLRC in its Resolution dated June 22, 1998. Radon Security then filed a Petition for Certiorari docketed as G.R. No. 134891 with this Court, but we dismissed this petition in our Resolution of August 31, 1998. In dismissing the appeal of AFPGIC, the NLRC pointed out that AFPGICs theory that the bond cannot anymore be proceeded against for failure of Radon Security to pay the premium is untenable, considering that the bond is effective until the finality of the decision. The NLRC stressed that a contrary ruling would allow respondents to simply stop paying the premium to frustrate satisfaction of the money judgment. AFPGIC then moved for reconsideration, but the NLRC denied the motion in its Resolution dated February 29, 2000. AFPGIC then filed a special civil action for certiorari, docketed as CA-G.R. SP No. 58763, with the Court of Appeals, on the

Page 148 of 154

INSURANCE G01 CASE DIGESTS ground that the NLRC committed a grave abuse of discretion in affirming the Order dated March 30, 1999 of the Labor Arbiter. Issue: WON THE COURT OF APPEALS SERIOUSLY ERRED IN SUSTAINING THE PUBLIC RESPONDENT NLRC ALTHOUGH THE LATTER GRAVELY ABUSED ITS DISCRETION WHEN IT ARBITRARILY IGNORED THE FACT THAT SUBJECT APPEAL BOND WAS ALREADY CANCELLED FOR NONPAYMENT OF PREMIUM AND THUS IT COULD NOT BE SUBJECT OF EXECUTION OR GARNISHMENT. Held: The filing of a cash or surety bond is a jurisdictional requirement in an appeal involving monetary award, and the bond shall be in effect until the final disposition of the case. A surety bond, once accepted by the obligee (the employee to whom money benefits were due), becomes valid and enforceable, irrespective of whether or not the premiums thereon have been paid by the obligor (the employer liable for payment). 152. REPARATIONS COM. V. UNIVERSAL DEEP SEA FISHING 83 SCRA 764 (1978) Reparations Commission vs. Universal Deep-Sea Fishing Corp. Nos. L-21901 and L-21996; June 27, 1

Universal Deep-Sea Fishing was awarded 6 trawl boats by the Reparations Commission as end-user of reparations goods. The six fishing boats were delivered to Universal two a time. In each transaction, Universal executed a performance bond with Manila Surety as the Surety in favor of the Reparations Commission. A corresponding indemnity agreement was executed to indemnify the surety company for any damage, loss, charges, etc., which it may sustain or incur as a consequence of having become surety upon the performance bond.

Facts: Universal Deep-Sea Fishing Corporation was awarded six (6) trawl boats by the Reparations Commission as end-user of reparations goods. These fishing boats, christened the M/S UNIFISH 1, M/S UNIFISH 2, M/S UNIFISH 3, M/S UNIFISH 4, M/S UNIFISH 5, and M/S UNIFISH 6, were delivered to Universal two at a time, f.o.b. Japanese port. M/S UNIFISH 1 and M/S UNIFISH 2, with an aggregate purchase price of P536,428.44, were delivered to Universal on November 20, 1958 and the Contract of Conditional Purchase and Sale of Reparations Goods executed by the parties on February 12, 1960 Page 149 of 154

INSURANCE G01 CASE DIGESTS provided among others, that the first installment representing 10% of the amount or P53,642.84 shall be paid within 24 months from the date of complete delivery thereof. The due date of the first installment was on May 8, 1961 and the term is for the payment of purchase price in ten (10) equal yearly installments beginning on May 8, 1962. To guarantee the faithful compliance with the obligations under said contract, a performance bond in the amount of P53,640.00, with Universal as principal and the Manila Surety & Fidelity Co., Inc., as surety, was executed in favor of the Reparations Commission. A Corresponding indemnity agreement was executed to indemnify the surety company for any damage, loss charges, etc., which it may sustain or incur as a consequence of having become a surety upon the performance bond. M/S UNIFISH 3 and M/S UNIFISH 4, with a total purchase price of P687,777.76 were delivered to Universal on April 20, 1959 and the Contract of Conditional Purchase and Sale of Reparation Goods, dated November 25, 1959, provided that the first installment representing 10% of the amount or P68,777.77 shall be paid within 24 months from the date of complete delivery thereof. The due date of the first installment was on July 1961 and the term is for the payment of ten (10) equal yearly installments beginning July, 1962. A performance bond in the amount of P68,777.77, issued by the Manila Surety & Fidelity Co., Inc., was also submitted to guarantee the faithful compliance with the obligations set forth in the contract, and indemnity agreement was executed in favor of the surety company in consideration of the said bond. The delivery of the M/S UNIFISH 5 and M/S UNIFISH 6 is covered by a contract for the Utilization of Reparations Goods executed by the parties on February 12, 1960. The first installment (10% of F.O.B. Cost), P54,500.00, was due October 17, 1961. The term is for the payment of ten (1) equal yearly installments beginning October 17, 1962. A performance bond in judgment, amount of P54,500.00 issued by judgment, Manila Surety & Fidelity Co., Inc., was submitted, and an indemnity agreement was executed by Universal in favor of judgment, surety company. On August 10, 1962, the Reparations Commission filed an action against Universal and Manila Surety to recover various amounts of money due under these contracts. Universal claimed that the amounts of money sought to be collected are not yet due and demandable. Manila Surety also contended that the action is premature, but set up a cross-claim against Universal for reimbursement of whatever amount of money it may have to pay by the plaintiff by reason of the complaint, including interest, and for the collection of accumulated and unpaid premiums on the bonds with interest thereon. It also filed a third-party complaint against Pablo S. Sarmiento, one of the indemnitors in the indemnity agreements. The latter denied personal liability claiming that he signed the agreements in question as his capacity as acting general manage of Universal. The Trial Court rendered judgment ordering Universal to pay the Reparations Commission the amount they were seeking to recover. Manila Surety & Fidelity was also ordered to pay, jointly and severally with Universal, P53,643.00, P68,777.77, and P54.058.00 in favor of the plaintiff. Page 150 of 154

INSURANCE G01 CASE DIGESTS Universal and Pablo Sarmiento were also ordered to pay, jointly and severally, Manila Surety, P53,643.00 and P68,777.77 with 12% interest per year from Aug. 10, 1962 until fully paid plus P2,000.00 as attorneys fees. Universal was likewise ordered to pay Manila Surety P54,508.00 with 12% interest per year from August 10, 1962 until fully paid. 153. ARRANZ V. MANILA FIDELITY, 101 PHIL. 272 (1957)

MELECIO ARRANZ vs. MANILA FIDELITY AND SURETY CO., INC G.R. No. L-9674 FACTS: 1. Manila Fidelity & Surety Co., executed and delivered to the Manila Ylang Ylang Distillery a surety bond, understood to pay jointly and severally with Arranz as principal, the sum of P90,000. The surety bond executed by Arranz and the Manila Fidelity contains the following stipulation: The surety hereunder waives notice of default and expressly agrees that it shall not be necessary for the Manila Ylang Ylang Distillery, Ltd. to proceed against the Principal upon his default or to exhaust the property of said Principal, before proceeding against the surety, the Surety's liability under this bond being a primary one and shall be eligible and demandable immediately upon occurrence of such default. 2. To secure the surety against loss arising from the surety bond, plaintiff executed a second mortgaged over the properties which were transferred by the Manila Ylang Ylang Distillery to Arranza. When the first installment of P50K became due the surety, Manila Fidelity, did not have funds to pay the same, and neither did it have funds to pay the second installment of P40K which became due. 3. Complaint was filed by the Manila Ylang Ylang Distillery and a supplemental complaint was later filed to include the second installment of P40K already due. Manila Fidelity had no funds with which to pay either the P50K or the P40K due under the agreement and the only amount it was able to raise was P20K. And that was paid to Manila Ylang Ylang Distillery on account. Page 151 of 154 April 29, 1957

Issue: Whether the trial court erred in not awarding in favor of Manila Surety the premiums on the performance bonds executed by Universal in favor of the Reparations Commission.

Held: YES. Universal is further ordered to pay the Manila Surety P7,251.42 for the premiums and documentary stamps on the performance bonds.

Ratio:
The payment of premiums on the bonds to the surety company

had been expressly undertaken by UNIVERSAL in the indemnity agreements executed by it in favor of judgment, surety company. The premium is judgment, consideration for furnishing judgment, bonds and judgment, obligation to pay judgment, same subsists for as long as judgment, liability of judgment, surety shall exist.

INSURANCE G01 CASE DIGESTS 4. Manila Fidelity had no money with which to respond for the obligation. Arranz was constrained to mortgage some of his properties to PNB to secure a loan to pay their obligation. PNB, however, required Manila Fidelity to release the second mortgage it obtained from Arranz. Arranz paid the property mortgaged from Manila Fidelity except one in order to release the property from the encumbrance. 5. Arranz then filed a complaint to recover the premiums it paid during the period when the property was under mortgage. He argued that the amounts were never due and owing to the defendant surety and that he paid it against his will in order to be able to save the properties from loss and obtain the credit accommodation from the Philippine National Bank. 6. 7. The TC ruled against Arranz and dismissed the case. Thus, this appeal. ISSUE: WON Arranz is obligated to pay the premium notwithstanding the failure of Manila Fidelity to pay the indebtedness secured by it? YES, he is still obligated to pay. HELD: The premium is the consideration for furnishing the bond or the guaranty. While the liability of the surety to the obligee subsists the premium is collectible from the principal. Under the terms of the contract of suretyship the surety's obligation is that the principal pay the loan and the interest thereon, and that the surety shall be relieved of his obligation when the loan or obligation secured is paid. Now, therefore, if the above abounded Principal shall pay promptly said installments and interest thereon and shall in all respects do and fully observe all and singular the covenants, agreements and conditions as provided for in the aforesaid agreement of November 21, 1949, Annexes "A" and "B" respectively, to the true intent and meaning thereof, this obligation shall be null and void, otherwise, it shall remain in full force and effect. (p. 16, R.O.A..)

As the loan and interest remained unpaid the surety continued to be bound to the creditor-obligee, and as a corollary its right to collect the premium on the bond also continued.

Plaintiff-appellant, therefore, cannot excuse himself from the payment of the premium on the bond upon the failure or refusal of the surety to pay the loan and the interest. Even if, therefore, the payment of the premium were against his will, still plaintiffappellant has no cause of action for the return thereof, because the surety was entitled thereto.

Page 152 of 154

INSURANCE G01 CASE DIGESTS On April 30, 1963 or about five (5) days before the expiration of the liability on the bond, P.D. Marchessini and Co., Ltd. and Delgado Shipping Agencies, Inc., filed a case in the Court of First Instance of Manila against the Philippine Merchants Steamship Co., Inc., Jose L. Bautista, doing business under the name and style of "Ronquillo Trading", and the herein appellant Capital Insurance & Surety Co., Inc. for the sum of $14,800.00 or its equivalent in Philippine currency, for the loss they allegedly suffered as a direct consequence of the failure of the defendants to load the stipulated quantity of 406 U.S. surplus army vehicles. The appellant was made party defendant because of the bond it posted in behalf of the appellees. Upon the expiration of the 12 months life of the bond, the appellant made a formal demand for the payment of the renewal premiums and cost of documentary stamps for another year in the amount of P1,827.00. The appellees refused to pay, contending that the liability of the appellant under the surety bond accrued during the period of twelve months the said bond was originally in force and before its expiration and that the defendants-appellees were under no obligation to renew the surety bond. The appellant, therefore, filed a complaint to recover the sum of P l,827.00 against the appellees in the City Court of Manila wherein said court rendered judgment absolving the appellees from the complaint. The appellant appealed the judgment to the Court of First Instance of Manila where the decision of the city court was affirmed and the complaint dismissed. Its motion for Page 153 of 154

154. CAPITAL INSURANCE V. RONQUILLO TRADING, 123 SCRA 526 (1983)

Capital Surety & Insurance Co. Inc. vs. Ronquillo Trading G.R. No. L-36488 July 25, 1983 Facts: Capital Surety and Insurance Co., Inc., thru its general agent, executed and issued a surety bond in the amount of $14,800.00 or its peso equivalent in behalf of Ronquillo Trading and in favor of S.S. Eurygenes, its master, and/or its agents, Delgado Shipping Agencies. The bond was a guarantee for any additional freight which may be determined to be due on a cargo of 258 surplus army vehicles consigned from Pusan, Korea to the Ronquillo Trading on board the S.S. Eurygenes and booked on said vessel by the Philippine Merchants Steamship Company, Inc. In consideration for the issuance by the appellant of the aforesaid surety bond the appellees executed an indemnity agreement whereby among other things, they jointly and severally promised to pay the appellant the sum of P1,827.00 in advance as premium and documentary stamps for each period of twelve months while the surety bond was in effect.

INSURANCE G01 CASE DIGESTS reconsideration having been denied, appellant filed the instant appeal. or accrued liability so as not to let that liability lapse or expire and thereby bar enforcement. It must be noted that in the surety bond it was stipulated that the "liability of surety on this bond would expire on May 5, 1963 and said bond would be cancelled 15 days after its expiration, unless surety was notified of any existing obligations thereunder." Under this stipulation the bond expired on the stated date and the phrase "unless surety was notified of any existing obligations thereunder" refers to obligations incurred during the term of the bond. Under the Indemnity Agreement, the appellees "agreed to pay the COMPANY the sum of ONE THOUSAND EIGHT HUNDRED ONLY (P1,800.00) Pesos, Philippine Currency, in advance as premium thereof for every twelve (12) months or fraction thereof, while this bond or any renewal or substitution thereof was in effect." Obviously, the duration of the bond was for "every twelve (12) months or fraction thereof, while this bond or any renewal or substitution was in effect." Since the appellees opted not to renew the contract they cannot be obliged to pay the premiums. More specifically, where a contract of surety is terminated under its terms, the liability of the principal for premiums after such termination ceases notwithstanding the pendency of a lawsuit to enforce a liability that accrued during its stipulated lifetime. The appeal was dismissed for lack of merit. The decision of the court a quo was affirmed.

Issue: Wether or not a surety's liability under the bond has accrued, during the period of twelve months the bond was originally in force and before its expiration and that herein appellees were under no obligation to pay the premiums and costs of documentary stamps for the succeeding period it was in effect? Held: Yes. Ratio: The bond was given to secure payment by appellees of such additional freight as would already be due on the cargo when it actually arrived in Manila. The bond was not executed to secure obligation or liability which was still to arise after its twelve month life. While it was true that the lower court held that the bond was still in effect after its expiry date, the effectivity was not due to a renewal made by the appellees but because the surety bond provided that "the liability of the surety will not expire if, as in this case, it was notified of an existing obligation thereunder". The meaning of the bond's still being in effect was that, the suit on the bond instituted by the obligees prior to the expiration of the "liability" thereunder was only for the purpose of enforcing that liability and amounted to notice to appellant of an already existing

Page 154 of 154

You might also like